Vous êtes sur la page 1sur 160

1.

questions: 1- Appearance of skull Xray in MM: Lytic lesions 2- High Ca, High ALP, normal PO4 : 1ry hyperparathyroidism 3-Artery liable to injury during ligation at SFJ (I did it wrong, but I think the correct answer is superficial external pudendal a) 4- Lesion at anal verge, what is the LNs involved 5- Lytic metastatic bone lesion, what is most probable primary :breast 6-pathology associated with Crohn's disease :granulomatous inflammation. 7- Cells forming giant cells: macrophage 8- Colorectal Ca going through to mesentry with 2 LNs involved : Duke C 9- Pancreatic tumor with groin and buttock rash seen at dermatology : glucagonoma 10- Athlete died during footbal game: ?intercerbral Hge/? subarachnoid hge 11-artery involved with patient coming with leg weakness: ? ACA 12- tendon involved in anatomical snuff box: extensor pollicis brevis 13- Patient with enlarged lateral and 3rd ventricles: stenosis at aquecuct of Sylvius. 14- Thyroid carcinoma with cervical LNs metastasis: papillary thyroid CA. 15- Intracranial bleeding with unilateral dilated fixed pupil: trantentorial herniation. 16- Deprssed skull fracture at vertex, which vein involved : SSS 17- Nerve injured in posterior triangle of the neck: spinal accessory N. 18- Golf player had blow to the face, probable EO muscle injury, which investigation: US 19- Inhaled FB, where it will settle: right lower lobe 20- Nerves supplying anal sphincter: S2,3,4 21- Catheterizing a male, what is the tightest part: membranous urethra 22- perinural parotid tumour: ?pleomorphic adenoma 23- Slim tall pregnant lady with chest pain: ? aortic dissection (probable Marfan syndrome) 24- prgnant lady with shock: ? acute massive PE 25- pregnant lady with pleuritic chest pain, haemoptysis: ? pulmonary infarction 26- 19 years old boy post appendecectomy, poor UOP: 500 mls gelo 27- Post stroke gentleman, day 7, not eating: consider PEG tube 28- daily requirment post op patient: 1 L saline+ 1.5 L hartmann's 29- 45 years old gentlema, known Barrett's oes, high grade dysplasia : ? for oesophagectomy 30- Advanced Ca oes with solitar liver met:? stent 31- A lady with known iron defeciency and dysphagia oes varices 32- young lady, chest pain, normal ECG and enzymes: oes spasm 33- Newborn, respiratory distress, trachea shifted, displaced cardiac apex: ? congenital diaphragmatic hernia 34-Newborn, cyanotic, improves with crying: ? subglottic stenosis 35- Newborn, unable to pass NG, air in stomach: congenital oes atresia with tracheo-oes fistula. 36- embryonic origin of right and left pul arteries. 37- Tumour marker for medullary thyroid CA: calcitonin 38- Tumour marker for pheochromocytoma: VMA 39- Prolonged constipation, LIF pain, fever: diverticulitis 40- Drug used in ITU , important in septic shock: noradrenaline 41- Post thyroidectomy teacher, unable to sing: unilateral external

laryngeal N injury 42- Vomitting, abdo pain, swelling at reversal of colostomy site: obstructed incisional hernia 43- Pfannenstiel incision, which layer divided: ? rectus sheath 44- Upper midline incision, which layer divided: linea alba 45- Bullet going through junction of linea semilunaris and costal margin on right side, which structure injured: GB 46- Structure at medial part of femoral ring: lacunar lig 47- pu;sating neck swelling, confirmed by angio: carotid a aneurysm 48- Enlarged tender liver, multiple lesions, calcification: ? hydatid disease 49- 11 years old child, painful scrotal 3 mm swelling, separated from testis: torsion hydatid of Morgagni 50- Man acute scrotum, oedematous, tender hemiscrotum: ? testicular torsion 51- big painless scrotal swelling: hydrocele 52- painless swelling above testis: ?epidydimal cyst 53- 26 years old male, rapidly growing swelling within testis: ? testicular tumor 54- site of ectopic testis: ? base of penis 55- footballer with twisting injury and tenderness just proximal to medial part of knee joint: ? medial collateral lig injury 56- structure felt in PV anteriorly at level fo cervix:? dome of bladder/ ? base of bladder 57- Artery injured in upper chest wall below clavicle: ? thoracoacromial a. o Reply With Quote
Share

Share this post on


Digg Del.icio.us Twitter

2. 09-23-2011 05:24 PM #2
guest2011

Senior Member Join Date May 2011 Posts 1,833 123456Structure liable to injury during fibulectomy: ?peroneal artery Recoprical of absolute risk: ?number needed to treat Sensitivity definition Difference between control and experiment: ?absolute risk Type of fracture in child: ?greenstick Type of fracture in twisting injury of tibia: ?spiral

7- Type of fracture in femur after car accident: I did it oblique (but I think correct answer is transverse) 8- Type of fracture in metastatic/osteoporotic bone: ?transverse 9- Hyperechoic lesion in liver: ?hemangioma 10- Cirrhotic and hep C liver: hepatocellular carcinoma 11- Another liver lesion (cant remember its description): ?metastases 12- Newborn with cyanosis, improves with crying: ?choanal atresia 13- Post thyroidectomy unable to cough and clear throat: superior laryngeal n 14- Neck swelling that appeared before infront of sternomastoid: branchial cyst 15- Neck swelling at base of neck, transilluminates in infant: cystic hygroma 16- Neck swelling moves sideways but not up and down: ?carotid body tumor 17- A boy with septic arthritis in paper 1 18- Theme in paper 2, (I think it was first question on paper) one of them had avascular necrosis and other had SUFE? o Reply With Quote
Share

Share this post on


Digg Del.icio.us Twitter

3. 09-23-2011 05:25 PM #3
guest2011

Senior Member Join Date May 2011 Posts 1,833 Organism causing tonsillitis: S. Pneumoniae Organism causing sinusitis (facial pain and post nasal drip) : S Pneumoniae Patient with lymphedema and infection: what organism? Perianal abscess organism? E coli Breast abscess organism? Staph Organism causing infection with dead tissue and cripitus: C perfringens Gangrene of hallux (3 themes) cant remember them Management of ulcer ... one with bed sores of the heel: conservative or is it debridement (can anyone remember the other 2?) Parasitic infestations, one patient with ova at anal verge: mebendazole ... another patient with ova and cysts in faeces: metronidazole Determinant of cranial blood flow in a patient with low GCS: ?intracranial pressure First response to hge: ?baroreceptors First substance that would directly cause vasoconstriction: Ag II (some of my colleagues say that rennin causes VC?)

Applications Translations Translations Applications Photos Video Groups Events Notes

Chat - Rooms Open Facebook Chat Pop Out Chat Notifications Notifications Chat (Offline) Chat Friend Lists Options

Home Profile Friends Recently Added All Friends Invite Friends Find Friends Inbox79 View Message Inbox (79) Compose New Message

Logout Settings Account Settings Privacy Settings Application Settings Maher Fawzy

Back to professional medical guide

Discussion Board Topic View Start New Topic

Topic:

Reply to Topic Displaying posts 1 - 30 out of 30 by 4 people.

Post #1 1 reply Hala Adel wrote6 hours ago MRCS Part 1 Practice Questions ( Physiology ) - 1 of 3 Correct ADH (Vasopressin) release in response to dehydration causes Single best answer question choose ONE true option only Decreased permeability of the collecting ducts to water

Decreased urine osmolality

Increased Na+ resorption in the ascending limb of the loop of Henle

Increased Na+ resorption in the descending limb of the loop of Henle

Increased permeability of the collecting ducts to water Your answer ADH is released by the posterior pituitary in response to dehydration, from stimulation of osmoreceptors adjacent to the supraoptic nucleus, as well as volume receptors in the aorta atria and great veins. Water absorption in the collecting ducts is independent of sodium concentration, and is under the control of ADH, which causes increased permeability of the ducts. Increased ADH levels will increase the osmolality of the urine via this method.

In the descending limb of the loop of Henle, sodium and water are passively resorbed. The ascending limb is impermeable to water, with active sodium resorption, producing a concentration gradient in the renal medulla, which is essential for the maintenance of water balance. Correct A woman, aged 55, presents with features consistent with Cushings syndrome. She is taking no medication. Her basal cortisol and plasma ACTH levels are significantly raised. She has failed the dexamethasone suppression test.

What is the most likely diagnosis? Single best answer question choose ONE true option only

Adrenal tumour

Carneys syndrome

Cushings disease Your answer Depression

Ectopic ACTH-secreting tumour

A raised adrenocorticotrophic hormone (ACTH) level with a raised cortisol implies the problem is caused by excess ACTH production, otherwise negative feedback would suppress ACTH. A low ACTH level would therefore be expected in patients with an adrenal tumour. Carneys syndrome comprises atrial myxoma and freckles with high cortisol levels independent of ACTH. Pituitary tumours producing ACTH and adrenal stimulation are the cause of Cushings disease. While ectopic ACTH-secreting tumours are associated with significantly raised ACTH and cortisol levels, its unusual to develop classic cushingoid features. Failure of the dexamethasone suppression test can occur in patients with depression, but cushingoid features are not expected. Correct Babinski's sign is produced by: Lateral cerebral sulcus lesions Cerebellar lesions Basal ganglia lesions Lesions of vestibular nuclei Lesions of the pyramidal tracts Your answer Diseases of the pyramidal system cause upper motor neurone lesions. The normal flexor plantar response becomes extensor (a positive Babinskis sign). Correct

A 61-year-old woman with known COPD is about to have a laparoscopic cholecystectomy. Having established a pneumonperitoneum the anaesthetist informs you that he is having difficulty oxygenating the patient. Which of the following factors is most likely responsible for this difficulty? Single best answer question choose ONE true option only Abdominal aorta compression

Increased FEV1 : FVC ratio

Increased peak airway pressures Your answer Increased respiratory rate

Reduced venous return

Laparoscopic surgery is performed through a transperitoneal or retroperitoneal approach with insufflation of CO2 under pressure to create a working space. Cardiovascular, respiratory, renal, and metabolic changes occur secondary to the raised intra-abdominal pressure (IAP) and absorption of CO2.

In reference to the question the raised IAP associated with CO2 insufflation pushes the diaphragm cephalad, reducing diaphragmatic movements. Functional residual capacity, vital capacity, and pulmonary compliance all decrease with raised IAP and peak airway pressures may increase by >50%. These changes are normally well tolerated but patients with underlying lung disease have poor lung compliance and are unable to compensate. These patients often require positive end expiratory pressure in order to achieve adequate gas exchange. Incorrect The infusion of 1 litre of which of the following solutions will initially lead to the greatest increase in extracellular fluid volume?

Single best answer question choose ONE true option only Gelatin colloid solution (e.g. Gelofusin or Haemaccel) Your answer Hypertonic NaCl Correct answer Normal (0.9 %) NaCl

5 % dextrose solution

Pure water

Colloids may be natural (e.g. blood, human albumin and gelatins) or synthetic (e.g. dextrans). They comprise large branching molecules with molecular weights in excess of 30,000. Assuming intact capillary integrity, the volume effects of colloid infusion are, at least initially, confined to the plasma compartment. In contrast, crystalloids, such as NaCl solution, pass more readily from the plasma fluid compartment and have more of a volume effect on the extracellular fluid compartment. In the case of 5 % dextrose solution, the dextrose component is rapidly metabolised and the remaining water distributes itself throughout the entire body water (i.e. intracellular and extracellular compartments).

Therefore, of the options listed above, infusions of NaCl will have the greatest initial increase in extracellular fluid volume. Hypertonic NaCl will have an even greater effect than normal (approximately isotonic) NaCl, since hypertonic solutions will draw additional water from the intracellular fluid compartment by osmosis. Correct In a lung function test, the functional residual capacity:

Single best answer question choose ONE true option only Is the sum of the tidal volume and residual volume

Is the sum of the inspiratory reserve volume, the expiratory reserve volume and the tidal volume

Can be measured directly by spirometry

Is equal to the sum of the residual volume and the expiratory reserve volume Your answer Is that volume of air that remains in the lung after forced expiration

Spirometry traces are easy to understand if you remember the following two rules:

1. There are 4 lung volumes and 5 capacities that you need to remember. 2. A capacity is made up of 2 or more lung volumes

The 4 lung volumes are: * Tidal volume = volume of air inspired or expired with each normal breath in quiet breathing; approximately 500mls. * Residual volume = that volume of air that remains in the lung after forced expiration. * Inspiratory reserve volume = extra volume of air that can be inspired over and above the normal tidal volume. * Expiratory reserve volume = extra volume of air that can be expired by forceful expiration after the end of a normal tidal expiration.

The 5 lung capacities are:

* Functional residual capacity = that volume of air that remains in the lung at the end of quiet expiration. Equal to the sum of the residual volume and the expiratory reserve volume. * Inspiratory capacity = inspiratory reserve volume + tidal volume * Expiratory capacity = expiratory reserve volume + tidal volume * Vital capacity = inspiratory reserve volume + tidal volume + expiratory reserve volume (or total lung capacity residual volume) * Total lung capacity = vital capacity + residual volume

The residual volume (and therefore functional residual capacity and total lung capacity) cannot be measured directly by spirometry. They are measured by either whole body plethysmography, or by using the helium dilution or nitrogen washout techniques. Correct Gluconeogenesis is best described as a process by which?

Single best answer question choose ONE true option only Glucose is generated from carbohydrate precursors Glucose is generated by the breakdown of glycogen stores

Glucose is generated from non carbohydrate sources Your answer Glucose is broken down to Acetyl CoA which enters the tri-carboxylic cycle

Glucagon is generated from carbohydrate precursors Glucose is an essential source of nutrition for the central nervous system and red blood cells. Glycogenolysis (the breakdown of glycogen stores to glucose) will maintain glucose levels for around 8-12 hours after which gluconeogensis will takeover. The main substrates for gluconeogensis include lactate (produced as the result of anaerobic respiration), glycerol (derived from the breakdown of fat) and amino acids (derived from the breakdown of protein). Correct A 70kg woman receives approximately 27% full thickness burns in a house fire to her chest and left arm circumferentially. How much fluid does she require over the initial 24hrs? Single best answer question - choose ONE true option only 1280 ml

2870 ml

3000 ml

5320 ml

7560 ml

Your answer The required fluids may be calculated by the following-

2-4 mls fluid per kg body weight per percent body surface area burns over 24hrs.

The rule of nines is a useful method used estimate the total body surface area (BSA) burns. The adult BSA is divided up into areas of 9% (or multiples of 9%)-

Head, face, arms all equal 9% BSA Chest, back, legs all equal 18% BSA

Thus, this patient BSA burned is 27% (arm 9% and chest 18%)

4 (ml fluid) x 70 (wt in kg) = 280 mls

280mls x 27 = 7560mls fluid requirement.

It must be stressed that the rule of nines only applies to adults as in children the head represents a proportionally larger area. A useful estimation that can be used for any patient is that the palmer surface of the patients hand (including the fingers) represents approximately 1% BSA. Incorrect In a starving patient, which of the following fluid regimens would be most appropriate for a 70kg man over a 24hr period?

Single best answer question choose ONE true option only 3L NSaline with 20mmols potassium chloride in each bag 3L Dextrose-saline

3L Hartmanns solution Your answer 1L NSaline with 20 mmols potassium chloride and, 2L 5% dextrose with 20mmols potassium chloride in each bag Correct answer 3L 5% dextrose with 20mmols potassium chloride in each bag The daily fluid and electrolyte requirements are 1-1.5 mmols Na+ /Kg/24 hours, 1mmols K+ /Kg/24 hours and 40ml H20 /Kg/24 hours. However, additional fluid should be supplemented if there are 3rd space losses (that commonly occur for instance in severe acute pancreatitis, burns and post major gastro-intestinal surgery) and for other sources of fluid loss including vomiting, diuresis and insensible losses Correct

Which of the following statements regarding the flow of air through the airways of the lung is correct?

Single best answer question choose ONE true option only Flow rate is proportional to the length of the airway

Flow rate is proportional to the cube of the radius of the airway

Flow rate is proportional to the viscosity of the gas passing along the airway

Flow rate is inversely proportional to the pressure gradient along the airway

None of the above Your answer This question tests knowledge and physiological application of Poiseuilles Law which states that for a rigid, wide bore tube:-

Fluid flow rate = pr4(DP) 8hL

where: r = radius of the tube, DP is the pressure gradient along the tube, h is the viscosity of the fluid running through the tube and L is the length of the tube.

Therefore, the flow rate is proportional to the fourth power of the radius and the pressure gradient along the tube, but is inversely proportional to the viscosity of the fluid and the length of the tube Incorrect A 77-year-old man presents with a history of vomiting undigested food. Routine biochemistry shows a serum bicarbonate concentration of 38 mmol/l.

Which of the following findings would most suggest that he had a chronic metabolic alkalosis? Single best answer question choose ONE true option only Alkaline urine

Base excess 18 mmol/l

Elevated arterial p(CO2) Correct answer Hypokalaemia Your answer Hypomagnesaemia The base excess provides no additional information: it is directly related to the high bicarbonate concentration. In prolonged metabolic alkalosis, the urine may become acidic, reflecting increased proximal bicarbonate resorption (a consequence of hypochloraemia). Gastric secretions contain about 10 mmol/l potassium and, although potassium depletion is likely to become more severe the longer vomiting occurs, hypokalaemia can develop at any time. However, the development of hypercapnoea as compensation for metabolic alkalosis tends to take some time. Although alkalosis inhibits respiration, the tendency for p(CO2) to increase acts as a respiratory stimulant, though with time, the sensitivity of the respiratory centre to carbon dioxide may decline so that significant hypoventilation does occur. Hypomagnesaemia is frequently found in patients with potassium depletion Correct Which of the following is not associated with a metabolic acidosis?

Single best answer question choose ONE true option only A fall in serum bicarbonate

Ketosis

Hypokalemia Your answer Hypovolaemic shock

Hyperventilation

Serum potassium levels are intimately linked with serum H+ levels via the sodium potassium ATPase. This cell membrane pump principally exchanges intracellular sodium ions with extracellular potassium ions in order to maintain the cell membrane potential. However, potassium ions compete with hydrogen ions in the exchange pump and therefore in the presence of hypokalemia, more hydrogen ions will move into the intracellular compartment via this pump. Conversely, in the presence of hyperkalaemia, less hydrogen ions will move out of the extracellular compartment which result in a metabolic acidosis. Incorrect A 54-year-old woman has undergone some blood tests as part of an employment health screen. She reports she is in good health and, being very health conscious, takes regular vitamin and mineral supplements. She is taking bendrofluazide 2.5 mg for hypertension and her blood pressure is 132/82 mmHg. The only abnormality is a serum calcium concentration of 2.94

mmol/l.

Which of the following is the most likely cause? Single best answer question choose ONE true option only

Diuretic treatment Your answer High dietary calcium intake

High dietary vitamin D intake

Occult malignancy

Primary hyperparathyroidism Correct answer Thiazides can cause hypercalcaemia but it is usually only mild. Vitamin D itself is physiologically inactive and, whereas 1-hydroxylated derivatives can be a cause of hypercalcaemia, vitamin D which has to be metabolised to activate it is less commonly so. Intestinal absorption of calcium is subject to tight control, and a high intake does not cause hypercalcaemia. The two most common causes of hypercalcaemia are primary hyperparathyroidism and malignancy. In an asymptomatic individual, primary hyperparathyroidism is the more likely cause Incorrect Which ECG feature is classically present in hypothermia? Single best answer question choose ONE true option only Thyroxine

Reduced PR interval

Tachycardia Your answer U waves

J waves Correct answer

The J wave may be present on the ECG in patients with hypothermia and is an additional upward peak immediately following the QRS complex. The U wave may be present on the ECG in hypokalaemia and is an additional upward peak which follows the T wave. Tachycardia and a reduction in the RR interval are ECG features of hyperthermia. Incorrect A patient undergoes respiratory function tests. Which of the following are normal readings for a 70-kg man?

Single best answer question choose ONE true option only

Peak expiratory flow of 376 l/min Your answer Total lung capacity of 3.5 litres

Functional residual capacity of 3.5 litres

Tidal volume of 250 ml

Inspiratory reserve volume of 2 litres Correct answer Normal readings for such a patient would be:

peak expiratory flow 520700 l/min total lung capacity 56.5 litres functional residual capacity 23 litres tidal volume 500700 ml

Correct What is the half life of free triiodothyronine (T3) in the blood? Single best answer question choose ONE true option only 1 minute

1 hour

1 day Your answer 1 week

1 month

Most of the T3 and thyroxine (T4) are carried in plasma bound to thyroxine binding globulin, and are inactive in this state. Only 1% of T3 and 0.05% of T4 is free. T3is the active hormone, and is formed from the intracellular deiodination of T4 by type 2 deiodinase. The half life of T4 is 1 week, and of T3 1 day, suggesting that T4 acts as a source of T3, rather than an active hormone in its own right Correct Cerebellar lesions produce: Waddling gait Festinant gait Ataxic gait Your answer Scissors gait High-stepping gait In disease of the lateral cerebellar lobes, the stance becomes broad based, unstable and tremulous. The gait tends to veer towards the side of the more affected cerebellar lobe. Weakness of proximal lower limb muscles (eg in polymyositis or muscular dystrophy) leads to difficulty in rising from sitting or squatting. Once upright, the patient walks with a waddling gait, as each lower limb, as it carries the full weight of the body, does not adequately support the pelvis. Festinant, or hurried gait occurs in Parkinsons disease. Broad-based, high stepping or stamping gait develops in peripheral sensory lesions (eg polyneuropathy) when there is loss of proprioception. Spasticity causes stiffness and jerkiness while walking scissors gait. Correct Which one of the following hormones is secreted by the anterior pituitary?

Single best answer question choose ONE true option only Testosterone

Oxytocin

TSH Your answer CRH

ADH

The pituitary gland (hypophysis) is the conductor of the endocrine orchestra. It is divided into both an anterior part and posterior part. The anterior pituitary (adenohypophysis or pars distalis) secretes 6 hormones namely:

FSH/LH: Reproduction ACTH: Stress response TSH: Basal metabolic rate GH: Growth Prolactin: Lactation

The posterior pituitary (neurohypophysis or pars nervosa) secretes only 2 hormones:

ADH (vasopressin): Osmotic regulation Oxytocin: Milk ejection and labour

Testosterone is produced from Leydig cells in the testis and from the adrenal glands. CRH is produced by the median eminence of the hypothalamus Correct Which of the following systemic effects are most likely to be caused by a space occupying lesion in the brain? Single best answer question choose ONE true option only Bradycardia Your answer Hypotension

Tachycardia

Tachypnoea

Venous ulceration

The cranium is a fixed volume containing blood, CSF and brain tissue in equilibrium. Increases in one component can be compensated by a decrease in the other components without increasing intracranial pressure (the Monroe-Kellie doctrine). Beyond a certain point, this compensation is insufficient, and raised intracranial pressure results (greater than 10-15mmHg).

The effects of raised intracranial pressure are hydrocephalus, cerebral ischaemia (due to decreased cerebral perfusion pressure) and systemic effects. The systemic effects include hypertension, bradycardia, slowed respiration and gastric ulceration (Cushings ulcer). These are thought to be due to autonomic dysregulation resulting from hypothalamic compression. Incorrect Which of the following biochemical parameters would not be useful in distinguishing haemolysis from haemorrhage in an anaemic patient? Single best answer question choose ONE correct option only Serum ferritin Correct answer Serum haptoglobin Serum LDH Bilirubin Serum iron Your answer With haemolysis, iron is recycled by combining with serum haptoglobin which falls as a result. Patients with haemolytic states do not therefore become iron deficient, unlike patients who are bleeding who lose on average 1mg of iron with every mL of blood. Unconjugated bilirubin is marker of haemolyis and is generated by the breakdown of the Haem ring from haemoglobin. In addition, LDH is released from red blood cells if haemolysis is intravascular. Incorrect Which of the following physiological characteristics relates to the lining of the respiratory tract? Single best answer question choose ONE true option only About 1 litre of mucus is produced every day

The cilia are under the control of a physiological motor, dynein Correct answer The mucociliary escalator moves at 0.2 cm/minute

The bronchioles have cartilage in their wall

The bronchioles have diameters up to 5 mm Your answer About 100 ml of mucus is produced every day. The cilia are under the control of a physiological motor, dynein (which is absent in Kartageners syndrome). The mucociliary escalator moves at 2 cm/minute. The bronchioles do not have cartilage in their wall (which distinguishes them from bronchi). The bronchioles can be up to 1 mm in diameter Correct The Chief cells of the stomach produce which of the following substances? Single best answer question choose ONE true option only Gastric acid Intrinsic factor Pepsinogen Your answer Mucus Somatostatin Chief cells produce pepsinogen which is a precursor and is activated to pepsin by gastric acid.

Pepsin digests protein. Gastric parietal cells produce gastric acid hydrochloric acid. Intrinsic factor is also produced by parietal cells and is necessary for vitamin B12 absorption in the terminal ileum. Mucus cells produce mucus which forms a protective layer over the gastric mucosa preventing autodigestion. Incorrect Hypothyroidism due to disease of the thyroid gland is associated with increased plasma level of? Single best answer question choose ONE true option only Cholesterol Correct answer Albumin

RT3

Iodide

Thyroid binding globulin (TBG) Your answer Thyroid hormone lowers circulating cholesterol level. The plasma cholesterol level drops before the metabolic rate rises Correct Regarding the clinical physiology of the adrenal gland in Cushings disease, which of the following pertains? Single best answer question choose ONE true option only

The zona glomerulosa of the cortex is predominantly responsible for sex steroid production

The zona fasciculata is predominantly controlled by ACTH and is often hypertrophied A 24-year-old woman undergoes resection of the terminal ileum with fashioning of an ileostomy for Crohns disease. Some 2 weeks after surgery, she is making a good recovery, and is eating a high-energy, low-residue diet, but has a high ileostomy volume, necessitating intravenous fluid replacement. Her serum calcium concentration is 1.82 mmol/l, phosphate 1.28 mmol/l, alkal Reply to HalaReport

Post #2 Hala Adel wrote6 hours ago Your answer The zona reticularis is predominantly responsible for mineralocorticoid production

About 15% of glucocorticoid production takes place in the adrenal medulla

The zona fasciculata is primarily responsible for mineralocorticoid production

The zona glomerulosa of the cortex is predominantly responsible for mineralocorticoid production, the zona fasciculata for glucocorticoid production and the zona reticularis for sex corticoid production. The adrenal medulla originates from the neural crest and hence there is almost complete demarcation of function, with the medulla being responsible for the production of catecholamine-related compounds Incorrect Which of the following metabolic effects is most likely to be caused by thyroid hormone? Single best answer question choose ONE true option only Decreased glycogenolysis in the liver

Increased glucose absorption in the gut Correct answer Decreased lipolysis

Decreased expression of adrenergic receptors Your answer Decreased oxygen uptake in the mitochondria

Thyroid hormone has widespread metabolic effects. Increased glycogenolysis in the liver, increased glucose absorption in the gut and increased insulin breakdown all tend to increase blood glucose. The glycogenolytic effects of catecholamines are also potentiated. These effects can make the diagnosis and management of diabetes in thyrotoxicosis difficult. There is an overall lipolytic effect, with decreased serum cholesterol seen in thyrotoxicosis, and an increase in hypothyroidism. There is an increased expression of b-adrenergic receptors in many tissues including skeletal and cardiac muscle. There is a positive inotropic effect with increased cardiac output and heart rate. A raised metabolic rate and increased heat production are due to increased oxygen uptake and ATP production in the mitochondria. There are also effects on bone, with an overall breakdown of bone, sometimes leading to hypercalcaemia. Increased serum 2,3 DPG leads to a right shift of the haemoglobin dissociation curve. Thyroid hormones are also essential for fetal development, with deficiency leading to cretinism. The fetus produces its own hormone from 18 weeks of gestation. Correct A patient in the intensive care unit following liver transplant surgery has a metabolic alkalosis.

Which of the following biochemical abnormalities is MOST specifically indicative of this? Single best answer question - choose ONE true option only

Acidic urine

High arterial blood pH (low hydrogen-ion concentration)

High arterial partial pressure of carbon dioxide p(CO2)

High plasma bicarbonate concentration Your answer Hypochloraemia

Arterial pH is increased in both metabolic and respiratory alkalosis: plasma bicarbonate is always increased in metabolic alkalosis and can be low in chronic respiratory alkalosis. A high p(CO2) can occur in metabolic alkalosis as a result of respiratory compensation, but it is also a feature of respiratory acidosis. Although the urine may become paradoxically acidic in metabolic alkalosis, it is normally acidic, except sometimes immediately following a meal. Hypochloraemia is present in metabolic alkalosis due to a loss of gastric acid, but may not occur with alkalosis from other causes. Correct Which of the following is NOT a characteristic of the loop of Henle? Single best answer question choose ONE true option only Is under the control of aldosterone Is permeable to water and electrolytes along its distal limb Proximal limb absorption is isotonic Generates high osmolality in the renal medulla Is impermeable to water along its distal limb Your answer The loop of Henles main function is to produce a high medullary osmolality which is the driving force for water reabsorption from the collecting ducts. In the loop of Henle there is a concentration and reduction in volume of filtrate as sodium and chloride pass into the descending limb and water is osmotically moved out. In the ascending limb there is active reabsorption of sodium chloride from the filtrate producing a low osmolality filtrate. Aldosterone acts on the distal convoluted tubules and collecting ducts. Correct Which of the following hormones is synthesised in the hypothalamus and secreted from the posterior pituitary? Single best answer question choose ONE true option only Anti diuretic hormone (ADH) Your answer Adrenocorticotrophic hormone (ACTH)

Corticotrophin releasing hormone (CRH)

Thyrotrophin releasing hormone (TRH)

Thyroid stimulating hormone (TSH)

Vasopressin (ADH) and oxytocin are synthesised in the hypothalamic nuclei and pass down axons to the posterior pituitary where they are secreted into the blood stream.

In contrast, the trophic hormones such as CRH and TRH are secreted by the hypothalamus in response to neural stimuli, and drain into the hypothalamohypophyseal portal vessels to the anterior pituitary. There is then resultant stimulation of ACTH and TSH secretion. The other hormones produced by a similar mechanism by the anterior pituitary are growth hormone (GH), prolactin (PRL), lutenising hormone (LH) and follicle stimulating hormone (FSH). Incorrect A 21-year-old male medical student who has been feeling non-specifically unwell for several days is noticed to have slightly icteric sclerae by his girlfriend and has liver function tests performed. The results of these are normal apart from a serum bilirubin concentration of 44 mmol/l (317). His urine does not contain bilirubin.

Which of the following is the most likely diagnosis? Single best answer question choose ONE true option only

DubinJohnson syndrome

Gilberts syndrome Correct answer Hereditary spherocytosis

Infectious mononucleosis Your answer Rotor syndrome

DubinJohnson, Rotor and Gilberts syndromes are all inherited disorders of bilirubin metabolism.

However, in the first two, there is a defect in the secretion of bilirubin from the liver and the bilirubin that accumulates in the plasma is conjugated, water-soluble and thus is excreted in the urine. Infectious mononucleosis can cause hepatitis and jaundice but an elevated transaminase activity would be expected. Hereditary spherocytosis is a chronic haemolytic disorder due to a defect in the red cell membrane (most frequently in spectrin, a structural protein). It can present with a wide range of severity, from jaundice at birth to asymptomatic anaemia or jaundice in adults, but is much less common (approximately 1:5000 in Northern Europeans) than Gilberts syndrome (approximately 1:20). Correct Which of the following is NOT a defining feature of the systemic inflammatory response syndrome (SIRS)?

Single best answer question choose ONE true option only Temperature >37.5 oC Your answer Heart rate >90/min

Respiratory rate >20/min

PaCO2 <32 mmHg (4.3 kPa)

White blood cell count of >12 x109/l

SIRS is the syndrome arising from the bodys inflammatory reaction to a damaging insult such as infection, trauma, burns or acute pancreatitis. SIRS is recognised by the presence of the following clinical criteria: Temperature >38 oC or <36 oC Heart rate >90/min Respiratory rate >20/min or PaCO2 <32 mmHg (4.3 kPa) White blood cell count of >12 x109/l, or <4 x109/l, or the presence of >10 % immature forms Correct Which of the following statements fulfil the criteria for the correct definition of sepsis?

Single best answer question choose ONE true option only The presence of micro-organisms in the blood stream The presence of micro-organisms within a normally sterile viscus

Hypotension refractory to resuscitation in the presence of demonstrable infection

A systemic inflammatory response occurring as a direct result of infection Your answer A raised respiratory rate , a high white cell count and the presence of a proven source of

infection

The American College of Chest Physicians and the Society of Critical Care have defined sepsis as a systemic inflammatory response syndrome (SIRS) as the result of a confirmed infectious process. The SIRS is defined when two of the following are present: * * * * Pyrexia Tachycardia Tachypnoea A raised white cell count

As a SIRS can occur secondary to non infectious causes (e.g. trauma, malignancy), sepsis is defined as a SIRS occurring as a direct result of infection. Incorrect The 3' 5' exonuclease activity possessed by some DNA polymerases that enables the enzyme to replace misincorporated nucleotide is called what?

Single best answer question choose ONE true option only

Proofreading Correct answer Replication

Recombination Your answer Retrotransposition

Splicing

Retrotransposition is transposition via an RNA intermediate (transposition is the movement of a genetic element from one site to another in a DNA molecule). Splicing is the removal of introns from the primary transcript of a discontinuous gene. Correct Which of the following is produced by the duodenum?

Single best answer question choose ONE true option only Cholecystokinin Secretin Your answer Amylase Lipase Elastase The duodenum secretes secretin in response to acid chyme from the stomach. Secretin promotes production of water and bicarbonate from the pancreatic duct cells. Incorrect A 25-year-old man is admitted to hospital with persistent vomiting. He is clinically dehydrated and hypotensive. His serum sodium concentration is 124 mmol/l, potassium 4.9 mmol/l, urea 9.8 mmol/l, creatinine 96 mmol/l. Urine sodium concentration in a specimen passed on admission is 62 mmol/l.

Which of the following is the most likely cause of the hyponatraemia? Single best answer question choose ONE true option only

Adrenal failure Correct answer Cerebral salt wasting

Gastrointestinal fluid loss Your answer Low sodium intake

Syndrome of inappropriate antidiuresis (SIAD)

Natriuresis in a dehydrated, hyponatraemic patient suggests that there is uncontrolled renal loss of sodium, such as occurs in adrenal failure. Cerebral salt wasting can also cause dehydration and hyponatraemia due to excessive natriuresis, but typically occurs following a head injury or brain surgery. Hyponatraemia and dehydration due to gastrointestinal fluid loss or sodium deficiency due to a low intake should lead to renal conservation of sodium. Although SIAD is an important cause of hyponatraemia and sodium excretion may be high, the hyponatraemia is due to water excess and patients are not dehydrated. Correct You are called to ICU to see a 65-year-old patient who requires controlled mechanical ventilation after major non-cardiac surgery but is becoming hypoxaemic when the FiO2 is reduced from 0.4 to 0.3.

Which of the following statements is true? Single best answer question choose ONE true option only

Simple indices of circulatory status such as urine output, blood pressure and CVP correlate well with outcome from high-risk surgery

Survivors after major surgery decrease their cardiac index and oxygen delivery in the perioperative period below baseline normal values

Measurement of mixed venous oxygen saturation (SVO2) requires a pulmonary venous (PV) catheter to sample pulmonary capillary blood

Cardiac index and oxygen delivery correlate poorly with outcome from high-risk surgery

Pre- or perioperative beta-blockade can improve survival after major non-cardiac surgery in patients with pre-existing cardiac disease Your answer Simple indices of circulatory status such as urine output, blood pressure and CVP correlate poorly with outcome from high-risk surgery. Survivors after major surgery increase their cardiac index and oxygen delivery in the perioperative period above baseline normal values. Measurement of mixed venous oxygen saturation (SVO2) requires a pulmonary artery (PA) catheter to sample pulmonary capillary blood. Cardiac index and oxygen delivery correlate well with outcome from high-risk surgery. Two recent multicentre trials have confirmed the advantage of using highly selective pre- or perioperative beta-blockade to improve survival after major non-cardiac surgery in patients with pre-existing cardiac disease, eg previous heart failure, moderate hypertension and myocardial infarction (MI). The regime is started 4872 h preoperatively and continued for 1428 days post-surgery. Correct A 75 kg man has suffered acute loss of 25 % of his blood volume, has a pulse rate of 110/min, a ventilatory rate of 25/min and a urine output of 25 ml/h. Which class of haemorrhagic shock most appropriately describes this patient?

Single best answer question choose ONE true option only Class I haemorrhagic shock

Class II haemorrhagic shock Your answer Class III haemorrhagic shock

Class IV haemorrhagic shock

None of the above

The patient exhibits signs of class II haemorrhagic shock. ATLS guidelines classify haemorrhagic shock into 4 categories as shown in the table below:-

Class Class I Class II Class III Class IV Blood loss (ml) <750 750 - 1500 1500 - 2000 >2000 % blood vol lost <15% 15 - 30% 30 - 40% >40% Pulse rate (min) <100 >100 >120 >140 Systolic BP Unchanged Unchanged Decreased Decreased Diastolic BP Unchanged Increased Decreased Decreased Pulse Pressure Unchanged Decreased Decreased Decreased Urine output (ml/h) >30 20 - 30 5 - 15 Anuria CNS features Slight anxiety Mild anxiety Anxiety/Confusion Confusion Correct Myeloid stem cells give rise to several different cell types. Which of the following is not one of these? Single best answer question choose ONE correct option only Neutrophils Monocytes Platelets Lymphocytes Your answer Macrophages Bone marrow produces pluripotential stem cells which give rise to two lines of cells myeloid and lymphoid stem cells. Myeloid stem cells differentiate into the polymorphonuclear leucocytes neutrophils, eosinophils and basophils. It also gives rise to monocytes and macrophages. The lymphoid stem cell line produces lymphocytes, both T and B types. Incorrect A 75-year-old woman is being followed by her GP for suspected developing primary hypothyroidism.

Which of the following biochemical changes would you most expect to occur first? Single best answer question choose ONE true option only

Fall in serum free thyroxine Your answer Fall in serum thyroxine-binding globulin

Fall in serum free triiodothyronine

Fall in serum total triiodothyronine

Increase in serum TSH Correct answer Hypothyroidism develops gradually, often over many months or even years. In the early stages, free thyroxine concentrations are maintained in the normal range by the increased secretion of TSH. Patients with a slightly elevated TSH and lownormal thyroxine are said to have compensated or borderline hypothyroidism. In some individuals, it appears that this state can be maintained without progression to frank hypothyroidism. Triiodothyronine concentrations tend to fall later than thyroxine concentrations in hypothyroidism; the concentration of thyroxine-binding globulin does not change significantly Incorrect In estimating the physiological clearance of 10 ml of an intravenous substance which has been administered at 10 mg/ml, the plasma concentration at equilibration is 15 mg/litre, the urine concentration is 150 mg/litre and the subject produces 1440 ml of urine during a 24h collection.

What is the clearance of the substance? Single best answer question choose ONE true option only

1 ml/min

10 ml/min Correct answer 0.1 ml/min

100 ml/min

Cannot say from the information given Your answer Clearance is calculated using the formula (U V)/P where U = urine concentration in mg/ml, V = urine production in ml/min, P = plasma concentration in mg/ml. The bolus size of the substance is irrelevant to the clearance. Incorrect Which of the following organs has the greatest blood flow per 100 g of tissue? Single best answer question choose ONE true option only Brain

Heart

Skin

Liver Your answer Kidneys Correct answer

Organ Blood flow in ml/100g/min Kidneys 420.0 Heart 84.0 Liver 57.7 Brain 54.0 Skin 12.8 Incorrect The action potential of skeletal muscle? Single best answer question choose ONE true option only Has a prolonged plateau phase

Spread inwards to all parts of the muscle via the T tubes Correct answer Causes immediate uptake of Ca into the sarcoplasmic reticulum Your answer Is longer than the action potential of cardiac muscle

Is not essential for contraction

The action potential of the skeletal muscle spreads out from the motor end plate, through the T tube system this causes mobilization of Ca2+ from the sarcoplasmic reticulum to the cytoplasm and this action potential is essential for contraction.

The action potential of cardiac muscle is longer than that of the skeletal muscle and has plateau phase. Incorrect Botulinum toxin has which of the following features? Single best answer question choose ONE true option only

It is produced by a Gram-positive, aerobic bacillus

The bacillus has 15 serotypes

Its main activity is at the presynaptic membrane Your answer It may be used in the treatment of myasthenia gravis

It may be used in the treatment of blepharospasm Correct answer Clostridium botulinum is a Gram-positive, spore-forming, obligate anaerobe. The bacillus has seven serotypes, A to G. They have a wide range of therapeutic usage, from glabellar lines, blepharospasm, spasticity, anismus, anal fissure to dystonia. However, myasthenia gravis would be expected to worsen with such treatment. Incorrect A 34-year-old woman with a body mass index of 44 kg/m2 seeks medical help for her obesity.

Which one of the following treatments offers her the highest probability of achieving a long-term reduction in weight? Single best answer question choose ONE true option only

An energy-deficient diet (600 kcal/day (~ 143 J/day) less than requirements) for 6 months

Jaw-wiring and milk feeding for 3 months

Treatment with orlistat for 12 months

Treatment with sibutramine for 12 months Your answer Vertical banded gastroplication Correct answer Both sibutramine and orlistat have been shown to induce and maintain a greater weight loss than diet alone, but a patients weight often plateaus before adequate weight loss has occurred. Orlistat is only licensed for use for 1 year in the UK, and sibutramine for 2 years. Energydeficient diets, particularly if coupled with increased exercise, are effective, but the lost weight is almost invariably regained, as it is after jaw-wiring and milk feeding. Surgery offers the best chance of achieving long-term weight loss, the results from vertical banded gastroplication combined with a by-pass procedure being even better than those with gastroplication alone Correct A 32-year-old woman on nasogastric aspiration for paralytic ileus following surgery develops a metabolic alkalosis.

Which of the following intravenous fluids would be the preferred treatment for the alkalosis? Single best answer question choose ONE true option only

5% dextrose

Dextrose saline

Normal (0.9%) saline Your answer

Ringers lactate

Twice normal (1.8%) saline

The metabolic alkalosis secondary to a loss of gastric acid is a hypochloraemic alkalosis. This is perpetuated by the hyperchloraemia, which prevents renal excretion of the excess bicarbonate since its proximal tubular reabsorption (with sodium) is enhanced. Provision of adequate chloride ions allows the excess bicarbonate to be excreted and corrects the alkalosis. Dextrose 5% contains no chloride and dextrose saline contains insufficient for this purpose. Twice normal saline is occasionally used for treating severe hyponatraemia but has no place in this clinical situation. Ringers lactate is inappropriate, since the metabolism of the lactate that it contains to bicarbonate would exacerbate the alkalosis. Correct The actions of active Vitamin D include all of the following except:

Single best answer question choose ONE true option only Increased calcium excretion from the kidneys Your answer Inhibition of PTH release from the parathyroid glands

Increased phosphate absorption from the intestines

Increased calcium absorption from the intestines

Increased bone mineralisation

Active Vitamin D plays a crucial role in the homeostasis of calcium. Whether the source is from the skin or dietary, 25- and 1-alpha hydroxylation is required in the liver and kidney respectively to convert Vitamin D into its metabolically active form. Its main function in calcium homeostasis includes an increase in the absorption of both calcium and phosphate from the gut and the stabilisation and the promotion of mineralization in bone. It also acts directly on the parathyroid gland to inhibit the release of PTH. This provides a negative feedback mechanism as PTH is required for hydroxylation of Vitamin D in the kidney. Incorrect You are asked to see a patient who had a chest drain removed 4 days ago. There appears to be some infection.

What are the stages in the cell biology of normal wound healing? Single best answer question choose ONE true option only

Demolition is the first phase Your answer Maturation and remodelling can continue for up to a year Correct answer Acute inflammation usually lasts for 612 hours

Epithelial cell proliferation is the hallmark of the demolition phase

Collagen deposition is the key process during demolition

The first phase in healing by first intention is the phase of acute inflammation that lasts up to 3 days, if uncomplicated. The initiating factor appears to originate from platelets activated by mature collagen exposed in the wound. Platelets first aggregate then release a variety of active agents including lysosomal enzymes, ATP, serotonin and wound cytokines. A fibrin clot develops, which completes haemostasis and provides strength and support to the wound. The surface dries to form a scab. Platelets and macrophage factors cause local vasodilatation, which produces warmth and increases capillary permeability, allowing serum and white blood cells to accumulate and cause swelling. After the initial acute inflammation, macrophages become active as the main agents of demolition, removing unwanted fibrin, dead cells and bacteria and creating fluid-filled spaces for granulation tissue. Macrophages also release factors that stimulate the formation of new capillary buds during this phase, and later they initiate and control fibroblast activity during repair. Within the connective tissue, randomly orientated collagen begins to form after a few days, reaching a peak of activity after 57 days. Epithelial cells at the edge of the wound start to proliferate after 24 h and this phase can last for up to 3 weeks. Reply to HalaReport

Post #3 Hala Adel wrote6 hours ago Finally, the phase of maturation and remodelling lasts for up to 12 months, during which time the tensile strength of the wound increases and the random collagen is replaced by a more stable form orientated along lines of stress. Incorrect Which of the following is the most important direct stimulus to respiration? Single best answer question choose ONE true option only Increased pCO2 of the CSF

Your answer Increased H+ concentration of the CSF Correct answer Decreased arterial pO2

Decreased arterial pH

Decreased arterial pCO2

Chemoreceptors involved with the control of respiration are present in the central nervous system and peripherally. The central chemoreceptors are situated in the ventral medulla, and increase firing in response to the H+ concentration of the brain extra cellular fluid, which is directly related to the H+ concentration in the CSF. CO2 / HCO3 cannot cross the blood brain barrier, but CO2 does so readily. This frees H+ ions, causing a low CSF pH, increased firing of the central chemoreceptors and increased ventilation.

Peripheral chemoreceptors are found in the carotid bodies and aortic arch, and increase their firing rate in response to decreased PaO2, decreased arterial pH and increased paCO2. These are much less important, however, in stimulating respiration than the central chemoreceptors. Incorrect What is the term for the volume of expired air at forced expiration? Single best answer question choose ONE true option only Forced vital capacity (FVC) Correct answer Functional residual capacity (FRC) Expiratory reserve volume (ERV) Your answer Residual volume (RV) Total volume (TV) FVC is the amount of air expelled during forced expiration and has clinical significance. It is reduced in restrictive disease. FRC is the volume of gas left in the lung at the end of quiet respiration. ERV is the maximum volume of expired air. RV is the volume of air remaining in the lungs after forced expiration (FRC). TV is the total volume of air in the lungs and includes the residual volume. Incorrect Reply to HalaReport

Post #4 Hala Adel wrote6 hours ago MRCS part I 10 sept 2007 recalls these are the questions i able to remember. SBA questions 1) young man has pelvic fracture ---> sudden onset of acute urinary retention. What is the possible cause? a) urethral injury b) bladder rupture c) ureter injury

2) Causes of raise PSA a) Prostatic ca b) prostitis 3) Bee sting, presented with HR 120, BP 60/40 first treatment.... a) IV antihistamine b) IV fluid c) IV steroid d) local antihistamine e) s/c adrenaline 4) noradrenaline binds to... a) a1 receptor b) a2 receptor c) b1 receptor d b2 receptor 5) post op developed high glucose level. prior to op, pt is not DM. This is due to ..... GH secretion post-op? 6) pt has splenic rupture. denied any trauma. what infection can cause spenic rupture? a) EBV b) mumps c) measles 7) recurrent UTI, pneumuria, and irregular bowel habits. CT shown mass involved both the sigmoid and bladder. a) diverticulitis b) sigmoid ca c) Crohn's ds d) UC loss of appetite, malaise, multiple lymphadenopathy involvement...axillary, inguinal a) malignant lymphoma 9) what is the course of median nerve related to brachial a. .......from medial to ant to lat to brachial a. 10) a cut above the ulnar olecranon cause unbale to extend UL. what is the tendon being cut? ---> tricep tendon 11) sprinting duirng playing football --> pain and post of the thigh. later severe pain and the lat side of the knee. unable to extend knee dt pain. what is the tendon being involve? --> tendon of biceps femoris 12) the first structure being noted after open up the popliteal fossa a) popliteal vein b) femoral n c) popliteus 13) accident --> multiple tibial and fibula # --> intramedullary nailing done. 6 hours later ---> severe pain at leg a) DVT b) compartment syndrome 14) malaise, weight loss, cervical lymp nodes. biopsy --> epitheliod macrophages and giant cell ----> TB 15) the border of the snuff box a) extensor pollicis longus b) abductor pollicis brevis c) flexor digitorum longus 16) cut at the midline between the base of the little finger and the wrist ---> cause loss of thumb adduction power what is the nerve being injured? a) superficial ulnar n b) deep ulnar n c) medial nerve

d) radial n 17) after the varicose vein surgery, loss sensation and the dorsal of the foot, unable to dorsiflex the foot what is the nerve being injured? a) common peroneal n b) sup peroneal n c) deep peroneal n 1 2-days old neonates, cyanoses at the LL. Weak pulse and LL. BP 60/40 and both UL. what is the abn a) pulmonary atresia b) aortic arch abn c) VSD with pul stenosis 19) trauma to the chest, CXR shown widening of the mediastinum. what of the structure being rupture? ascending aorta descending aorta 20) a knife penetrate the midline of the sternal angle with injure a) trachea b) oesophagus c) sup vena cava d) azygos V 21) IN surgical ICU, pt develop metabolic acidosis. what is the most common cause? a) vomiting b) nasogastric aspiration i will post later....pls other whom remember pls post Reply to HalaReport

Post #5 Hala Adel wrote6 hours ago what is the acute management for gaining a airway in acute resp distress? a) chest tube b) needle thro the cricothyroid membrane fresh blood noted at the chest tube, the bleeding is from? a) intersostal a b) pericardiophrenic a c) r ventricular insulin dependent, h/o chest infection started with antibiotic, admitted with drowsiness --> DKA what is the electrolyte imbalance ------> hyperkalemia non-alcoholic, with palpable nodular liver, biopsy confirm is HCC. what is the cause for the patient --> HBV cirrhosis 2 degree partial thickness burn, developed bilaterally LL swelling. what is the cause? ------> hypoalbuminemia hlo back pain, walking cause pain at the L LL, loss os the sensation over the surface of the knee, what is the cord lesion ----> L3 profuse LGIB, contrast accumulate at the left iliac fossa, for vessel embolisation, which level of artery is cannulate? ----> L3 ( inferior mesenteric artery ) during prolapsed interventricular disc, what is the structure compressed on the nerve? --> nucleus pulposus after the mastectomy, the woman has a wing scapula, what is the nerve being injured? -----> long thoracic nerve Reply to HalaReport

Post #6 Hala Adel wrote6 hours ago woman presented with the lump at ant neck, move with swallowing, FNAC done confirm is malignancy......what is the CA --> papillary thyroid ca a surgery done for the sweating palm dissert at the base of the neck ant to the first rib, what is the complication --> phrenic nerve injury ? R diaphragm elevation during hypotensive shock what is the first substances to be secreted -- angiotensinogen -- angintensinI -- angintensin II -- aldosterone -- renin a man hav a trauma over the medial part of the thigh, clean wound, closure done. few day later patient developed pulsatile mass... ----> false aneurysm of the femoral a. hip replacement woman, walk with the tredelenburg gait. what is the defect? -- sciatic nerve -- gluteus medium -- femoral nerve what is the mechanism of the counter-current in the nephron for the concentration of the urine? --> impermeability of the thick ascending for the water Reply to HalaReport

Post #7 Hala Adel wrote6 hours ago MRCS09 JAN 13TH QUESTION/THEMES [b]1. WHAT IS THE ANTERIOR BOUNDARY OF FEMORAL CANAL 2. WHAT IS THE POSTERIOR WALL OF FEMORAL CANAL 3. DIRECT INGUINAL HERNIA IS THE WEAKNESS OF WHICH WALL NAME 4. anterior surface of heart is formed by 5. heart valve is made of 6. MEN II b comprises of 7. medullary carcinoma presnts as 8. 1st web is supplied by which nerve 9. distribution of supf peroneal nerve 10. level of bifurcation of aorta 11. level of hilum of kidney 12. level of inf mesenteric artery 13. level of ext iliac artery 14. distribution of genitor femoral nerve 15. definition of hypoxia

16. tissue development in # atrophy 17. tissue development in spina bifida hypoplasia 18. post gastrectomy deficiency of 19. pt with persistent vomiting biochem abnormality 20. passive rectal incontinence sphinter envolved 21. presentation of long standing catheter 22. tubercular cystitis 23. nerve supply of ant aspect of knee jt 24. relation of str at popliteal fossa 25. muscle attach to lat side of popliteal 26. nes of adductor of thigh 27. flexion at distal ip jt at ring finger 28. flexion at distal ip jt at thumb 29. relation of ulnar nerve with ulnar artery 30. ulnar injury at elbow 31. foment sign test for 32. nerve for adduction of thumb 33. deep br of ulnar nerve supply 34. function of cortisol in stress 35. ecg changes in pulm embolism 36. nerve supply at angle of mouth 37. lateral border of tongue is supplied by 38. gastric lymphoma are mc of which type 39. cut injury at side of face will cut 40. epydymorchitis in sexually active male without uti 41. cause of swelling scrotum with sec neck 42. multiple swelling all over neck/axill /inguinal malignant lymphoma 43. follicular tumour thyroid IOC 44. sesation over deltoid muscle 45. injury medial to deltopectoral groove 46. head of radius articulate with 47. Head of radius is kept in place by which ligament 48. cereberal perfusion pressure 49. central chemo receptors 50. baroreceptors

51. foramina for transmission of mandibular nerve 52. foramina for transmission of vagus/ hypoglossal 53. foramina for transmission of middle meningeal artery 54. nucleus of 6th and 7th cr. Nerve is at which part of brain 55. surface marking of heart valve 56. cardiac tamponade can cuase sudden death 57. peptic acid secretion is stimulated by 58. bee sting with bp 80/60 and p.rate 122/mt TtOC 59. dorsum of foot supplied by which nerve 60. ligation of varicocele ultimately vein leadin to gonadal vn 61. relation of vein at renal hilum 62. COPD 63. blood gas analysis 64. blood gas analysis 65. blood gas analysis 66. difficulty in smiling nerve for it 67. pain in inf molar nerve responsible 68. ganglion for lacrimal dut 69. nerve involved in submandibular gland excision 70. type of reaction rhinorrhea with rashes after two hrs 71. intrinsic factor absorption 72. anemia in gastrctomy 73. iron deficiency anaemia 74. granulomatous intestinal dis crohn. 75. biopsy of gastric antrum 76. causative agent for gastric ulcer and bladder carcinoma 77. virus implicated for cervical cancer 78. virus implicated for anal cancer 79. virus implicated for Kaposi sarcoma 80. pus in diabetic cholycystitis e. g. of 81. injury at third intercostals space at sternum 82. third ventricle to fourth cereberal duct of aqueduct 83. SOL lt in parital causes herniation of 84. vertebral artey supplies which cortex 85. pituitary tumour causing pressure at optic chiasma

86. erosion at lateral angle of eye by swell in three years 87. position of ampulla of vater opening 88. SVR/CO/HR in crdiogenic shock 89. SVR/CO/HR in hemmorrafgicc 90. SVR/CO/HR in septic 91. child with ear discharge fever convulsion high grade fever 92. best criteria fo acute pancreatitis 93. diagnostic marker of carcinoid 94. in aneurysm defect in arterial wall is 95. popliteal aneurysm size 2.5 cnm 96. nodule in rheumatoid arthritis 97. wrist drop wekness at elbow 98. sensation of middle finger root value 99. sensation at lat calf disc protrusion 98 sensation loss at great toe spinal levet 100. lspinal level in uretric pain 101. passage of small stone cause 102. ns of lateral part of arm 103 parasympathetic causes arterial vaso constriction /dilatation 104 sa node is inervated by 105 adrenaline causes st of which receptors 106 role of adrenaline 107 role of dobutamine 108 role of ephedrtine 109 changes in lung capacity in emphysema 110 FEV1/FCV is reduced in 111 Ist to reponse to haemmorrage 112 Patent ductus arteriosus develop fromn which arch 113 Trauma to medial third of clavicle will injure 114 Bendrofluadizide acts at 115 Inrease urine osmolalty in op adissonian crisis\ 117 Left femoral hernia op for small bowel obstruction lead to chest pain? 118 Factor controlling bp from kidney 119 Post operative hypocalcemia in thyroid ectomy

120 Mode of action in pth in ca metabolism 121 Ca breast role of in osteoporotic Calcitonin 122 Cuases of hypercalcemia burn? 123 b/l swelling in 60% burn pt due to hypoprotienemia 124 insulin is increased with c peptide 125 partail gastrectomy result in dumping syndrome 126 multiple neurological envolvement in diabetes 127 insulin given in hyper glycemic will 128 effect of cortisol on various hormone 129 barret esophagus > 5cm pathology envolved 130 para neoplastic syndrome see in small cell tumour 131 in fetal circulation blood passes from rt atrium to left atrium 132 intermittent positive pressure incr/decreases venous return 133 basic mechanism in pulm embolism 134 post splenectomy diffuse opacity lung caused by pneumoccocal pneumonia 135 ca brest operation nerve envolved in winging of scapula 136 upper arm injury with swelling and pain fasciotomy Reply to HalaReport

Post #8 Hala Adel replied to Hala's post5 hours ago MRCS Part 1 Practice Questions ( Physiology ) - 2 of 3 Correct In a patient with anaphylaxis, which of the following should be given to inhibit the important late-phase reaction? Single best answer - choose ONE true option only

Antihistamines

Epinephrine

Leukotriene inhibitor

Hydrocortisone Your answer NSAID

Hydrocortisone blocks the generation of leukotrienes and prostaglandins, and hence prevents the late-phase reaction often characterised by asthma. It should be given intravenously/intramuscularly at a dose of 100200 mg. None of the other agents listed above affect this aspect of anaphylaxis. Approximately 30% of deaths related to anaphylaxis occur as a consequence of this late-phase reaction. Correct Which of the following is true of bile? Single best answer question choose ONE true option only Is secreted into the terminal ileum Is necessary for protein absorption Contains urobilinogen Your answer Is produced by the cells lining the common bile duct Is concentrated in hepatocytes Bile is a solution of bile salts (bilirubin), pigments and cholesterol. It is secreted by the hepatocytes and concentrated in the gall bladder. Following ingestion of a fat-containing meal, cholecystokinin stimulates the gall bladder which in turn contracts and expels bile through the cystic duct into the common bile duct. Bile is secreted into the duodenum. Correct In a patient with small bowel ischaemia, what metabolic picture would most likely be seen on blood gas analysis? Single best answer question choose ONE true option only Compensated metabolic acidosis

Metabolic acidosis and increased anion gap Your answer Metabolic acidosis and normal anion gap

Metabolic alkalosis

Respiratory acidosis

This patient has had a significant operation during which infarcted bowel has been resected. The most likely abnormality is a metabolic acidosis secondary to mesenteric ischaemia and hypovolaemia resulting in anaerobic metabolism and accumulation of lactic acid. By definition, this patient will have a low arterial pH and a low bicarbonate concentration. An increased anion gap will also be seen. The anion gap may be calculated by;

[Na+] + [K+] [CL-] [HCO3-]

The normal anion gap is 8-16mmol/L. The anion gap is a useful tool in differentiating between an acidosis due to the accumulation of organic acids e.g. lactic acid (as in this patient) and acidosis that are secondary to the loss of base or ingestion of acid where there will be a normal anion gap. Correct Absorption of calcium from the digestive tract? Single best answer question choose ONE true option only Takes place mostly in the proximal jejunum Your answer Is prevented by the presence of small amounts of phytic acid in the diet , even when an excess calcium is ingested

Is facilitated by the presence of fat in food

Can be reversed (calcium is secreted into bowel lumen) when plasma calcium concentration is raised by a calcium infusion

Is about as rapid as that of sodium

Phytic acid produces insoluble calcium phytate, when all phytic acid has been precipitated the excess calcium is absorbed.

Fatty acids form insoluble calcium salts (soaps).

The shift of calcium ions across the intestinal mucosa is virtually one way.

Sodium is absorbed at a speed fifty times that for calcium absorption Correct Following a motorbike RTC, a young patient has a heart rate of 42 , BP of 70/45 mmHg, and warm peripheries. His blood pressure does not improve despite IV fluids. What is the likely diagnosis? Single best answer question choose ONE true option only Cardiogenic shock

Hypovolaemic shock

Neurogenic shock Your answer Septic shock

Septic shock

Neurogenic shock is a result of interruption of the descending sympathetic pathways of the spinal cord causing loss of vasomotor tone. There is subsequent pooling of blood in the extremities and the development of hypotension. Additionally if the lesion is above T6 there may be associated loss of cardiac sympathetic innervation, therefore, these patients are often bradycardic or are unable to mount an appropriate tachycardic response to hypovolaemia. As the primary problem in these patients is loss of sympathetic tone the observed hypotension does not respond to fluids and must be corrected with the use of vasopressors that increase vascular tone and atropine if indicated to counter the bradycardia. Correct Which of the following respiratory physiology tests would be consistent with a diagnosis of moderately established cryptogenic fibrosing alveolitis?

Single best answer question choose ONE true option only

Diffusion capacity decreased, FEV1/FVC normal, total lung capacity reduced Your answer Diffusion capacity increased, FEV1/FVC normal, total lung capacity increased

Diffusion capacity normal, FEV1/FVC reduced, total lung capacity reduced

Diffusion capacity decreased, FEV1/FVC normal, total lung capacity n ormal

Diffusion capacity decreased, FEV1/FVC increased, total lung capacity increased

Diffusion capacity is characteristically decreased in restrictive lung disorders. FEV1/FVC reduced would be seen in obstructive airways disease, which would be reversible in asthma and irreversible in COPD. In restrictive conditions FEV1/FVC ratio is normal or increased. Total lung capacity is reduced in restrictive lung disease, whilst it is normal or increased in obstructive airways disease. Correct The plateau phase of the cardiac action potential is due to:

Single best answer question choose ONE true option only Magnesium influx

Potassium influx

Calcium influx Your answer Chloride efflux

Sodium influx

The most important source of activator calcium in cardiac muscle remains its release from the sarcoplasmic reticulum. Calcium however also enters from the extracellular space during the plateau phase of the action potential. This calcium entry provides the stimulus that induces calcium release from the sarcoplasmic reticulum (calcium induced calcium release).

The result is that tension generated in cardiac, but not in skeletal, muscle is profoundly influenced both by extracellular calcium levels and factors that affect the magnitude of the inward calcium current. This is of practical value in two key clinical situations; in heart failure where digoxin is utilised to increase cardiac contractility (by increasing the intracellular calcium concentration) and in hyperkalaemia where calcium gluconate is used to stabilise the myocardium.

The plateau phase of the action potential in cardiac muscle (principally due to calcium influx) maintains the membrane at a depolarised potential for as long as 500ms. The result is that the cell membrane is refractory throughout most of the mechanical response, largely due to the inactivation of fast sodium channels. This prevents tetany upon repetitive stimulation which would be detrimental to cardiac output. Furthermore, the prolonged refractory period in cardiac muscle allows the impulse that originates in the sino-atrial node to propagate throughout the entire myocardium just once, thereby preventing re-entry arrhythmias. Correct Which components of the nephron are most important with regulation of extracellular fluid osmolality?

Single best answer question choose ONE true option only Proximal convoluted tubule and distal convoluted tubule

Glomerulus and distal convoluted tubule

Loop of Henle and collecting ducts Your answer Glomerulus and proximal convoluted tubule

Glomerulus and loop of Henle

Each component of the nephron is associated with particular predominant functions. The

glomerulus is involved with passive filtration of the plasma and formation of tubular filtrate. The proximal convoluted tubule is mainly involved with conservation of filtered solutes and water as well as secretion of certain waste products. The distal convoluted tubule plays a role in regulating preferential reabsorption of Na+ ions at the expense of K+ and H+ ions, under the control of aldosterone. It is the loop of Henle and collecting ducts which play the most important role in regulating extracellular fluid osmolality. The loop of Henle creates the large medullary interstitial osmotic driving force for the reabsorption of water through the walls of the collecting ducts whose permeability is regulated by antidiuretic hormone (arginine vasopressin). Correct A patient with a significant head injury has a GCS of 6, a dilated left pupil and is found to be coning. Which of the following vital signs is this patient likely to exhibit? Single best answer question choose ONE true option only Hypertensive and bradycardic Your answer Hypertensive and tachycardic

Hypotensive and bradycardic

Hypotensive and normal heart rate

Normotensive and tachycardic

This patient has signs of raised intra-cranial pressure (ICP). The dilated left pupil reflects oculomotor nerve compression secondary to transtentorial cerebral herniation. Hypertension and bradycardia may be observed in such patients, this is known as Cushings reflex and it reflects an attempt to maintain cerebral perfusion in the face of rising ICP.

Cerebral perfusion pressure (CPP) is the Mean arterial pressure (MAP) minus the intracranial pressure (ICP). i.e. CPP = MAP ICP.

The Monroe-Kellie hypothesis states that the skull is a rigid box that contains brain, CSF, and blood, therefore, ICP = VCSF + VBrain + VBlood.

It stands to reason that if the volume of any one of these components increases i.e. an intracerebral haemorrhage, the ICP will rise. The rise in ICP may be minimally compensated by a decrease in the two other components, but after this point the ICP will rise steeply. Incorrect Which one of the following statements about renin secretion is true?

Single best answer question choose ONE true option only Renin is secreted by the epithelial cells of the renal glomerulus Your answer

Reduced delivery of NaCl to the macula densa cells of nephrons increases renin secretion Correct answer A rise in pressure in the renal afferent arteriole increases renin secretion

Renin secretion is reduced by increased activity in the renal sympathetic nerves

Renin secretion is reduced by inhibition of angiotensin-converting enzyme

Renin is an enzyme involved in activating the angiotensin-aldosterone system. It is produced and secreted by modified smooth muscle cells of the afferent arterioles of the kidney. Renin secretion is stimulated by a local fall in blood pressure in the afferent arterioles, by reduced delivery of filtered NaCl to the macula densa cells of the nephrons (tubulo-glomerular feedback) and by increased activity in the renal sympathetic nerves. Renin secretion is increased by inhibition of angiotensin-converting enzyme since the resulting reduction in angiotensin II and aldosterone levels reduces the negative feedback effect on renin secretion. Correct Aldosterone is secreted from the:

Single best answer question choose ONE true option only Liver

Zona glomerulosa of the adrenal cortex Your answer Juxtaglomerular apparatus

Adrenal medulla

Zona fasciculata of the adrenal cortex

The adrenal gland comprises an outer cortex and an inner medulla, which represent two developmentally and functionally independent endocrine glands within the same anatomical structure. The adrenal medulla secretes adrenaline (70%) and noradrenaline (30%). The adrenal cortex consists of 3 layers, or zones. The layers from the surface inwards may be remembered by the mnemonic GFR:

G = Zona glomerulosa (secretes aldosterone) F = Zona fasciculata (secretes cortisol and sex steroids) R = Zona reticularis (secretes cortisol and sex steroids)

Aldosterone is a steroid hormone that facilitates the reabsorption of sodium and water and the excretion of potassium and hydrogen ions from the distal convoluted tubule and collecting ducts.

Conns syndrome is characterised by increased aldosterone secretion from the adrenal glands. Incorrect A patient on enteral nutrition develops constipation. What could explain the underlying clinical physiology? Single best answer question choose ONE true option only

Hyperosmolar feed Your answer Bacterial contamination

Low feed temperature

Inadequate fluid replacement Correct answer Reduced intestinal absorptive capacity

Hyperosmolar feed, bacterial contamination, low feed temperature, too rapid or irregular administration, lactose intolerance, reduced intestinal absorptive capacity can all explain diarrhoea. Incorrect An overweight 32-year-old woman presents with a short history of painless jaundice. There is no previous history of illness and, apart from the jaundice, she has no signs of chronic liver disease. Initial investigations reveal a haemoglobin of 12.7 g/dl, MCV 105 fl, serum bilirubin 162 mmol/l, AST 145 U/l, alkaline phosphatase 224 U/l, gamma-glutamyltransferase 200 U/l.

Which of the following is the most likely diagnosis? Single best answer question choose ONE true option only

Alcoholic liver disease Correct answer Autoimmune chronic hepatitis

Carcinoma of the head of the pancreas

Cholecystitis Your answer Hepatitis A infection

Jaundice with an elevation of both AST and alkaline phosphatase suggests mixed hepatocellular damage and cholestatic liver disease, typical of acute alcoholic hepatitis on a background of chronic liver disease (and is not excluded by the lack of physical signs). The high gammaglutamyltransferase lends support to this (although it may be increased in liver disease of any cause). Macrocytosis is typical of chronic excessive alcohol intake and is not a feature of the other conditions; although were it not present, autoimmune liver disease would need to be considered. In hepatitis A, AST is typically higher than alkaline phosphatase, while the reverse is true of pancreatic carcinoma. Chronic cholecystitis can cause jaundice but it would be unusual for there to be no history of acute episodes. Correct Which of the following physiological characteristics relates to the lining of the respiratory tract? Single best answer question choose ONE true option only About 1 litre of mucus is produced every day

The cilia are under the control of a physiological motor, dynein Your answer The mucociliary escalator moves at 0.2 cm/minute Reply to HalaReport

Post #9 Hala Adel wrote5 hours ago The bronchioles have cartilage in their wall

The bronchioles have diameters up to 5 mm

About 100 ml of mucus is produced every day. The cilia are under the control of a physiological motor, dynein (which is absent in Kartageners syndrome). The mucociliary escalator moves at 2 cm/minute. The bronchioles do not have cartilage in their wall (which distinguishes them from bronchi). The bronchioles can be up to 1 mm in diameter. Correct Which of the following hormones is secreted by the kidney in response to sympathetic nervous

stimulation? Single best answer question choose ONE true option only Aldosterone

Angiotensin I

Angiotensin II

Erythropoetin

Renin Your answer Renin is produced by the juxtaglomerular apparatus of the kidney in response to hypovolaemia, via 3 mechanisms: 1. increased catecholamine levels secondary to sympathetic stimulation from arterial receptors 2. direct effect of hyponatraemia on the juxtaglomerular apparatus 3. reduction of renal perfusion pressure via afferent arteriolar baroreceptors

Renin acts to cleave angiotensin I from angiotensinogen produced in the liver. Angiotensin converting enzyme is present in many tissues, especially the lungs, and converts angiotensin I to angiotensin II. Angiotensin II is a powerful vasoconstrictor, causing vasoconstriction of renal arteries, as well as a positive inotropic effect on the heart. It also causes release of ADH and adrenaline. Along with aldosterone, whose release is also stimulated, Angiotensin II conserves Na+ and H2O in the gut. Aldosterone acts to conserve Na+ and H2O in the distal renal tubule and collecting ducts. These mechanisms combine to restore the plasma volume in hypovolaemia. Erythropoetin is released by the kidney in response to hypoxia and high levels of the products of red cell breakdown, and increases the rate of red cell production Correct In metabolic alkalosis associated with prolonged nasogastric aspiration in postoperative ileus, what is the most important cause of the acidbase disturbance?

Single best answer question choose ONE true option only

Hypoventilation

Increased renal bicarbonate reabsorption

Loss of gastric acid

Your answer Potassium depletion

Secondary aldosteronism

Loss of unbuffered gastric acid is the cause of the metabolic alkalosis seen under these circumstances if there is inadequate replacement of the fluid lost with intravenous physiological saline. Increased renal bicarbonate reabsorption (needed to allow adequate renal sodium reabsorption in the presence of hypochloraemia), potassium depletion (gastric secretions contain about 10 mmol/l of potassium) and secondary aldosteronism (a result of extracellular fluid loss) all help to maintain the alkalosis, but they do not cause it. Hypoventilation is a compensatory change: on its own, hypoventilation causes carbon dioxide retention and a respiratory acidosis. Correct Which of the following malignancies could be responsible for a hypercalcaemia and low serum phosphate level?

Single best answer question choose ONE true option only Osteoclastoma

Squamous cell carcinoma of the lung Your answer Prostate cancer

Transitional cell carcinoma of the Bladder

Basal cell carcinoma

Metastatic cancerous bone lesions can result in the release of mineralised calcium and phosphate into the blood stream which can result in both hypercalcaemia and hyperphosphatemia. Squamous cell carcinoma of the lung can result in hypercalcaemia with a normal or low phosphate level due to the release of PTH related peptide in a paraneoplastic phenomenon. PTH related peptide acts in similar fashion to PTH (although it will not be detected by standard PTH assays) by increasing the activation of Vit D and therefore increasing the absorption of calcium and phosphate from the intestines. In addition, calcium and phosphate is released from bone by a direct action on osteoclasts. However, PTH also increases the renal excretion of phosphate and the net effect can be a low or normal serum phosphate level Correct The ejection fraction is defined as:

Single best answer question choose ONE true option only The ratio of the end diastolic volume to stroke volume

The ratio of stroke volume to end diastolic volume Your answer End diastolic volume minus end systolic volume

End systolic volume divided by stroke volume

The ratio of stroke volume to end systolic volume

During diastole, filling of the ventricles normally increases the volume of each ventricle to about 120mls. This volume is known as the end diastolic volume. Then, as the ventricles empty in systole, the volume decreases about 70mls, which is known as the stroke volume. The remaining volume in each ventricle, about 50mls, is known as the end systolic volume and acts as a reserve which can be utilised to increase stroke volume in exercise.

The fraction of end diastolic volume that is ejected is called the ejection fraction usually equal to about 60%. The ejection fraction is often used clinically as an indirect index of contractility. It is a particularly useful in assessing the state of the myocardium prior to aortic aneurysm repair where cross-clamping of the aorta places particular stress on the myocardium. Correct Haemolytic disease of the newborn is typically restricted to the presence of Rhesus antigens on red cells rather than ABO antigens. Predominantly, such anti-Rh antibodies cross the placenta during the third trimester.

Which of the following statements best explains the background physiology?

Single best answer - choose ONE true option only Antibodies to ABO blood groups are IgM, whereas antibodies to Rhesus antigens are IgG Your answer Antibodies to ABO blood groups are IgG, whereas antibodies to Rhesus antigens are IgM

Antibodies to ABO blood groups are IgA, whereas antibodies to Rhesus antigens are IgG

Antibodies to Rhesus antigens are IgD, whereas anti-ABO blood groups are IgM

Antibodies to Rhesus antigens are IgE, whereas anti-ABO blood groups are IgG

IgG antibodies to Rhesus antigens can cross the placenta during the last trimester, whereas ABO antibodies are IgM and hence cannot cross the placenta. The function of serum IgD is unknown. The transplacental passage of immunoglobulin only applies to IgG. Correct Which of the following TFTs is suggestive of Graves disease? Single best answer question choose ONE true option only Raised TSH, free T4, raised free T3

Normal TSH, raised free T4, decreased T3

Decreased TSH, raised free T4, raised free T3 Your answer Decreased TSH, decreased free T4, decreased free T3

Raised TSH, normal free T4, normal free T3

The thyroid produces T3 and T4 upon stimulation from TSH released from the anterior pituitary, which in turn is regulated by the hypothalamic secretion of TRH (Thyrotophin releasing hormone). TRH is transported to the anterior pituitary along the hypophyseal tract. The negative feedback effects of T3 and T4 levels regulate the whole mechanism. Graves disease is the commonest cause of hyperthyroidism and is a result of IgG antibodies binding to TSH receptors, stimulating thyroid hormone production. The TFTs in such patients classically show a much-reduced TSH concentration with inappropriately raised T3 and T4 levels. Correct In the two step hydroxylation process for activation of Vitamin D, where does the first hydroxylation take place? Single best answer question choose ONE true option only Kidney Lung Liver Your answer Skin Duodenum Vitamin D is a fat soluble vitamin that is derived from our diet or via the skin from direct sunlight. It is converted to 25-hydroxycholecalciferol in the liver and is further hydroxylated in the kidney to 1, 25-hydroxycholecalciferol. In this active form it increases calcium uptake in the gut and promotes phosphate absorption too. It increases kidney reabsorption of calcium and phosphate and at very high concentration will promote osteoclastic reabsorption of bone. Incorrect A 45-year-old woman with type-2 diabetes is making an apparently good recovery 7 days after a partial resection of the small intestine following trauma sustained in a stabbing incident. She is receiving parenteral nutrition with additional normal saline and, because of a history of deep vein thrombosis some 10 years previously, is on prophylactic heparin. Before her admission she was well, with no ongoing medical problems and taking no regular medication. Serum electrolyte

results are as follows: sodium 129 mmol/l, potassium 6.5 mmol/l, bicarbonate 24 mmol/l, urea 8.5 mmol/l, creatinine 120 mol/l, glucose 10.2 mmol/l. Her potassium concentration has risen over the past 3 days. The potassium content of the parenteral feed has been reduced from 60 to 20 mmol/24 h during this period. Urine output is appropriate to her fluid input. Her red cell, white cell and platelet counts are all normal.

What is the most likely cause of the hyperkalaemia? Single best answer question - choose ONE true option only

Heparin treatment Correct answer Overprovision of potassium in the parenteral feed Your answer Primary adrenal failure (Addisons disease)

Pseudohyperkalaemia

Renal impairment

Approximately 20 mmol/24 h is the minimum obligatory potassium output, while the typical potassium requirements for patients on parenteral feeding are 4080 mmol/24 h. Pseudohyperkalaemia is hyperkalaemia occurring as a result of a loss of potassium from white cells and platelets during clotting, usually seen in patients with high white cell or platelet counts. Typically, the plasma potassium concentration is significantly lower than the serum potassium concentration in this condition. The elevated urea may be due to an excessive provision of amino acids, but neither it nor the creatinine level suggest sufficient renal impairment to cause such a severe hyperkalaemia. Incipient adrenal failure could have been made overt by the stress of surgery, but this is uncommon. The heparin is more likely to be responsible: heparin inhibits aldosterone secretion by the adrenal cortex, leading to impaired renal potassium excretion, particularly in patients with diabetes or those who are acidotic Correct A medical SHO is required to give a blood sample to check his HepB status. He received a course of vaccinations nine months ago.

What is his blood test likely to show? Single best answer - choose ONE true option only.

Anti-HBeAb

Anti-HBsAb Your answer Anti-HBsAb + anti-HBcAb

HBsAg + HBcAg

IgM to HBcAg

Surface and core antigens (HBsAg, HBcAg) are detectable during acute infection. HbeAg (envelope) is a good marker of high infectivity, while anti-HbeAg suggests a patient who is less infective. Acute infection is also implied by IgM to HbcAg, while IgG to HBcAg suggests a previous infection. Viral clearance and recovery correlate with the disappearance of antigens and the appearance of antibodies. Previous vaccination is suggested by the presence of only anti-HBsAb. Correct In which of the following types of shock is the primary problem due to loss of peripheral vascular resistance mediated by microorganisms? Single best answer question choose ONE true option only Cardiogenic Anaphylaxis Septic Your answer Neurogenic

Hypovolaemic

Septic shock is due to bacteria-mediated vasodilation. This results in a relative loss of circulating blood volume. Patients are peripherally warm and pink in contrast to other types of chock where the skin is cold, clammy and shutdown. Cardiogenic shock arises due to a failure of the hearts pump mechanism, usually post-myocardial infarction. Anaphylaxis is a severe allergic reaction resulting in a profound release of histamine and other inflammatory mediators. There is a relative hypovolaemia due to vasodilatation, however bacteria are not implicated in the process. Incorrect Which of the following metabolic effects is most likely to be caused by thyroid hormone? Single best answer question choose ONE true option only Decreased glycogenolysis in the liver

Increased glucose absorption in the gut Correct answer Decreased lipolysis

Your answer Decreased expression of adrenergic receptors

Decreased oxygen uptake in the mitochondria

Thyroid hormone has widespread metabolic effects. Increased glycogenolysis in the liver, increased glucose absorption in the gut and increased insulin breakdown all tend to increase blood glucose. The glycogenolytic effects of catecholamines are also potentiated. These effects can make the diagnosis and management of diabetes in thyrotoxicosis difficult. There is an overall lipolytic effect, with decreased serum cholesterol seen in thyrotoxicosis, and an increase in hypothyroidism. There is an increased expression of b-adrenergic receptors in many tissues including skeletal and cardiac muscle. There is a positive inotropic effect with increased cardiac output and heart rate. A raised metabolic rate and increased heat production are due to increased oxygen uptake and ATP production in the mitochondria. There are also effects on bone, with an overall breakdown of bone, sometimes leading to hypercalcaemia. Increased serum 2,3 DPG leads to a right shift of the haemoglobin dissociation curve. Thyroid hormones are also essential for fetal development, with deficiency leading to cretinism. The fetus produces its own hormone from 18 weeks of gestation. Correct Which of the following factors is involved in the extrinsic coagulation cascade? Single best answer question choose ONE true option only VII Your answer VIII IX XI XII The clotting cascade is the ordered stepwise enzyme-controlled activation of soluble clotting factors to produce an insoluble fibrin mesh; a thrombus. There are two different pathways, intrinsic and extrinsic. The intrinsic pathway is so called as all the elements necessary for its activation are in the blood. It is triggered by exposure of collagen in damaged vascular endothelium. The extrinsic pathway requires the release of tissue factors from damaged tissues to start the process. Both pathways converge in the common pathway. Sequential activation of factors XII, XI, IX and VIII comprises the intrinsic pathway. The extrinsic pathway involves tissue factor and activated factor VII. Both the intrinsic and extrinsic pathways activate factor X, II and I to form fibrin and this is the common pathway. Correct A 75-year-old woman undergoes total gastrectomy for carcinoma of stomach.

With which of the following nutrients is she most likely to require parenteral replacement? Single best answer - choose ONE true option only

Ascorbic acid

Folic acid

Iron

Vitamin B12 Your answer Vitamin D

No significant absorption of nutrients takes place in the stomach. However, because of the lack of secretion of pepsin, and hence reduced activation of pancreatic proenzymes, and the fact that the ability to eat normal amounts of food may be greatly decreased, patients who have had total gastrectomies may require general nutritional supplementation, eg with proprietary high-energy, high-protein liquids. However, the absorption of vitamin B12, although it takes place in the terminal ileum, is critically dependent on the availability of intrinsic factor, which is only secreted by the parietal (oxyntic) cells of the stomach. Correct Reply to HalaReport

Post #10 Hala Adel wrote5 hours ago A 25-year-old man is admitted to hospital with persistent vomiting. He is clinically dehydrated and hypotensive. His serum sodium concentration is 124 mmol/l, potassium 4.9 mmol/l, urea 9.8 mmol/l, creatinine 96 mmol/l. Urine sodium concentration in a specimen passed on admission is 62 mmol/l.

Which of the following is the most likely cause of the hyponatraemia? Single best answer question choose ONE true option only

Adrenal failure Your answer Cerebral salt wasting

Gastrointestinal fluid loss

Low sodium intake

Syndrome of inappropriate antidiuresis (SIAD)

Natriuresis in a dehydrated, hyponatraemic patient suggests that there is uncontrolled renal loss of sodium, such as occurs in adrenal failure. Cerebral salt wasting can also cause dehydration and hyponatraemia due to excessive natriuresis, but typically occurs following a head injury or brain surgery. Hyponatraemia and dehydration due to gastrointestinal fluid loss or sodium deficiency due to a low intake should lead to renal conservation of sodium. Although SIAD is an important cause of hyponatraemia and sodium excretion may be high, the hyponatraemia is due to water excess and patients are not dehydrated. Incorrect Which of the following statements regarding precautions with using colloids is true?

Single best answer question choose ONE true option only Dextrans do not carry a risk of anaphylaxis

Dextrans are less likely to interfere with blood cross-matching than starches Your answer Gelatins are less likely to cause pruritis or anaphylaxis than starch solutions

Haemaccel and blood are compatible through the same IV cannula

Colloids may worsen peripheral oedema Correct answer Dextrans (e.g. Dextran 40 or 70) comprise solutions of multiply-branched polysaccharides. They carry a risk of anaphylaxis, interfere with blood cross-matching and may reduce platelet adhesion.

Gelatins (e.g. Gelofusin and Haemaccel) are formed from the hydrolysis of bovine collagen. They are much more likely than starch-based colloid solutions to cause pruritis or anaphylaxis. In addition, the calcium content of Haemaccel can cause blood to clot if infused through the same cannula.

All colloids may worsen peripheral oedema if there is loss of capillary wall integrity with resultant leak of the colloid into the interstitial fluid compartment. Incorrect With respect to vomiting which of the following statements is the best answer? Single best answer question choose ONE true option only Chief cells

The CTZ is outside the blood brain barrier

Correct answer 5HT3 agonists may be effective in controlling cisplatin induced vomiting

H2 receptors are abundant in the vomiting centre

The vomiting centre is present in the reticular formation of the mid brain Your answer The vomiting centre is present in the reticular formation of the medulla, the CTZ is outside the blood brain barrier and the main receptors are dopaminergic D2 receptors. 5HT3 antagonist is effective in controlling vomiting. H1 receptors have been identified in the vomiting centre. Correct You review a 39-year-old sportsman who complains of knee pain. Arthroscopy reveals damage to the cartilage.

Which of the following stems best describes a property of hyaline cartilage? Single best answer question choose ONE true option only

It has a blood supply from small arterioles

It is rich in type 1 collagen

Chondrocytes secrete collagen only

It is avascular Your answer Pressure from normal joint loading accelerates damage to cartilage

Hyaline cartilage forms the articular surface and is avascular, relying on diffusion from synovial fluid for nutrients. It is rich in type II collagen and forms a meshwork containing proteoglycan molecules that retain water. Intermittent pressure from joint loading is essential to maintain normal cartilage function. Chondrocytes secrete proteoglycans and collagen and are embedded in the cartilage. They migrate to the joint surface along with the matrix that they produce. Incorrect Flow through a vessel or lumen is:

Single best answer question choose ONE true option only Is inversely proportional to the pressure head of flow

Is inversely proportional to the radius Your answer Is directly proportional to the length of the tube

Is directly proportional to the viscosity of blood passing through it

Is directly proportional to the fourth power of radius Correct answer

The Hagen-Poiseuille law states that the flow through a vessel is: * Directly proportional to the pressure head of flow * Directly proportional to the fourth power of radius * Inversely proportional to the viscosity * Inversely proportional to the length of the tube

The radius of the tube is therefore the most important determinant of flow through a blood vessel. Thus, doubling the radius of the tube will lead to a 16-fold increase in flow at a constant pressure gradient. The implications of this are several fold.

First, owing to the fourth power effect on resistance and flow, active changes in radius constitute an extremely powerful mechanism for regulating both the local blood flow to a tissue and central arterial pressure. The arterioles are the main resistance vessels of the circulation and their radius can be actively controlled by the tension of smooth muscle within its wall.

Second, in terms of intravenous fluid replacement in hospital, flow is greater through a peripheral cannula than through central lines. The reason is that peripheral lines are short and wide (and therefore of lower resistance and higher flow) compared to central lines, which are long and possess a narrow lumen. A peripheral line is therefore preferential to a central line when urgent fluid resuscitation, or blood, is required. Correct In estimating the physiological clearance of 10 ml of an intravenous substance which has been administered at 10 mg/ml, the plasma concentration at equilibration is 15 mg/litre, the urine concentration is 150 mg/litre and the subject produces 1440 ml of urine during a 24h collection.

What is the clearance of the substance? Single best answer question choose ONE true option only

1 ml/min

10 ml/min Your answer 0.1 ml/min

100 ml/min

Cannot say from the information given

Clearance is calculated using the formula (U V)/P where U = urine concentration in mg/ml, V = urine production in ml/min, P = plasma concentration in mg/ml. The bolus size of the substance is irrelevant to the clearance. Incorrect How much of 1 litre of 5% dextrose infused intravenously will remain in the intravascular compartment?

Single best answer question choose ONE true option only 250 mls Your answer 400 mls

<100 mls Correct answer 500 mls

333.33 mls

5% dextrose has no oncotic properties (the dextrose is absorbed) and therefore 1 litre of 5% dextrose will be distributed equally amongst the total body water. 1/3 of total body water is extracellular and 2/3 intracellular. In addition, around of extracellular fluid is intravascular and therefore only 1/12th (1/3 x ) of infused 5% dextrose will remain in the intravascular space. In comparison of 0.9% NSaline will remain in the intravascular space as it contains 154 mmols/l of Na+ which is similar to the concentration Na+ found in the extracellular compartment. Correct

What is the site of action of antidiuretic hormone (ADH) in a nephron? Single best answer question choose ONE true option only Proximal convoluted tubule Ascending limb of loop of Henle Descending limb of loop of Henle Distal convoluted tubule Collecting duct Your answer ADH is produced by the posterior pituitary gland in response to reduced extracellular osmolality, blood volume and blood pressure. It promotes reabsorption of water from the collecting ducts, resulting in reduced osmolality and expanded blood volume. Incorrect Which immunoglobulin can fix complement via the alternative pathway? Single best answer - choose ONE true option only

IgA Correct answer IgM

IgG Your answer IgE

IgD

IgA is unusual in that it can fix complement via the alternative pathway. IgG and IgM can fix complement via the classical pathway through the Fc portion of the immunoglobulin. Correct By which process are particles moved along a concentration gradient across a selectively permeable membrane?

Single best answer question choose ONE true option only

Endocytosis

Diffusion Your answer Exocytosis

Osmosis

Phagocytosis

Fat-soluble molecules, such as glycerol, can diffuse through the membrane easily. They dissolve in the phospholipid bilayer and pass through it in the direction of the concentration gradient, from a high concentration to a low concentration. Water, oxygen and carbon dioxide can also diffuse through the bilayer, passing easily through the temporary small spaces between the tails of the phospholipids. Incorrect Some 24 hours after sustaining major trauma in a road traffic accident, a 22-year-old man, not known to have diabetes, is found to have a high blood glucose concentration.

Increased secretion of which of the following substances is most likely to be responsible? Single best answer - choose ONE true option only Adrenaline (epinephrine) Correct answer Cortisol Your answer C-reactive protein

Growth hormone

Insulin

During the metabolic response to trauma, there is increased secretion of catecholamines, cortisol, glucagon and growth hormone. The first three of these tend to increase blood glucose concentration; catecholamines, cortisol and glucagon act directly, whereas growth hormone

appears to potentiate the action of cortisol and opposes the action of insulin. Adrenaline and glucagon act most rapidly, by stimulating glycogenolysis; cortisol tends to act more slowly, through the stimulation of gluconeogenesis. Insulin is a hypoglycaemic hormone. C-reactive protein is a marker of inflammation, but does not affect glucose homeostasis. Incorrect Calcitonin Increases plasma calcium levels Promotes osteoclastic bone resorption Increases renal excretion of phosphate Correct answer Is produced in the parathyroid glands Deficiency causes osteoporosis Your answer Calcitonin is produced by thyroid C cells. Total thyroidectomy (absent calcitonin) has no significant skeletal effects. Plasma calcitonin levels rise with increasing serum calcium. Calcitonin inhibits osteoclastic bone resorption and increases renal excretion of calcium and phosphate. Correct The 3' 5' exonuclease activity possessed by some DNA polymerases that enables the enzyme to replace misincorporated nucleotide is called what?

Single best answer question choose ONE true option only

Proofreading Your answer Replication

Recombination

Retrotransposition

Splicing

Retrotransposition is transposition via an RNA intermediate (transposition is the movement of a genetic element from one site to another in a DNA molecule). Splicing is the removal of introns from the primary transcript of a discontinuous gene. Incorrect An 81-year-old, nursing-home resident is admitted to hospital in an unconscious state. His blood sugar is measured as 1.5 mmol/l (normal 36 mmol/l). You administer glucagon.

Which of the following best describes one of the main actions of glucagon? Single best answer question choose ONE true option only

Decreased ketone body production from fatty acids

Increased lipogenesis in adipose tissue

Decreased glycogenolysis

Decreased gluconeogenesis Your answer Increased glycogenolysis and gluconeogenesis Correct answer Glucagon is produced by pancreatic islet cells and its main action is on the liver to promote glycogenolysis and gluconeogenesis. It also increases lipolysis in adipose tissue and increases ketone body production from fatty acids. The actions of glucagon on adipose tissue are mediated by cyclic AMP to stimulate lipolysis, producing free fatty acids that can act as a major alternative energy source. Catecholamines act in a similar way to glucagon, but in addition have effects on muscle. Insulin promotes the synthesis of glycogen, protein and fat, inhibiting lipolysis and gluconeogenesis. Incorrect What is the average daily volume of gastric secretions (ml per day)?

Single best answer question choose ONE true option only 500

1,000

1,500 Your answer 2,000 Correct answer 2,500

Approximate average fluid secretion volumes (ml per day) for each of the component parts of the adult human gastrointestinal tract are given below:-

Secretion ml/day Saliva 1,500 Gastric 2,000 Bile 500 Pancreatic 1,500 Small intestinal 1,500

The vast majority of this secreted fluid is reabsorbed by the small intestine. Incorrect Increased venous return to the heart is most likely to be caused by Single best answer question choose ONE true option only Deep inspiration Correct answer Forced expiration Your answer Hypovolaemia

Positive pressure ventilation

Tension pneumothorax

Blood returns to the heart from the lower limbs via the action of the calf muscle pumps, valves in the veins of the leg, and the effect of negative intra-thoracic pressure generated during inspiration. Anything causing the intra thoracic pressure to become less negative will decrease the venous return to the right atrium. Tension pneumothorax, positive pressure ventilation and forced expiration all cause this effect, and therefore reduce the venous return. Although hypovolaemia may cause vasoconstriction in an attempt to increase venous return, it is unlikely to increase above normal levels. Correct A 34-year-old woman with a body mass index of 44 kg/m2 seeks medical help for her obesity.

Which one of the following treatments offers her the highest probability of achieving a long-term reduction in weight? Single best answer question choose ONE true option only

An energy-deficient diet (600 kcal/day (~ 143 J/day) less than requirements) for 6 months

Jaw-wiring and milk feeding for 3 months

Treatment with orlistat for 12 months

Treatment with sibutramine for 12 months

Vertical banded gastroplication Your answer Both sibutramine and orlistat have been shown to induce and maintain a greater weight loss than diet alone, but a patients weight often plateaus before adequate weight loss has occurred. Orlistat is only licensed for use for 1 year in the UK, and sibutramine for 2 years. Energydeficient diets, particularly if coupled with increased exercise, are effective, but the lost weight is almost invariably regained, as it is after jaw-wiring and milk feeding. Surgery offers the best chance of achieving long-term weight loss, the results from vertical banded gastroplication combined with a by-pass procedure being even better than those with gastroplication alone. Correct A 15-year-old youth with haemophilia A has suffered recurrent bleeding episodes into his joints. As a consequence he has arthropathies in his knees, elbows and wrists.

What is the most likely coagulation deficiency causing his bleeding tendency? Single best answer question choose ONE true option only

Thromboxane

Factor X

Protein C

Factor IX

Factor VIII Your answer

Deficiency of either factor VIII (haemophilia A) or factor IX (haemophilia B), which together make up the factor VIIIa/factor IXa intrinsic tenase enzymatic complex, results in the clinical phenotype commonly known as haemophilia. Haemophilia principally presents with haematoma formation, easy bruising and bleeding at the site of venepuncture during the toddler period. The disease exists in severe, moderate and mild forms. These are classified as such on the basis of a clinical laboratory blood coagulation test, which is performed to assess the level of functional coagulant protein (per cent activity of factor VIII or factor IX). The pathological problem in both haemophilia A, factor VIII deficiency and haemophilia B, factor IX deficiency (also called Christmas disease) is the inability to form a functional tenase complex to activate factor X to factor Xa. The clinical features of haemophilia predominantly include bleeding into joints and soft tissues. The incidence of central nervous system bleeding has dramatically decreased with concentrate therapy. The life expectancy of people with severe haemophilia had increased from 11 years at the beginning of the twentieth century to approximately 60 years in the early 1980s, before the devastating effects of blood-borne viral disease again shortened average life expectancy. Correct A 21-year-old male medical student who has been feeling non-specifically unwell for several days is noticed to have slightly icteric sclerae by his girlfriend and has liver function tests performed. The results of these are normal apart from a serum bilirubin concentration of 44 mmol/l (317). His urine does not contain bilirubin.

Which of the following is the most likely diagnosis? Single best answer question choose ONE true option only

DubinJohnson syndrome

Gilberts syndrome Your answer Hereditary spherocytosis

Infectious mononucleosis

Rotor syndrome

DubinJohnson, Rotor and Gilberts syndromes are all inherited disorders of bilirubin metabolism. However, in the first two, there is a defect in the secretion of bilirubin from the liver and the bilirubin that accumulates in the plasma is conjugated, water-soluble and thus is excreted in the urine. Infectious mononucleosis can cause hepatitis and jaundice but an elevated transaminase activity would be expected. Hereditary spherocytosis is a chronic haemolytic disorder due to a defect in the red cell membrane (most frequently in spectrin, a structural protein). It can present with a wide range of severity, from jaundice at birth to asymptomatic anaemia or jaundice in adults, but is much less common (approximately 1:5000 in Northern Europeans) than Gilberts syndrome (approximately 1:20). Correct Reply to HalaReport

Post #11 Hala Adel wrote5 hours ago Which of the following is the most important causative factor in the development of obesity in the majority of patients?

Single best answer question choose ONE true option only

Energy intake in excess of expenditure Your answer

Genetic predisposition

Insulin resistance

Intrauterine malnutrition

Leptin deficiency

The ultimate cause of obesity is always an intake of energy in excess of expenditure, but many factors govern both intake and expenditure. There is undoubtedly a genetic predisposition in some individuals. Intrauterine malnutrition may be important in others. Leptin deficiency is a very rare cause of obesity: more frequently, obese individuals are resistant to the actions of obesity. Insulin resistance is probably a consequence of obesity, not a cause. Correct The oxygenhaemoglobin dissociation curve is shifted to the left by which of the following factors? Single best answer - choose ONE true option only

Rise in pH Your answer Rise in 2,3-DPG (2,3-diphosphoglycerate)

Rise in plasma temperature

Rise in blood CO2 content

Fall in plasma bicarbonate concentration

All the above shift the dissociation curve to the right, with the exception of a rise in pH. Correct Which of the following cells secretes intrinsic factor?

Single best answer question choose ONE true option only Goblet cells

Kupffer cells

Peptic cells

Chief cells

Parietal cells Your answer Goblet cells are mucus-secreting cells, widely distributed throughout epithelial surfaces, but especially dense in the gastrointestinal and respiratory tracts.

Kupffer cells have phagocytic properties and are found in the liver. They participate in the removal of ageing erythrocytes and other particulate debris.

The gastric mucosa contains many cell subtypes, including acid-secreting cells (also known as parietal or oxyntic cells), pepsin secreting cells (also known as peptic, chief or zymogenic cells) and G-cells (gastrin-secreting cells). Peptic cells synthesise and secrete the proteolytic enzyme, pepsin. Parietal cells actively secrete hydrochloric acid into the gastric lumen, accounting for the acidic environment encountered in the stomach. However parietal cells are also involved in the secretion of the glycoprotein, intrinsic factor.

Intrinsic factor plays a pivotal role in the absorption of vitamin B12 from the terminal ileum. Autoimmune damage to parietal cells leads to a lack of intrinsic factor and hydrochloric acid, leading to vitamin B12 deficiency and achlorhydria. This is known as pernicious anaemia. Pernicious anaemia is associated with a 3-fold increase in gastric cancer risk. Correct Nociception (pain) Single best answer question choose ONE true option only Is transmitted faster through C fibers than through A delta fibers

Pain impulse received in the dorsal horn can be modulated by other descending spinal inputs Your answer Opioids act on receptors in the peripheral nerves

Side effects of opioids can be reversed by neostigmine

Glycine is excitatory pain neurotransmitter

Pain impulse received by dorsal horn can be modulated by other ascending and descending

spinal inputs (Gate Theory). Pain is transmitted faster in myelinated A delta fibers, opioids act on and ? opioid receptors in the central nervous system and their effects can be reversed by naloxone. Glycine is an inhibitory neurotransmitter. Incorrect A 24-year-old woman undergoes resection of the terminal ileum with fashioning of an ileostomy for Crohns disease. Some 2 weeks after surgery, she is making a good recovery, and is eating a high-energy, low-residue diet, but has a high ileostomy volume, necessitating intravenous fluid replacement. Her serum calcium concentration is 1.82 mmol/l, phosphate 1.28 mmol/l, alkaline phosphatase 82 U/l (normal < 150), albumin 30 g/l, creatinine 80 m mol/l. Prior to surgery, her serum calcium concentration was 2.18 mmol/l, albumin 36 g/l.

What is the most likely cause of her hypocalcaemia? Single best answer question choose ONE true option only

Formation of insoluble calcium salts in the intestine Your answer Hypoalbuminaemia

Hypomagnesaemia Correct answer Malabsorption of calcium

Malabsorption of vitamin D

Impaired fat absorption can lead to the formation of insoluble calcium salts in the gut. Fat and calcium are absorbed in the proximal small intestine, so, too, is vitamin D. Although bile salts are absorbed distally, and impaired absorption can lead to a secondary decrease in proximal fat absorption, this is unlikely to be responsible for hypocalcaemia developing so quickly. The normal alkaline phosphatase level also militates against vitamin D deficiency. Hypocalcaemia would normally be expected to stimulate parathyroid hormone secretion and cause the plasma phosphate concentration to fall (PTH is phosphaturic). Patients with ileostomies can lose large amounts of magnesium through their stomas; hypomagnesaemia impairs PTH secretion and can cause hypocalcaemia that is resistant to an increased provision of calcium. Correct A patient is found to have hyponatraemia. Which condition should be excluded by subsequent investigations?

Single best answer question choose ONE true option only Diabetes insipidus

Syndrome of inappropriate antidiuretic hormone secretion (SIADH) Your answer Diabetes mellitus

Conns syndrome

Cushings syndrome

SIADH causes excess water retention over sodium retention by promoting water reabsorption in the collecting ducts of the kidneys. This results in hyponatraemia. Diabetes insipidus and diabetes mellitus may both cause hypernatraemia by resulting in excess water loss over Na loss. In contrast, Conns syndrome and Cushing syndrome cause hypernatraemia by promoting excess sodium retention over water retention. Correct A 2-day-old male infant is referred for a surgical opinion after his parents bring him into the Emergency Department with abdominal distension and what his parents describe as green vomiting. Which of the following would NOT be on your list of differential diagnoses? Single best answer question choose ONE true option only Duodenal Atresia Hirschprungs disease Pyloric stenosis Your answer Malrotation Meconium Ileus Pyloric stenosis usually presents between 3-12 weeks of age. The vomit is NON-BILIOUS due to the high level of obstruction (the thickened pyloric muscle), which is above the entrance of the common bile duct into the duodenum. Distension is not usually a feature. Bilious vomiting in a newborn is a surgical emergency until proved otherwise. Malrotation is the diagnosis, which needs exclusion with an upper GI contrast to look at the layout of the intestine, specifically whether the duodenal-jejunal flexure is on the correct side of the vertebrae the LEFT is the correct side. Correct The cardiovascular effects of raised intracranial pressure include:

Single best answer question choose ONE true option only decreased blood pressure, decreased heart rate, decreased cerebral perfusion pressure

decreased blood pressure, increased heart rate, decreased cerebral perfusion pressure

increased blood pressure, increased heart rate, decreased cerebral perfusion pressure

increased blood pressure, decreasd heart rate, decreased cerebral perfusion pressure Your answer decreased blood pressure, increased heart rate HR, increased cerebral perfusion pressure

The important relationship between the cerebral perfusion, mean arterial blood pressure and intracranial pressure is as follows:

CPP = MABP ICP, where CPP = cerebral perfusion pressure MABP = mean arterial blood pressure ICP = intracranial pressure

It stems from the fact that the adult brain is enclosed in a rigid, incompressible box, with the result that the volume inside it must remain constant (Monroe-Kelly doctrine). A rise in intracranial pressure therefore decreases cerebral perfusion pressure (and hence cerebral blood flow).

In raised intracranial pressure, as the brainstem becomes compressed, local neuronal activity causes a rise in sympathetic vasomotor drive and thus a rise in blood pressure. This is known as the Cushings reflex. This elevated blood pressure evokes a bradycardia via the baroreceptor reflex. The Cushings reflex helps to maintain cerebral blood flow and protect the vital centres of the brain from loss of nutrition if the intracranial pressure rises high enough to compress the cerebral arteries. Correct With regard to CO2 transported in the blood, most of the CO2 is Single best answer question choose ONE true option only Dissolved in plasma

In the form of carbamino compounds formed from plasma proteins

In the form of carbamino compounds formed from haemoglobin

Bound to Chloride

In the form of HCO3Your answer Carbon dioxide is transported in three main ways:

Carbamino compounds between CO2 and proteins. Most of these reactions are with the globin portion of haemoglobin, accounting for 20-30% of the transported CO2. Dissolved CO2 accounts for about 10% of the transported CO2. HCO3- accounts for about 60-70% of the transported CO2. Correct The glomerular filtration rate is increased by? Single best answer choose ONE true option only Increased plasma colloid osmotic pressure Constriction of the glomerular afferent arterioles

Constriction of the glomerular efferent arterioles Your answer Saline depletion Respiratory alkalosis Constriction of the glomerular efferent arterioles increases the hydrostatic pressure within the glomerulus and hence the filtration pressure. Correct Following a decrease in core body temperature, what causes a rise in circulating plasma thyroxine?

Single best answer question choose ONE true option only An increase production of thyroxine by the thyroid gland

A decrease in renal excretion of thyroxine

Release of thyrotrophin releasing hormone from the hypothalamus Your answer Release of thyroid stimulating hormone from the anterior pituitary gland

An increase in iodine absorption from the intestines

The hypothalamus is thought to be the control centre for thermoregulation. Studies have shown that cooling the hypothalmic area in the brain will result in an increase in the secretion of thyrotrophin releasing hormone from the hypothalamus. This in turn will result in an increase in the secretion of thyroid stimulating hormone from the anterior pituitary gland which acts directly on the thyroid gland to increase the secretion of thyroxine. Thyroxine counters a decrease in body temperature by increasing the cellular metabolic rate in a process that can take several weeks and can result in hypertrophy of the thyroid gland. Correct You are asked to see a patient who had a chest drain removed 4 days ago. There appears to be some infection.

What are the stages in the cell biology of normal wound healing? Single best answer question choose ONE true option only

Demolition is the first phase

Maturation and remodelling can continue for up to a year Your answer Acute inflammation usually lasts for 612 hours

Epithelial cell proliferation is the hallmark of the demolition phase

Collagen deposition is the key process during demolition

The first phase in healing by first intention is the phase of acute inflammation that lasts up to 3 days, if uncomplicated. The initiating factor appears to originate from platelets activated by mature collagen exposed in the wound. Platelets first aggregate then release a variety of active agents including lysosomal enzymes, ATP, serotonin and wound cytokines. A fibrin clot develops, which completes haemostasis and provides strength and support to the wound. The surface dries to form a scab. Platelets and macrophage factors cause local vasodilatation, which produces warmth and increases capillary permeability, allowing serum and white blood cells to accumulate and cause swelling. After the initial acute inflammation, macrophages become active as the main agents of demolition, removing unwanted fibrin, dead cells and bacteria and creating fluid-filled spaces for granulation tissue. Macrophages also release factors that stimulate the formation of new capillary buds during this phase, and later they initiate and control fibroblast activity during repair. Within the connective tissue, randomly orientated collagen begins to form after a few days, reaching a peak of activity after 57 days. Epithelial cells at the edge of the wound start to proliferate after 24 h and this phase can last for up to 3 weeks. Finally, the phase of maturation and remodelling lasts for up to 12 months, during which time the tensile strength of the wound increases and the random collagen is replaced by a more stable form orientated along lines of stress. Correct A 25-year-old patient suffered recurrent deep vein thromboses and also one pulmonary embolism. She was extensively investigated and diagnosed with protein C deficiency.

What pathological process is most likely to be responsible for her venous thromboembolisms? Single best answer question choose ONE true option only

Reduced degradation of factors Va and VIIIa Your answer Reduced factor Xa complex

Reduced inhibition of tissue-factor expression

Reduced protein S

Reduced synthesis of antithrombin III

Protein C acts to inactivate the active forms of the procoagulant cofactors, factors Va and VIIIa. Protein C is a vitamin K-dependent serine protease structurally similar to factors VII, IX and X. Thrombin activates protein C when bound to thrombomodulin, a protein which acts like an endothelial-cell receptor for thrombin. Symptomatic manifestations of protein C deficiency are similar to those of antithrombin III deficiency. Deep venous thrombosis, with or without pulmonary embolism, occurs in 50% of patients by the a Reply to HalaReport

Post #12 Hala Adel wrote5 hours ago MRCS Part 1 Practice Questions ( Physiology ) - 3 of 3 Correct Which of the following physiological abnormalities occurs as a direct consequence of septic shock?

Single best answer question choose ONE true option only A decrease in cardiac output

A decrease in systemic vascular resistance Your answer A decrease in vascular permeability

A decrease in intravascular volume

An increase in cardiac contractility

Septic shock is defined as shock (decreased tissue perfusion resulting in end-organ dysfunction) secondary to a demonstrable source of infection most commonly bacterial in origin. Exotoxin in bacterial cell walls results in the production of cytokines and other inflammatory mediators that reduce vasuclar tone and increase vascular permeability. This in turn can result in a loss of intravascular fluid across capillaries and intravascular volume depletion as a secondary event. Cardiac output can both increase and decrease in septic shock due to the normal physiological response to a decreased blood pressure or cardiac dysfunction caused by circulating cytokines and inflammatory mediators respectively. Correct Bile salt reuptake principally occurs in the:

Single best answer question choose ONE true option only Duodenum

Jejenum

Ileum Your answer Colon

Caecum

90-95% of the bile salts are absorbed from the small intestine and then excreted again from the liver; most are absorbed from the terminal ileum. This is known as the enterohepatic circulation. The entire pool recycles twice per meal and approximately 6-8x per day.

Disruption of the enterohepatic circulation, either by terminal ileal resection or through a diseased terminal ileum (e.g. Crohns disease), results in decreased fat absorption and cholesterol gallstone formation. The latter is believed to result because bile salts normally make cholesterol more water-soluble through the formation of cholesterol micelles. Loss of reuptake also results in the presence of bile salts in colonic contents, which alters colonic bacterial growth and stool consistency. Incorrect A 24-year-old, unconscious man is admitted to A&E. No history is available. The results of arterial blood gas analysis are: [H+] 80 nmol/l (pH 7.1), p(CO2) 7.0 kPa, p(O2) 8.2 kPa, [HCO3] 17.1 mmol/l.

These results indicate which one of the following acidbase disturbances? Single best answer question choose ONE true option only Metabolic acidosis with respiratory compensation

Mixed metabolic and respiratory acidosis Correct answer Respiratory acidosis Your answer Respiratory acidosis with metabolic alkalosis

Uncompensated metabolic acidosis

The high hydrogen-ion concentration (low pH) indicates acidosis. The elevated p(CO2) indicates a respiratory component; in compensated metabolic acidosis, p(CO2) is reduced; in an uncompensated metabolic acidosis (a very unusual situation, since the respiratory response to a metabolic acidosis is usually a rapid one), it would be normal. The hydrogen-ion concentration is too low to be accounted for by a respiratory acidosis alone: there must therefore be a metabolic acidosis in addition (as the low bicarbonate concentration also indicates). Incorrect Concerning Glomerular Filtration, which of the following is true of the proximal convoluted tubule? Single best answer question choose ONE true option only Reabsorbs water by sodium secretion Reabsorbs phosphate Correct answer Increases the volume of reabsorbed fluid under aldosterone stimulation Contains renin-secreting cells Receives most of its blood supply from the vasa recta Your answer The proximal convoluted tubule actively reabsorbs sodium. This sets up an osmotic gradient and water is drawn out of the tubule. It is the site of both phosphate and calcium reabsorption under control of parathyroid hormone. Aldosterone acts on the distal convoluted tubules. Renin is secreted by the cells of the juxtaglomerular apparatus in the distal convoluted tubules. Incorrect Which of the following do not normally occur as a response to a decrease in core body temperature?

Single best answer question choose ONE correct option only Bradycardia

Vasocontriction

A decrease in CNS metabolism

A reduction in plasma catecholamine levels Correct answer A rise in plasma thyroxine Your answer The hypothalamus and the lower brain stem are the most important neural structures that regulate body temparature. A fall in core body temparature is associated with a decrease in CNS activity and can result in bradycardia secondary to depression of cardiac pacemaker cells. The bodys response to a fall in temparature includes shivering, peripheral vasoconstriction and the release of metabolic factors including thyroxine, cortisol and catecholamines. Incorrect A 72-year-old woman is found to have a serum calcium concentration of 3.12 mmol/l. Which of the following clinical features, if present, would most direct you towards a specific cause?

Single best answer question - choose ONE true option only

Bone pain Your answer Hilar lymphadenopathy Correct answer Polyuria

Short QT interval

Ureteric colic

Bone pain can occur with hypercalcaemia secondary to malignancy or hyperparathyroidism. Polyuria is a feature of severe hypercalcaemia, irrespective of the cause. A short QT interval is also a feature of hypercalcaemia. Ureteric colic is particularly associated with primary hyperparathyroidism, but is not specific to this cause. The presence of hilar lymphadenopathy in a patient with hypercalcaemia should raise a suspicion that the latter is due to sarcoid (in which the granulomas secrete calcitriol, 1,25-dihydroxycholecalciferol). Correct In the respiratory system, physiological shunt?

Single best answer question choose ONE true option only Is greater than the anatomical shunt Your answer Is not present in healthy adult

Affects arterial carbon dioxide more than arterial oxygen tension

Has the same effect on respiratory gas exchange as does physiological dead space

Is abolished when the subject breathes pure oxygen

The physiological shunt is the sum of the anatomical shunt (blood passing from the right ventricle to the systemic circulation via normal anatomical pathways, e.g. the bronchial vessels, without passing through the pulmonary alveolar capillaries), and the element of pulmonary alveolar capillary blood that has passed through non or poorly aerated alveoli. Therefore physiological shunt is always at least as great as or greater than the anatomical shunt.

There is always a normal anatomical shunt even in the young healthy adult.

The difference in carbon dioxide tension between arterial and mixed venous blood is a little less than 1 kPa, and therefore even a 50% shunt only increases arterial carbon dioxide tension by about 0.5 kPa. A 50% shunt would reduce arterial oxygen tension from 13.5kPa to below 9 kPa.

The physiological dead space results primarily in a failure to remove carbon dioxide from alveolar gas, i.e. a rise in arterial carbon dioxide tension if ventilation not increased.

The breathing of pure oxygen cannot eliminate the anatomical right to left portion of the physiological shunt. Correct A 70-year-old male complains of constantly feeling cold and lethargic. What is the most likely hormonal deficiency to account for this?

Single best answer question choose ONE true option only Somatostatin

Cholecystokinin

Testosterone

Thyroxine Your answer Insulin

Thyroxine deficiency is the most likely cause, particularly if other features of hypothyroidism are present such as fatigue, weight gain, dry skin and hair, slowly-relaxing reflexes and non-pitting oedema. Amongst other roles, thyroxine is important in regulating basal metabolic rate and body heat production. Testosterone deficiency is likely to result in loss of libido and secondary sexual characteristics. Insulin deficiency may present with features of diabetes mellitus, such as thirst, polyuria and polydipsia. Somatostatin and cholecystokinin are important gastrointestinal regulatory peptides. Correct A patient has the following urea and electrolytes results: Sodium 140 mmol/l Potassium 4 mmol/l Chloride 105 mmol/l Bicarbonate 20mmol/l

Calculate the anion gap.

Single best answer - choose ONE true option only

19 meq/l Your answer 5 meq/l

10 meq/l

30 meq/l

0 meq/l

Anion gap = ([Na+] + [K+]) - ([Cl] + [HCO3]) (all units mmol/l).

Normal range is 816 meq/l Correct The largest contribution to systemic vascular resistance (SVR) is made by the Single best answer question choose ONE true option only Aortic valve

Great arteries

Arterioles Your answer Venules

Great veins

The capillaries and arterioles each account for around 25% of the SVR. The large surface area of the capillaries, as well as the low flow and pressure drop through the capillary beds is vital to their function in exchange of gases and nutrients. The arterioles have abundant smooth muscle in their walls, and flow is regulated to a large degree by the sympathetic nervous system. They therefore exert a great deal of control over the flow through the capillary beds, as well as which capillary beds are open at a given time. Incorrect Within normal physiological limits, which of the following factors does not influence cardiac stroke volume?

Single best answer question choose ONE true option only Preload

Afterload Correct answer Heart rate Your answer Cardiac sympathetic nerve activity Reply to HalaReport

Post #13 Hala Adel wrote5 hours ago Myocardial contractility

Increases in preload (up to a limit) increase stroke volume by Starlings Law of the heart. Starlings Law also indicates that increases in afterload (up to a limit), whilst causing ventricular stretch and an increase in end-diastolic volume, do not increase stroke volume but instead maintain it. Increases in myocardial contractility increase the force of contraction during systole and therefore increase stroke volume. If preload does not become limiting, increases in heart rate increase myocardial contractility, and therefore stroke volume, via the Bowditch effect (a rate-related phenomenon thought to be due to accumulation of intracellular calcium in the cardiomyocytes). Cardiac sympathetic nerve activity increases stroke volume by increasing cardiac contractility and heart rate. Correct A 21-year-old man presents with a 2-day history of persistent vomiting and abdominal pain. His blood gas shows; pH 7.44 PaCO2 7.3 kPa PaO2 12.0 kPa Base excess +12 mmol/l HCO3- 38 mmol/l Cl- 90 mmol/l What does this blood gas demonstrate? Single best answer question - choose ONE true option only Compensated metabolic alkalosis Your answer Compensated respiratory alkalosis

Uncompensated metabolic alkalosis

Respiratory acidosis

Uncompensated metabolic acidosis

It can be seen that the striking features of this blood gas are a pH within the normal physiological range with elevated bicarbonate and base excess. The vomiting in this patient has resulted in the loss of hydrochloric acid and loss of total body H+ concentration, causing a metabolic alkalosis. This patient has compensated for this through respiratory hypoventilation and retention of CO2 (an acidic gas). Ultimately the acid-base imbalance cannot be normalised by respiratory retention of CO2, this is merely a compensatory measure and requires renal modulation of H+ and HCO3- levels Correct The action potential of skeletal muscle? Single best answer question choose ONE true option only Has a prolonged plateau phase

Spread inwards to all parts of the muscle via the T tubes Your answer Causes immediate uptake of Ca into the sarcoplasmic reticulum

Is longer than the action potential of cardiac muscle

Is not essential for contraction

The action potential of the skeletal muscle spreads out from the motor end plate, through the T tube system this causes mobilization of Ca2+ from the sarcoplasmic reticulum to the cytoplasm and this action potential is essential for contraction.

The action potential of cardiac muscle is longer than that of the skeletal muscle and has plateau phase. Correct In a starving patient, which of the following fluid regimens would be most appropriate for a 70kg man over a 24hr period?

Single best answer question choose ONE true option only 3L NSaline with 20mmols potassium chloride in each bag 3L Dextrose-saline

3L Hartmanns solution

1L NSaline with 20 mmols potassium chloride and, 2L 5% dextrose with 20mmols potassium chloride in each bag Your answer 3L 5% dextrose with 20mmols potassium chloride in each bag The daily fluid and electrolyte requirements are 1-1.5 mmols Na+ /Kg/24 hours, 1mmols K+ /Kg/24 hours and 40ml H20 /Kg/24 hours. However, additional fluid should be supplemented if there are 3rd space losses (that commonly occur for instance in severe acute pancreatitis, burns and post major gastro-intestinal surgery) and for other sources of fluid loss including vomiting, diuresis and insensible losses. Correct Gastric acid secretion is stimulated by:

Single best answer question choose ONE true option only Somatostatin

Gastrin Your answer Secretin

The glossopharyngeal nerve

Cholecystokinin

Gastric acid is stimulated by 3 factors: * Acetylcholine: From parasympathetic neurones of the vagus nerve that innervate parietal cells directly. * Gastrin: produced by pyloric G-cells. * Histamine: Produced by mast cells. This stimulates the parietal cells directly and also potentiates parietal cell stimulation by gastrin and neuronal stimulation. H2 blockers such as ranitidine are therefore an effective way of reducing acid secretion.

Gastric acid is inhibited by 3 factors: * Somatostatin * Secretin * Cholecystokinin There are 3 classic phases of gastric acid secretion: * Cephalic (preparatory) phase [significant]: Results in the production of gastric acid before food actually enters the stomach. Triggered by the sight, smell, thought and taste of food acting via

the vagus nerve. * Gastric phase [most significant]: Initiated by the presence of food in the stomach, particularly protein rich food. * Intestinal phase [least significant]: The presence of amino acids and food in the duodenum stimulate acid production.

Incorrect The rate at which a liquid meal leaves the stomach is? Single best answer question choose ONE true option only Greater in the upright than in the supine position

Proportional to the volume of stomach content Correct answer Greater if the meal contains fat

Slower if the meal is 5% glucose than if it is 50% glucose

Slower if vagotomy and drainage procedure (such as gastroenterostomy or pyloroplasty) has been performed Your answer Gastric emptying accelerates on lying down. The rate of gastric emptying at any moment is proportional to the volume present in the stomach at that moment

When the fat reaches the duodenum it stimulates mixed hormonal and vagal mechanisms that slow the rate of stomach emptying. An isotonic meal will empty at maximal rate but osmotically stronger or weaker solutions will empty more slowly.

Vagotomy may temporarily slow gastric emptying, but its long term effect is to increase the rate of gastric emptying or leave it un changed so if a drainage procedure is accompanied by vagotomy there will be a tendency towards accelerating gastric emptying. Correct During digestion of a fatty meal, which hormone causes contraction of the gall bladder and relaxation of the sphincter of Oddi? Single best answer question choose ONE true option only Cholecystokinin Your answer Gastrin

Insulin

Secretin

Somatostatin

Cholecystokinin secretion from the duodenal and jejunal mucosa is stimulated by the presence of fatty acids, amino acids and peptides in the lumen of the duodenum and jejunum. As well as causing contraction of the gall bladder and relaxation of the sphincter of Oddi, it stimulates release of pancreatic enzymes, and increases the secretin mediated secretion of HC03- by pancreatic duct cells. Its release is inhibited by somatostatin. Correct In a breathless patient, a pleural effusion with less than 3g of protein per 100ml of fluid is most likely to be caused by Single best answer question choose ONE true option only Bronchial carcinoma

Mitral regurgitation Your answer Pneumonia

Tuberculosis

Tricuspid regurgitation

An effusion with less than 3g of protein per 100ml is a transudate. Other biochemical characteristics of a transudate include LDH < 200 iU/l, WCC < 1000/ml, glucose mmol/l. Transudative effusions are most commonly due to factors such as decompensated liver failure and left ventricular failure. Malignancy and infection are causes of an exudative pleural effusion. Pulmonary embolism can cause either an exudative or transudative effusion, although the former is more common.

In this question, B is more likely than E to be associated with left ventricular failure, and therefore a pleural effusion. Tricuspid regurgitation is usually functional and secondary to an enlarged right ventricle in right ventricular failure, and causes a pulsatile liver, peripheral oedema and ascites. Correct High titres of antithyroid microsomal and antithyroglobulin antibodies would suggest which of the following diagnoses in a patient presenting with a complaint of tiredness?

Single best answer - choose ONE true option only

Hashimotos thyroiditis

Your answer Reidels thyroiditis

Graves disease

Hypoparathyroidism

Idiopathic hypothyroidism

This finding in Hashimotos thyroiditis is characteristic, but lower titres can occur in Reidels thyroiditis and Graves disease. High titres of these antibodies in euthyroid individuals indicate the possibility of future thyroid failure, but this may be many years away; hence the need for thyroid function tests every 12 years in such individuals. Correct Which of the following constituents is NOT present in Hartmanns solution?

Single best answer question choose ONE true option only Na+

Cl-

K+

Lactate

HCO3Your answer The composition of Hartmanns solution is as follows:Na+ = 131 mmol/l Cl- = 111 mmol/l K+ = 5 mmol/l Ca2+ = 2 mmol/l

Lactate = 29 mmol/l

Therefore, the osmolality of Hartmanns solution is (131+111+5+2+29) = 278 mmol/l. The lactate present in the solution is metabolised in vivo to form HCO3-. Bicarbonate is not added to Hartmanns solution since this would result in the precipitation of calcium carbonate in the storage container. Incorrect What is the main method by which intracellular pH is regulated?

Single best answer question choose ONE true option only The bicarbonate buffer system Your answer The phosphate buffer system

Cytoplasmic proteins Correct answer Carbonic anhydrase

The globin component of haemoglobin

Cytoplasmic proteins provide the main contribution to pH buffering of the intracellular compartment.

In the interstitial (i.e. extracellular and extravascular) compartment, the bicarbonate system is the main mechanism of pH buffering.

In the intravascular (plasma) compartment, pH buffering mechanisms include:-

The bicarbonate buffer system: CO2 + H2O H2CO3 H+ + HCO3- catalysed by the enzyme carbonic anhydrase The phosphate buffer system: HPO42- + H+ H2PO4 Plasma proteins The globin component of haemoglobin Incorrect A 60-year-old obese smoker has been admitted to hospital with chest pain due to unstable angina. A nitrate infusion is started to relieve his chest pain.

Which blood vessels are most sensitive to the vasodilatatory effect of nitrates? Single best answer question choose ONE true option only

Large arteries

Coronary arteries Your answer Capillaries

Large veins Correct answer Pulmonary arteries

The antianginal and haemodynamic effects are mediated predominantly by vasodilatation of the venous system, leading to a fall in left ventricular preload and cardiac work. Correct A patient on total parenteral nutrition (TPN) regimen presents with drowsiness and abnormal serum electrolytes.

What is the most likely cause? Single best answer - choose ONE true option only

Hypocalcaemia

Hypercalcaemia

Hypernatraemia

Hypophosphataemia Your answer Hypomagnesaemia

Administering carbohydrate lowers serum phosphate by stimulating the release of insulin, which moves phosphate and glucose into cells. This so-called refeeding syndrome occurs when starving or chronically malnourished patients are re-fed or given intravenous (iv) glucose. Phosphate deficiency commonly impairs neurological function, which may be manifested by confusion, seizures, and coma. Peripheral neuropathy and ascending motor paralysis, similar to GuillainBarr syndrome, may also occur. Weakness of skeletal or smooth muscle is the most common clinical manifestation of phosphate deficiency. It can involve any muscle group, alone or in combination, ranging from ophthalmoplegia to proximal myopathy, to dysphagia or ileus. Respiratory insufficiency may occur in some patients with severe hypophosphataemia, particularly when the underlying cause is malnourishment. Impaired cardiac contractility occurs, leading to generalised signs of myocardial depression. The hypophosphataemic myocardium also has a reduced threshold for ventricular arrhythmias. Incorrect A 60 kg man suffers 20 % burns. What is the estimated volume of intravenous fluid replacement that should be administered in the first 8 hours from the time of the burn?

Single best answer question choose ONE true option only 800 1,000 ml

1,000 1,200 ml

1,200 2,400 ml Correct answer 2,400 4,800 ml Your answer 4,800 5,000 ml

Intravenous fluids (crystalloid or colloid) should be administered if burns of greater than 15 % in adult or 10 % in paediatric patients are present. The rate of fluid administration ultimately employed is dependent on clinical indices, such as urine output, capillary refill and peripheral perfusion, central venous pressure and core:peripheral temperature differentials.

Various formulae are available for estimating initial rates of intravenous fluid replacement in burns victims. These initial rates of fluid administration are then modified based on clinical response.

Two widely-used formulae are as follows:-

Parkland formula : 2 4 ml/kg/%burn (full or deep partial thickness) in first 24 h from time of burn. Half of this calculated volume (crystalloid) should be administered in the first 8 h and the remainder administered in the subsequent 16 h.

Mount Vernon Formula This formula subdivides fluid administration into discrete time periods over the first 24 h: 4, 4, 4, 6, 6 and 12 h from the time of burn. The amount of fluid (colloid) administered in each of these periods is calculated as: (patient weight in kg x %burn)/2 Correct

A 3-week-old baby exhibits projectile vomiting shortly after feeding and failure to thrive. On examination an olive-shaped mass is palpable in the right upper quadrant of the abdomen. A clinical diagnosis of pyloric stenosis is made. What biochemical laboratory features would support the diagnosis?

Single best answer question choose ONE true option only Hypokalaemia, metabolic alkalosis, low urinary pH Your answer Hyperkalaemia, metabolic acidosis, high urinary pH

Hypokalaemia, metabolic acidosis, high urinary pH

Hyperkalaemia, metabolic alkalosis, low urinary pH

Hypokalaemia, metabolic alkalosis, high urinary pH

Following a diagnosis of pyloric stenosis, the first concern is to correct the metabolic abnormalities that invariably coexist with the condition. The serum electrolytes and capillary gases should be measured and corrected prior to surgery.

With prolonged vomiting, the infant becomes dehydrated, with a hypochloraemic metabolic alkalosis. The alkalosis is a result of loss of unbuffered hydrogen ions in gastric juice with concomitant retention of bicarbonate.

Fluid loss stimulates renal sodium reabsorption, but sodium can only be reabsorbed either with chloride, or in exchange for hydrogen and potassium ions (to maintain electroneutrality). Gastric juice has a high concentration of chloride and patients losing gastric secretions become hypochloraemic. This means that less sodium than normal can be reabsorbed with chloride.

However, it appears that the defence of extracellular fluid volume takes precedence over acidbase homeostasis and further sodium reabsorption occurs in exchange for hydrogen ions (perpetuating the alkalosis) and potassium ions (leading to potassium depletion). This explains the apparently paradoxical finding of acidic urine in patients with pyloric stenosis. Potassium is also lost in the gastric juice and thus patients frequently become potassium-depleted and yet are losing potassium in their urine. Correct A 75-year-old woman is being followed by her GP for suspected developing primary hypothyroidism.

Which of the following biochemical changes would you most expect to occur first? Single best answer question choose ONE true option only

Fall in serum free thyroxine

Fall in serum thyroxine-binding globulin

Fall in serum free triiodothyronine

Fall in serum total triiodothyronine

Increase in serum TSH Your answer Hypothyroidism develops gradually, often over many months or even years. In the early stages, free thyroxine concentrations are maintained in the normal range by the increased secretion of TSH. Patients with a slightly elevated TSH and lownormal thyroxine are said to have compensated or borderline hypothyroidism. In some individuals, it appears that this state can be maintained without progression to frank hypothyroidism. Triiodothyronine concentrations tend to fall later than thyroxine concentrations in hypothyroidism; the concentration of thyroxine-binding globulin does not change significantly. Incorrect The acute blood loss of 1.5 liters leads to a decrease in? Single best answer question choose ONE true option only The rate of oxygen extraction by the peripheral tissues

Renin secretion

Platelet count Your answer The cardiac output Correct answer Coronary and cerebral blood flow due to sympathetic overactivity

The rate of oxygen extraction by the peripheral tissues is increased in response to acute blood loss, renin secretion is also increased due to renal hypoperfusion.

Platelet count is increased and cardiac output decreased as the stroke volume decreases.

The blood flow to the brain and the heart remains unchanged. Correct A patient receives too many infusions after an operation resulting in a 20% increase in his blood volume.

What is the physiological process that is most likely to correct this abnormality? Single best answer question choose ONE true option only

Reduced activity of arterial pressure sensors

Increased activity of renal sympathetic nerves

Aldosterone release

Atrial natriuretic peptide (ANP) release Your answer Venous dilatation

The atria contain granulated cells that release peptides, atrial natriuretic peptide (ANP), in response to stretch. This natriuretic agent also relaxes the peripheral vasculature and thereby opposes the actions of the sympathetic and reninangiotensin systems. Correct A 54-year-old woman has undergone some blood tests as part of an employment health screen. She reports she is in good health and, being very health conscious, takes regular vitamin and mineral supplements. She is taking bendrofluazide 2.5 mg for hypertension and her blood pressure is 132/82 mmHg. The only abnormality is a serum calcium concentration of 2.94 mmol/l.

Which of the following is the most likely cause? Single best answer question choose ONE true option only

Diuretic treatment

High dietary calcium intake

High dietary vitamin D intake

Occult malignancy

Primary hyperparathyroidism Your answer Thiazides can cause hypercalcaemia but it is usually only mild. Vitamin D itself is physiologically inactive and, whereas 1-hydroxylated derivatives can be a cause of hypercalcaemia, vitamin D which has to be metabolised to activate it is less commonly so. Intestinal absorption of calcium is subject to tight control, and a high intake does not cause hypercalcaemia. The two most common causes of hypercalcaemia are primary hyperparathyroidism and malignancy. In an asymptomatic individual, primary hyperparathyroidism is the more likely cause. Correct A 56-year-old woman sustains a myocardial infarction. ST elevation and Q waves are present in leads V4V6, I and AVL.

Which of the following aspects of the heart is most likely to have been involved in the infarct? Single best answer question choose ONE true option only Anterior

Anterolateral Your answer Anteroseptal

Inferior

Lateral

This combination suggests an anterolateral infarct. Purely anterior infarcts tend to involve the chest leads only (typically V2V5), anteroseptal V1V3, lateral infarcts chest leads only (I, II, AVL) and inferior infarcts II, III and AVF.

Correct A parathyroid adenoma will be most likely to cause Single best answer question choose ONE true option only Decreased osteoclastic activity

Decreased urinary phosphate excretion

Hypocalcaemia

Increased osteoblastic activity

Increased osteoclastic activity Your answer The parathyroid glands produce parathyroid hormone (PTH) in response to serum calcium levels via a negative feedback mechanism. High levels of serum Ca2+ inhibit PTH secretion, and low levels stimulate PTH secretion. The response to Ca2+ levels is very rapid, so effects are seen very quickly after removal of the glands.

PTH affects calcium levels by its action on the bone, kidney and gut.

In bone, increased osteoclastic activity causes calcium levels to rise. This is due firstly to acid secretion onto the bone surface, and secondly to proteases dissolving the matrix.

In the kidney, PTH controls the hydroxylation of 25,hydroxy cholecalciferol D to 1,25 hydroxy cholecalciferol. This has the indirect effect of increasing calcium uptake in the gut. In the proximal tubule, PTH increases the urinary excretion of phosphate, which in turn increases the ionisation of calcium. There is also an increase in Ca2+ reabsorption in the distal tubule. Bicarbonate resorption is inhibited in the kidney, causing a hyperchloraemic acidosis which increase calcium ionisation and resorption from bone.

PTH excess therefore causes hypercalcaemia, hypophosphataemia and hyperchloraemia, as well as raised urinary phosphate. Correct A 20-year-old man presents with mild jaundice following a flu-like illness. Following review by a gastroenterologist, he has been told that a diagnosis of Gilberts syndrome is probable.

Which laboratory test is most likely to confirm this diagnosis? Single best answer question choose ONE true option only

Absence of bilirubin in the urine

Your answer Decreased serum haptoglobin concentration

Elevated serum aspartate aminotransferase (transaminase, AST) activity

Increased reticulocyte count

Increased urinary urobilinogen excretion

In Gilberts syndrome, the excess bilirubin is unconjugated, and does not appear in the urine. The same is true for jaundice secondary to haemolysis. However, in haemolytic jaundice, urinary urobilinogen is increased (increased production of bilirubin, and hence of urobilinogen), the reticulocyte count may be elevated and serum haptoglobin concentration decreased. Haemolysis may also cause a slight increase in serum aminotransferase (transaminase) activity. Correct Which of the following physiological responses occur in an acute hypoglycaemic episode?

Single best answer question choose ONE true option only A rise in serum insulin

A decrease in liver glycogen

A decrease in serum glucagon

A rise serum adrenaline Your answer A rise in serum ketone bodies Acute hypoglycaemia commonly occurs in insulin dependant diabetic patients who fail to match their carbohydrate intake with their insulin dose. It also occurs in patients with beta cell pancreatic tumours (insulinoma) due to a pathological overproduction of insulin. The acute response to hypoglycaemia is the result of an increase in serum adrenaline, glucagon (both of which are gluconeogenic) and due to a lack of glucose available for the brain (termed neuroglycopenia). These result in flight or fright symptoms, the feeling of hunger and a variety of neurological symptoms including blurred vision, slurred speech and impaired mental function. Correct Which of the following is not a well recognised feature of excessive glucocorticoid levels? Single best answer question choose ONE true option only

Hypertension Hyperglycaemia Alopecia Your answer Acne Reply to HalaReport

Post #14 Hala Adel wrote5 hours ago Osteoporosis Cortisol and its analogues are glucocorticoids and levels are raised either endogenously in Cushings disease, or exogenously causing Cushings syndrome. There are numerous side effects of glucocorticoid excess. Hypertension as a result of increased renal reabsorption of sodium and water. Hyperglycaemaia as a result of mineralocorticoid activity. Acne and hirsutism, not alopecia are a result of androgenic activity. Other side effects are osteoporosis, weakened skin, muscle wasting, immunosuppression and increased rates of infection, cataracts and fat redistribution to give the moon face and buffalo hump appearance. Correct Which of the following is not typically a cause of hypercalcaemia?

Single best answer question choose ONE true option only

Hyperparathyroidism

Hypothyroidism Your answer Milkalkali syndrome

Sarcoid

Squamous-cell carcinoma

Two of commonest causes of hypercalcaemia in the western world are primary hyperparathyroidism and malignancy. In primary hyperparathyroidism there is excess production of parathyroid hormone (PTH); although usually from a benign adenoma, this sometimes results from hyperplasia of the parathyroid glands and, in rare cases, a carcinoma. Thyrotoxicosis can cause hypercalcaemia as well as osteoporosis. The milkalkali syndrome can occur in patients who suffer from dyspepsia and drink milk and alkali-containing antacids, which may reduce the renal excretion of calcium. Around one-fifth of those with sarcoid have increased

calcium levels. Various mechanisms cause raised hypercalcaemia of malignancy. Correct Which of the following is the site of renin production? Single best answer question choose ONE true option only Collecting ducts Proximal convoluted tubule Loop of Henle Juxtaglomerular apparatus Your answer Liver The juxtaglomerular apparatus is formed of specialised juxtaglomerular cells in the wall of afferent arterioles and macula densa of the distal convoluted ducts. Renin secretion is stimulated by reduced renal perfusion. Angiotensinogen is produced by the liver and is catalysed by renin to form angiotensin I. This is in turn catalysed by angiotensin converting enzyme (ACE) to produce angiotensin. Angiotensin has several functions which aim to increase blood pressure and restore renal perfusion. It causes vasoconstriction, stimulates the adrenal cortex to produce aldosterone which promotes renal reabsorption of sodium and water from the distal convoluted tubules and collecting ducts. Incorrect Hypothyroidism due to disease of the thyroid gland is associated with increased plasma level of? Single best answer question choose ONE true option only Cholesterol Correct answer Albumin

RT3 Your answer Iodide

Thyroid binding globulin (TBG)

Thyroid hormone lowers circulating cholesterol level. The plasma cholesterol level drops before the metabolic rate rises. Incorrect Which one of the following is MOST likely to increase during exercise?

Single best answer question choose ONE true option only

Peripheral vascular resistance

Pulmonary vascular resistance

Stroke volume Correct answer Diastolic pressure

Venous compliance Your answer During exercise, increased oxygen consumption and increased venous return to the heart result in an increase in cardiac output and an increase in blood flow to both skeletal muscle and coronary circulation, when oxygen utilization is greatest. The increase in cardiac output is due to an increase in both heart rate and stroke volume. Systemic arterial pressure also increases in response to the increase in cardiac output. However, the fall in total peripheral resistance, which is caused by dilatation of the blood vessels within the exercising muscles, results in a decrease in diastolic blood pressure. The pulmonary vessels undergo passive dilatation as more blood flows into the pulmonary circulation. As a result, pulmonary vascular resistance decreases. The decrease in venous compliance, caused by sympathetic stimulation, helps to maintain ventricular filling during diastole. Correct The function of luteinising hormone in the male is: Promotion of spermatogenesis Stimulation of testosterone secretion Your answer Promotion of adrenal androgen secretion Stimulation of Sertolis cells to produce inhibin Promotion of spermiogenesis Follicle-stimulating hormone (FSH) and testosterone are required for spermatogenesis (division of spermatogonia to form spermatids) and spermiogenesis (maturation of spermatids to mature sperm). FSH also stimulates Sertolis cells to produce androgen-binding proteins and inhibin. Adrenal androgen secretion is not affected by luteinising hormone. Incorrect Pulmonary gas exchange occurs under which of the following physiological principles? Single best answer question choose ONE true option only Gas exchange can occur in the final seven branches of the bronchoalveolar tree Correct answer The first 12 branches of the bronchial tree are collectively known as the conducting zone

The equilibration of gases takes about 2.5 s in the resting lung

Only about 0.15% of oxygen is carried in solution in the plasma

Carbon dioxide is less water-soluble than oxygen

Gas exchange can occur in the final seven branches of the bronchoalveolar tree (the respiratory zone). The first 16 branches of the bronchial tree are collectively known as the conducting zone. The equilibration of gases takes about 0.25 s in the resting lung. Only about 1.5% of oxygen is carried in solution in the plasma. Carbon dioxide is more water-soluble than oxygen, between 5 and 10% of and this is the predominant method of carriage of CO2 is carried in dissolved form. Correct Reply to HalaReport

Post #15 Hala Adel wrote5 hours ago Splenectomy increases susceptibility to which of the following organisms?

Single best answer question choose ONE true option only Streptococcus pyogenes

Schistosoma haematobium

Bacteroides fragilis

Neisseria meningitidis Your answer Staphylococcus aureus

The spleen plays an important role in the removal of dead and dying erythrocytes and in the defence against microbes. Removal of the spleen (splenectomy) leaves the host susceptible to a wide array of pathogens, but especially to encapsulated organisms.

Certain bacteria have evolved ways of evading the human immune system. One way is through the production of a slimy capsule on the outside of the bacterial cell wall. Such a capsule resists phagocytosis and ingestion by macrophages and neutrophils. This allows them not only to escape direct destruction by phagocytes, but also to avoid stimulating T-cell responses through the presentation of bacterial peptides by macrophages. The only way that such organisms can be defeated is by making them more palatable by coating their capsular polysaccharide surfaces in opsonising antibody.

The production of antibody against capsular polysaccharide primarily occurs through T-cell independent mechanisms. The spleen plays a central role in both the initiation of the antibody response and the phagocytosis of opsonised encapsulated bacteria from the bloodstream. This helps to explain why the asplenic individuals are most susceptible to infection from encapsulated organisms, notably Streptococcus pneumoniae (pneumococcus), Neisseria meningitidis (meningococcus) and Haemophilus influenzae.

The risk of acquiring such infections is reduced by immunising individuals against such organisms and by placing patients on prophylactic penicillin, in most cases for the rest of their

lives. In addition, asplenic individuals should be advised to wear a MedicAlert bracelet to warn other health care professionals of their condition. Correct Which ECG feature is classically present in hypothermia? Single best answer question choose ONE true option only Thyroxine

Reduced PR interval

Tachycardia

U waves

J waves Your answer The J wave may be present on the ECG in patients with hypothermia and is an additional upward peak immediately following the QRS complex. The U wave may be present on the ECG in hypokalaemia and is an additional upward peak which follows the T wave. Tachycardia and a reduction in the RR interval are ECG features of hyperthermia. Incorrect Which of the following is a function of atrial natruretic peptide (ANP)? Single best answer question choose ONE true option only Increases renin secretion Decreases aldosterone secretion Correct answer Promotes the effects of antidiuretic hormone (ADH) Your answer Causes renal vasoconstriction Promotes the feeling of thirst

ANP is released from atrial muscle cells when the atria are stretched due to increased circulating blood volume. Therefore ANP works to reduce blood volume by inhibiting the release of renin, aldosterone and ADH resulting in increased sodium and water excretion. It promotes renal vasodilatation. Correct Concerning the salivary glands Single best answer question choose ONE true option only They secrete around 150 ml of saliva per day

They secrete saliva with a pH of 4-5

They secrete saliva which is hypertonic

They are supplied by the parasympathetic nervous system Your answer

They secrete saliva containing trypsinogen

Saliva is secreted from the acini, and transported via the salivary ducts to the oral cavity. The secretion from the sublingual gland is predominately mucous, the parotid serous and the submandibular mixed. The pH of saliva varies from 7-8, and around 1.5L is produced per day. As well as a-amylase, saliva contains lipase and glycoproteins to lubricate food and protect the oral mucosa. Lysozyme, IgA and lactoferrin act as bacteriostatic agents, and proteins protect the tooth enamel.

The saliva is isotonic when it is excreted from the acini; Na+ and Cl- are exchanged for K+ and HC03- in the ducts, and the saliva becomes hypotonic by the time it reaches the mouth. Correct Which of the following is the most important direct stimulus to respiration? Single best answer question choose ONE true option only Increased pCO2 of the CSF

Increased H+ concentration of the CSF Your answer Decreased arterial pO2

Decreased arterial pH

Decreased arterial pCO2

Chemoreceptors involved with the control of respiration are present in the central nervous system and peripherally. The central chemoreceptors are situated in the ventral medulla, and increase firing in response to the H+ concentration of the brain extra cellular fluid, which is directly related to the H+ concentration in the CSF. CO2 / HCO3 cannot cross the blood brain barrier, but CO2 does so readily. This frees H+ ions, causing a low CSF pH, increased firing of the central chemoreceptors and increased ventilation.

Peripheral chemoreceptors are found in the carotid bodies and aortic arch, and increase their firing rate in response to decreased PaO2, decreased arterial pH and increased paCO2. These are much less important, however, in stimulating respiration than the central chemoreceptors. Correct Vasopressin (ADH) Single best answer question choose ONE true option only Is synthesised in the posterior pituitary gland

Deficiency leads to a risk of water intoxication

Excessive secretion usually results in diabetes insipidus

Increased plasma osmolarity is the primary physiological stimulus Your answer Acts on the proximal convoluted tubules of the kidney

Vasopressin is synthesised in the supraoptic nucleus of the hypothalamus and transported to the posterior pituitary via the axons. Excessive secretion is associated with the risk of impaired water excretion. Diabetes insipidus results from deficient secretion or action of this hormone leading to thirst and polyuria. It acts mainly on the distal convoluted tubules and the collecting ducts of the kidney. Incorrect The following metabolic changes occur in the ebb phase (first 24 hours) of response to injury? Single best answer question choose ONE true option only Plasma pH increases

The plasma level of free fatty acids decreases

Hypoglycemia

The plasma level of non protein nitrogen decreases Your answer Plasma glycerol increases Correct answer There is usually acidosis (pH decreases). Lipolysis increases leading to increase in fatty acids and glycerol. There is hyperglycemia and an increased level of non protein nitrogen. Incorrect Which of the following fluids would be the most appropriate to replace the fluid being lost in a patient with a paralytic ileus draining 2 litres of fluid a day through a nasogastric tube?

Single best answer question choose ONE true option only

Compound sodium lactate (Hartmanns solution) Your answer 5% dextrose

10% dextrose

0.18% sodium chloride with 4% dextrose (dextrose saline)

0.9% sodium chloride (normal saline) Correct answer In this situation, it is essential to supply sufficient chloride ions to replace the chloride being lost in the gastric fluid (gastric juice is essentially dilute hydrochloric acid). If this is not done, a metabolic alkalosis can ensue. The appropriate fluid is normal saline. The two dextrose solutions contain no chloride, and dextrose saline contains insufficient for this purpose. Hartmanns solution could exacerbate any tendency to alkalosis as the lactate it contains is metabolised to bicarbonate. Correct Which one of the following is higher at the apex of the lung than at the base when a person is standing? Single best answer question choose ONE true option only

V/Q ratio Your answer Ventilation

PaCO2

Compliance

Blood flow

The alveoli at the apex of the lung are larger than those at the base so their compliance is less. Because of the reduced compliance, less inspired gas goes to the apex than to the base. Also, because the apex is above the heart level, less blood flows through the apex than through the base. However, the reduction in air flow is less than the reduction in blood flow, so that the V/Q ratio at the top of the lung is greater than it is at the bottom. The increased V/Q ratio at the apex makes PaCO2 lower and PaO2 higher at the apex than they are at the base Correct A 49-year-old postmenopausal woman of Southern Asian origin complains of muscle weakness. She is found to have hypocalcaemia, and X-ray examination reveals two Loosers zones in her left upper femur.

A defect in which of the following physiological processes is most likely to be the cause of her illness? Single best answer question choose ONE true option only.

Absorption of calcium from the gut Your answer Osteoblastic activity

Osteoclastic activity

Parathyroid hormone secretion

Renal excretion of calcium

The findings in this woman suggest osteomalacia, and the most important reason for the impaired mineralisation of bone is reduced intestinal calcium absorption consequent on vitamin D deficiency. The decreased availability of calcium to mineralise bone leads to increased osteoblastic activity (and hence increased osteoid formation). Hypocalcaemia causes increased parathyroid hormone secretion (secondary hyperparathyroidism), which stimulates renal calcium reabsorption (hence reduced excretion). Thus, while this woman may have increased osteoblastic activity and increased PTH secretion, both these are secondary to vitamin D deficiency and decreased intestinal absorption of calcium. Osteoporosis (postmenopausal osteoporosis is due to increased osteoclastic activity) is not associated with hypocalcaemia. Incorrect In relation to the nutritional physiology of patients, which of the following would represent appropriate nitrogen requirements (g N/kg per day) and calorie requirements (kcal/kg per day)?

Single best answer question choose ONE true option only

Reduced food intake: nitrogen requirement 0.3 g N/kg per day, calorie requirement 35 kcal/kg per day Your answer Moderate injury: nitrogen requirement 0.15 g N/kg per day, calorie requirement 25 kcal/kg per day

Moderate sepsis: nitrogen requirement 0.3 g N/kg per day, calorie requirement 15 kcal/kg per day

Severe injury: nitrogen requirement 0.3 g N/kg per day, calorie requirement 35 kcal/kg per day Correct answer Severe sepsis: nitrogen requirement 0.2 g N/kg/day, calorie requirement 15 kcal/kg/day

Usual ranges for:

reduced food intake: nitrogen requirement 0.150.2 g N/kg per day calorie requirement 2530 kcal/kg per day moderate injury/sepsis: nitrogen requirement 0.20.3 g N/kg per day calorie requirement 3035 kcal/kg per day severe injury/sepsis: nitrogen requirement 0.30.35 g N/kg per day calorie requirement 3540 kcal/kg per day Correct You are called to see a 56-year-old man 2 h after a cardiac catheterisation. He is actively bleeding from his catheter site and his dressings and bedclothes are soaked with blood.

Which of the following statements is true? Single best answer - choose ONE true option only

Grade I shock applies with up to a 20% loss of circulating blood volume

Loss of 2 litres of blood is consistent with normal systolic blood pressure

The pulse can remain normal in patients with grade I shock Your answer Anuria is pathognomonic of grade III shock

Grade IV shock is seen with a 30% loss of circulating blood volume

Grade I shock Loss of up to 15% (750ml) of blood volume; blood pressure is normal but there may be a slight tachycardia Grade II shock 1530% (750 ml 1.5 l) blood-volume loss, systolic blood pressure is usually normal but a tachycardia is present Grade III shock 3040% (1.52 litres) loss, hypotension, tachycardia and fall in urine output seen Grade IV shock > 40% (> 2 l) blood-volume loss, anuria and severe shock observed Correct Formation of the external genitalia in the male fetus is dependent on: Testosterone Adrenal androgens The Y chromosome Dihydrotestosterone Your answer Mullerian inhibiting substance Dihydrotestosterone is the most potent androgen and is responsible for development of the external genitalia in the f Incorrect Lung compliance:

Single best answer question choose ONE true option only Is defined as the change in pressure per unit volume

Your answer Is synonymous with elastance

Is increased in emphysema Correct answer Is equal in inflation and deflation

Is reduced by the presence of surfactant

Compliance is expressed as volume change per unit change in pressure. Elastance is the reciprocal of compliance. The pressure-volume curve of the lung is non-linear with the lungs becoming stiffer at high volumes. The curves which the lung follows in inflation and deflation are different. This behaviour is known as hysteresis. The lung volume at any given pressure during deflation is larger than during inflation. This behaviour depends on structural proteins (collagen, elastin), surface tension and the properties of surfactant.

Surfactant is formed in and secreted by type II pneumocytes. The active ingredient is dipalmitoyl phosphatidylcholine. It helps prevent alveolar collapse by lowering the surface tension between water molecules in the surface layer. In this way it helps to reduce the work of breathing (makes the lungs more compliant) and permits the lung to be more easily inflated.

Various disease states are associated with either a decrease or increase in the lung compliance. Fibrosis, atelectasis and pulmonary oedema all result in a decrease in lung compliance (stiffer lungs). An increased lung compliance occurs in emphysema where an alteration is elastic tissue is probably responsible (secondary to the long term effects of smoking). The lung effectively behaves like a soggy bag so that a given pressure change results in a large change in volume (i.e. the lungs are more compliant). However, during expiration the airways are less readily supported and collapse at higher lung volumes resulting in gas trapping and hyperinflation. Incorrect The infusion of 1 litre of which of the following solutions will initially lead to the greatest increase in extracellular fluid volume?

Single best answer question choose ONE true option only Gelatin colloid solution (e.g. Gelofusin or Haemaccel) Your answer Hypertonic NaCl Correct answer Normal (0.9 %) NaCl

5 % dextrose solution

Pure water

Colloids may be natural (e.g. blood, human albumin and gelatins) or synthetic (e.g. dextrans). They comprise large branching molecules with molecular weights in excess of 30,000. Assuming intact capillary integrity, the volume effects of colloid infusion are, at least initially, confined to the plasma compartment. In contrast, crystalloids, such as NaCl solution, pass more readily from the plasma fluid compartment and have more of a volume effect on the extracellular fluid compartment. In the case of 5 % dextrose solution, the dextrose component is rapidly metabolised and the remaining water distributes itself throughout the entire body water (i.e. intracellular and extracellular compartments).

Therefore, of the options listed above, infusions of NaCl will have the greatest initial increase in extracellular fluid volume. Hypertonic NaCl will have an even greater effect than normal (approximately isotonic) NaCl, since hypertonic solutions will draw additional water from the intracellular fluid compartment by osmosis. Correct Reply to HalaReport

Post #16 Hala Adel wrote5 hours ago Single best answer question choose ONE true option only CD4 T-cells

CD8 T-cells Your answer B cells

TH1 cells

TH2 cells

Lymphocytes can be divided into two main subtypes T cells and B cells. B cells (or plasma cells) secrete antibodies. T cells can be divided into two further subtypes CD4 T-cells and CD8 T-cells. CD4 (helper) Tcells can recognise antigen only in the context of MHC Class II, whereas CD8 (cytotoxic) T-cells recognise cell-bound antigens only in association with Class I MHC. This is known as MHC restriction.

CD4 and CD8 T-cells perform distinct but somewhat overlapping functions. The CD4 helper T-cell can be viewed as a master regulator. By secreting cytokines (soluble factors that mediate communication between cells), CD4 helper T-cells influence the function of virtually all other cells of the immune system including other T-cells, B-cells, macrophages and natural killer cells. The central role of CD4 cells is tragically illustrated by the HIV virus which cripples the immune system by selective destruction of this T-cell subset. In recent years two functionally different populations of CD4 helper T-cells have been recognised TH1 cells and TH2 cells, each characterised by the cytokines that they produce. In general, TH1 cells facilitate cell-mediated immunity, whereas TH2 cells promote humoral-mediated immunity.

CD8 cytotoxic T-cells mediate their functions primarily by acting as cytotoxic cells (i.e. they are T-cells that kill other cells). They are important in the host defence against cytosolic pathogens. Two principal mechanisms of cytotoxicity have been discovered perforin-granzyme-dependent killing and Fas-Fas ligand dependent killing. Correct Reply to HalaReport

Post #17 Hala Adel wrote5 hours ago MRCS Part 1 Practice Questions ( Anatomy ) - 1 of 2 Here are some questions for revision: MRCS Part 1 - Anatomy MTF The right common carotid artery Incorrect bifurcates at the level of the upper border of the cricoid cartilage TrueFalse Incorrect is a branch of the aortic arch TrueFalse Incorrect has the cervical sympathetic chain as an anterior relation TrueFalse Incorrect lies lateral to the lateral lobe of the thyroid gland TrueFalse Incorrect is separated from the phrenic nerve by the prevertebral fascia TrueFalse Incorrect is enclosed within the carotid sheath throughout TrueFalse The right common carotid artery branches off the brachiocephalic artery. It bifurcates at the level of the upper border of the lamina of the thyroid cartilage. It lies posterior to the lobes of the thyroid gland and anterior to both the cervical sympathetic chain and the phrenic nerve on the scalenus anterior muscle; the latter is separated from the artery by prevertebral fascia. Recognised complications of sclerotherapy for varicose veins include Incorrect trash foot TrueFalse Incorrect brown discoloration of the skin TrueFalse Incorrect deep vein thrombosis (DVT) TrueFalse Incorrect ulceration of the skin TrueFalse Incorrect Sudeck's dystrophy TrueFalse Extravasation of the sclerosing agent may cause skin damage and ulceration. Patients should be warned about the possibility of brown pigmentation of the skin. Sclerotherapy is indicated for residual and recurrent varicosities after varicose vein surgery. Sudeck's atrophy is a recognised complication of trauma. The superior mesenteric artery Incorrect supplies the entire ileum and jejunum TrueFalse Incorrect

lies to the left of the inferior mesenteric artery TrueFalse Incorrect passes posterior to the splenic vein TrueFalse Incorrect lies to the right of the superior mesenteric vein TrueFalse Incorrect crosses anterior to the third part of the duodenum TrueFalse The superior mesenteric artery supplies the gut from the mid-second part of the duodenum to a level just short of the splenic flexure of the colon. It is directed downwards behind the splenic vein and by the pancreas, with the superior mesenteric vein on its right side. It lies anterior to the third part of the duodenum. Clinical signs suggestive of a urethral injury include Incorrect blood at the external urethral meatus TrueFalse Incorrect a butterfly haematoma TrueFalse Incorrect Battles sign TrueFalse Incorrect high riding prostate TrueFalse Incorrect haematuria TrueFalse The inability to void, an unstable pelvic fracture, blood at the external urethral meatus, a butterfly haematoma, or a high riding prostate on digital rectal examination (DRE) are indications for the surgeon to request a retrograde urethrogram to confirm that the urethra is intact prior to inserting a urethral catheter. In the case of a disrupted urethra a suprapubic catheter should be inserted. Reflux oesophagitis Incorrect is always present with hiatus hernia TrueFalse Incorrect is preferably treated with surgery TrueFalse Incorrect if untreated may cause stricturing of the oesophagus TrueFalse Incorrect does not require more than alteration of lifestyle to treat TrueFalse Incorrect is treated surgically principally by attempting to narrow the gastro-oesophageal junction TrueFalse The mainstay of treatment for symptomatic reflux oesophagitis is with acid suppression therapy. If untreated, structuring is common. Surgery restores adequate oesophageal length (high pressure) in the abdomen. The following statements concern the root values of peripheral nerves: Incorrect sciatic nerve (L4,5,S1,2) TrueFalse Incorrect phrenic nerve (C2,3,4) TrueFalse Incorrect

iliohypogastric nerve (L1) TrueFalse Incorrect obturator nerve (L2,3,4) TrueFalse Incorrect medial plantar nerve (L4,5) TrueFalse The sciatic nerve is L4,5,S1,2,3; and the phrenic nerve C3,4,5. Pes planus Incorrect is the condition where the medial border of the foot is in contact with the ground when standing TrueFalse Incorrect is often caused by a bony bridge between talus and calcaneus TrueFalse Incorrect typically presents with pain around age 6 years TrueFalse Incorrect can be treated by arthrodesis of thesubtalar and midtarsal joints TrueFalse Incorrect may be due to peroneal muscle paralysis TrueFalse Pes planus = pes valgus = flat foot. The whole foot is rotated into eversion around its longitudinal axis. It is asymptomatic in the vast majority of cases. There are two types: mobile and rigid. Rigid flat foot is often caused by synostosis between two of the tarsal bones: talocalcaneal and talonavicular. There is typically pain and limitation of movement in the foot around age of 12 years. Triple fusion is sometimes necessary if pain is the predominant feature, but most symptomatic cases are treated conservatively with splintage or plaster. Spasmodic flat foot is due to contraction of the peroneal muscles. Coronary artery bypass grafting is the usual form of treatment for patients with Incorrect stable angina and triple vessel coronary artery disease TrueFalse Incorrect single or double vessel coronary artery disease TrueFalse Incorrect stenosis of the left main coronary artery TrueFalse Incorrect post-myocardial infarction unstable angina TrueFalse Incorrect valvular heart disease requiring surgery and coronary artery disease TrueFalse According to the Coronary Artery Surgery Study the patient groups that derive particular benefit from coronary artery bypass grafting (CABG) are those with triple vessel disease, and those with >50% left main stem stenosis. Those with single or double vessel disease are usually more amenable to percutaneous intervention. Post-myocardial infarction, unstable angina is a primary indication for urgent CABG. Valvular disease with concomitant coronary artery disease is usually treated operatively. The thoracic duct Incorrect lies on the posterior intercostal vessels TrueFalse Incorrect has no valves TrueFalse Incorrect runs through the thoracic inlet to the left of the oesophagus TrueFalse

Incorrect receives the right bronchomediastinal lymph trunk TrueFalse Incorrect arches over the left suprapleural membrane TrueFalse The cisterna chyli runs between the aorta and the right crus of the diaphragm, passes through the aortic diaphragm opening and drains into the thoracic duct. The thoracic duct ascends anterior to the posterior intercostal vessels and has several valves. At the thoracic inlet, it lies to the left of the oesophagus and arches forward over the dome of the left pleura, draining into the left brachiocephalic vein. The right bronchomediastinal trunk drains into the right subclavian vein. Which of the following are correct? The internal carotid artery Incorrect Commences at the level of C6 TrueFalse Incorrect Passes through the foramen ovale TrueFalse Incorrect Has no extra-cranial branches TrueFalse Incorrect Gives off the ophthalmic artery TrueFalse Incorrect Divides into the middle and anterior cerebral arteries TrueFalse The common carotid artery bifurcates into the external and internal carotids at the level of the upper part of the C4 vertebra ie the upper border of the thyroid cartilage, however this bifurcation is frequently higher, near the tip of the great horn of the hyoid bone (C3 level). The internal carotid artery has no extra cranial branches and enters the base of the skull in the petrous temporal bone through the carotid canal. The internal carotid on entering the skull passes forwards through the temporal bone upwards into the cavernous sinus, turns forward and upwards through the roof of the sinus to lie medial to the anterior clinoid process before turning back on itself above the cavernous sinus and then passing once more lateral to the optic chiasma to end by dividing into the anterior and middle cerebral arteries. The ophthalmic artery originates from the internal carotid artery immediately above the roof of the cavernous sinus. Regarding an anterior dislocation of the shoulder: Incorrect it commonly occurs after an epileptic fit TrueFalse Incorrect it may produce decreased sensation over the lateral aspect of the deltoid muscle TrueFalse Incorrect it always needs reduction under a general anaesthetic TrueFalse Incorrect it is not associated with any fractures TrueFalse Incorrect it occurs less commonly than a inferior dislocation of the shoulder TrueFalse Dislocation of the shoulder can occur in thee directions: most common is anteriorly, followed by posteriorly and rarely inferiorly. An anterior dislocation produces a flattening in the deltoid muscle; the axillary nerve may be injured causing decreased sensation in the lateral aspect of this muscle (regimental badge area). Anterior dislocations may be associated with a compressional fracture of the humeral head known as a HillSach deformity. Posterior dislocation is more difficult to diagnose; this occurs more commonly following seizures and has a characteristic light bulb appearance due to rotation of the upper end of the humerus.

With reference to complications of total hip arthroplasty, the following are true: Incorrect sciatic nerve injury is a recognised complication TrueFalse Incorrect loosening is the commonest cause of long-term failure TrueFalse Incorrect there is no evidence that prophylactic antibiotics reduce infection rate TrueFalse Incorrect unfractionated heparin is more effective in preventing deep vein thrombosis (DVT) than low molecular weight heparin TrueFalse Incorrect uncemented arthroplasty has better survival than cemented arthroplasty TrueFalse Most dislocations occur within 6 months of surgery and are treated conservatively. Sciatic nerve injury complicates 1% of cases. Prophylactic antibiotics, gentamicin-impregnated cement and ultra-clean air enclosures have reduced infection rate. Loosening may be due to absorption of cement around the implant, hypersensitivity, low grade infection and/or imperfect prosthetic design, and is found in about 20% of patients 10 years post-operatively. Cemented arthroplasty is generally considered to be better than uncemented arthroplasty. The submandibular gland Incorrect lies below the digastric muscle TrueFalse Incorrect has the hypoglossal nerve running through it TrueFalse Incorrect lies both below and above the lower mandible TrueFalse Incorrect is superficial to the hyoglossus muscle TrueFalse Incorrect has the facial artery running through it TrueFalse The submandibular gland consists of a deep and a superficial part. The superficial part lies in the digastric triangle (above and between the two bellies of the digastric muscle). The hypoglossal nerve runs medial to the superficial part of the gland. The gland is superficial to the mylohyoid and hyoglossus muscles. A third of the submandibular gland lies below the lower border of the mandible and two-thirds above it. Injury to the sciatic nerve in the buttock causes Incorrect loss of active extension at the knee joint TrueFalse Incorrect loss of strength of the hamstring muscles TrueFalse Incorrect complete loss of sensation below the knee TrueFalse Incorrect weakness of dorsiflexion at the ankle joint TrueFalse Incorrect weakness of eversion of the foot TrueFalse The sciatic nerve, arising from nerve roots L4, L5, S13. It is really two nerves the tibial, and the common peroneal nerve which are bound together in the same connective tissue sheath. The tibial nerve supplies flexor muscles, and the common peroneal nerve supplies extensor and abductor muscles. The anterior femoral compartment containing the quadriceps extends the knee and is supplied by the femoral nerve.

Anatomy of the orbit Incorrect the supraorbital nerve passes through the superior orbital fissure TrueFalse Incorrect the ophthalmic artery passes through the superior orbital fissure TrueFalse Incorrect the optic nerve is surrounded by pia, arachnoid and dura mater TrueFalse Incorrect the frontal nerve passes through the tendinous membrane TrueFalse Incorrect the nasociliary nerve supplies the cornea TrueFalse Incorrect sectioning of the inferior ramus of the oculomotor nerve will produce a ptosis TrueFalse Incorrect the ophthalmic artery is a branch of the internal carotid artery TrueFalse The frontal nerve arises from the ophthalmic division of the trigeminal nerve in the lateral wall of the cavernous sinus. It enters the orbits through the superior orbital fissure. Just before it reaches the orbital margin it divides into the supratrochlear and supraorbital nerves. The supraorbital nerve passes through the supraorbital foramen, and supplies the skin of the forehead. The ophthalmic artery branches off the internal carotid artery at the cavernous sinus, and passes through the optic canal with the optic nerve. The optic nerve is surrounded by a sheath of pia, arachnoid and dura mater. The nasociliary nerve arises from the ophthalmic division of the trigeminal nerve in the lateral fourth of the cavernous sinus, and enters the orbit through the superior orbital fissure within the tendinous ring. The branches of the nasociliary nerve supply the ethmoidal sinuses, sphenoidal sinuses, skin of the upper eyelids and nose. The inferior ramus of the oculomotor nerve gives off branches to the inferior rectus, medial rectus and the inferior oblique muscles. The superior ramus of the oculomotor nerves supplies the levator palpebrae superioris, so may give rise to a ptosis if cut. Derivatives of the mesonephric ducts: Incorrect ureters TrueFalse Incorrect uterus TrueFalse Incorrect prostate TrueFalse Incorrect part of the vas deferens TrueFalse Incorrect part of the vagina TrueFalse The epididymis, vas deferens, seminal vesicle, ejaculatory duct and bladder trigone are derived from the mesonephric duct. The pancreas Incorrect overlies the right kidney TrueFalse Incorrect lies in the transpyloric plane TrueFalse Incorrect has an uncinate process lying anterior to the superior mesenteric vein TrueFalse Incorrect gives attachment to the transverse mesocolon TrueFalse

Incorrect has the inferior mesenteric vein passing behind the neck TrueFalse The head of the pancreas is related to the hilum but does not overlie the right kidney. It is, however, anterior to the left kidney. The transpyloric plane (L1) transects the pancreas obliquely, passing through the midpoint of the neck, with most of the head below the plane, and most of the body and tail above. The transverse mesocolon is attached to the head, neck and body of the pancreas. The uncinate process lies posterior to the superior mesenteric vessels, and the inferior mesenteric vein passes behind the body of the pancreas, where it joins the splenic vein. High anal fistula Incorrect Are more common than low fistula TrueFalse Incorrect Open into the rectum above the puborectalis muscle TrueFalse Incorrect Are associated with Crohns disease TrueFalse Incorrect May be laid open without hazard TrueFalse Incorrect Can be managed by a loose seton TrueFalse High fistulae are uncommon but may be due to carcinoma, diverticular disease, tuberculosis, Crohn's disease, ulcerative colitis, trauma or radiotherapy. Laying open divides the sphincter and produces incontinence. Osteochondritis of the navicular bone Incorrect is thought to be due to increase vascularity producing early calcification TrueFalse Incorrect mainly affects teenagers TrueFalse Incorrect usually resolves spontaneously in a year TrueFalse Incorrect usually presents with pain and a limp TrueFalse Incorrect treatment involves analgesia and continued activity TrueFalse Osteochondritis of the navicular bone is known as Khlers disease and affects children age 35 years. They complain of pain over the medial side of the foot and noticeably limp. It is thought to be due to a disturbance of the blood supply. Normally symptoms disappear after a few weeks of strapping the foot and restricting activity, but, rest in a cast may be necessary if there is severe pain. Eventually the foot becomes normal clinically and radiologically over a period of months. Which of the following are correct? Umbilical hernia in children and infants Incorrect Occur through the umbilical cicatrix TrueFalse Incorrect Is an example of a sliding hernia TrueFalse Incorrect Can be treated with a corset TrueFalse Incorrect Has a higher incidence in black than white children TrueFalse

Incorrect Can be treated with a Mayo repair TrueFalse An umbilical hernia protrudes through the umbilical cicatrix to lie in the subcutaneous tissues. They will often resolve as the child grows and few will require surgical treatmentthose that do can be repaired with the Mayo vest-over-pants approach. The surface of the right lung is indented by the Incorrect trachea TrueFalse Incorrect oesophagus TrueFalse Incorrect superior vena cava TrueFalse Incorrect right ventricle TrueFalse Incorrect subclavian vein TrueFalse Impressions on the mediastinal surface of the right lung include the trachea, vagus, superior vena cava, right atrium and subclavian artery. The oesophagus grooves the left lung above the arch of the aorta and below the hilum. For Le Fort I fractures, which of the following are true Incorrect Bilateral is more common than unilateral Le Fort I fractures TrueFalse Incorrect The fracture line passes above the palate TrueFalse Incorrect Is associated with Le Fort II injury TrueFalse Incorrect Intercanthal distance is usually increased TrueFalse Incorrect Airway should be protected with an oropharyngeal tube TrueFalse The French surgeon Le Fort performed experiments on cadavers in early 1900s and classified facial fractures into I, II and III. In Le Fort I injuries the fracture line passes above the palate, fracturing the pyrimadal processes of the maxilla on each side, the vomer and the lower parts of the pterygoid processes. Bilateral Le Fort I fractures represent 23% of these injuries, unilateral 11%. Le Fort I and II fractures occur together in 21% of cases. Le Fort II injuries involve the ethmoids and so increase intercanthal distance. Nasopharyngeal intubation is preferred to oropharyngeal intubation to secure the airway. Which of the following are true? The hepatic portal vein Incorrect Is formed by the union of the splenic and superior mesenteric veins TrueFalse Incorrect Runs behind the epiploic foramen TrueFalse Incorrect Lies posterior to the common hepatic artery TrueFalse Incorrect Lies anterior to the first part of the duodenum TrueFalse Incorrect Forms posterior to the neck of the pancreas TrueFalse

The portal vein forms the anterior boundary of the epiploic foramen, lying behind the bile duct and hepatic artery. It lies in front of the inferior vena cava, as it lies behind the pancreas and the first part of the duodenum. On the dorsum of the foot the Incorrect dorsalis pedis artery lies medial to the extensor hallucis longus tendon TrueFalse Incorrect deep peroneal nerve lies medial to the dorsalis pedis artery TrueFalse Incorrect L5 dermatome is present TrueFalse Incorrect great saphenous vein lies anterior to the medial malleolus TrueFalse Incorrect inferior extensor retinaculum loops under the medial longitudinal arch TrueFalse The dorsalis pedis artery lies between the extensor hallucis longus tendon medially, and the deep peroneal nerve lies laterally. The L5 dermatome lies over the medial half of the dorsum of the foot. The great saphenous vein is found anterior to the medial malleolus, and the lower limb of the extensor retinaculum passes under the medial longitudinal arch and blends with the plantar aponeurosis. Reply to HalaReport

Post #18 Hala Adel wrote5 hours ago Which of the following are true? Differential diagnosis of a femoral hernia includes Incorrect Varicocele TrueFalse Incorrect Psoas abscess TrueFalse Incorrect Saphena varix TrueFalse Incorrect Troisiers node TrueFalse Incorrect Spigelian hernia TrueFalse Differential diagnosis of a femoral hernia includes: inguinal hernia, saphena varix, lymphadenopathy, psoas abscess, lipoma, femoral aneurysm, sarcoma, ectopic testes and obturator hernia. Troisiers node is supraclavicular. Spigelian hernia arises from the semilunar line. Which of the following are correct? With regards to the anatomy of the pancreas Incorrect It lies along the transpyloric plane TrueFalse Incorrect Superior mesenteric vessels pass under the uncinate process TrueFalse Incorrect The inferior vena cava (IVC) is a posterior relation TrueFalse Incorrect The lesser sac is an anterior relation TrueFalse Incorrect

The portal vein is formed behind the pancreatic neck TrueFalse The pancreas has a head, neck, body and tail and lies along the transpyloric plane. The head is bound laterally by the curved duodenum and the tail extends to the hilum of the spleen. The superior mesenteric vessels pass behind the pancreas, then anteriorly, over the uncinate process and third part of the duodenum into the root of the small bowel mesentery. The inferior vena cava, coeliac plexus, left kidney, and the left adrenal gland are posterior pancreatic relations. Which of the following structures are parts of the hindbrain? Incorrect cerebellum TrueFalse Incorrect cerebral aqueduct TrueFalse Incorrect pons TrueFalse Incorrect basal ganglia TrueFalse Incorrect tectum TrueFalse The gross structure of the brain can be divided into the forebrain, midbrain and hindbrain. The main structures that form the hindbrain are the pons, medulla oblongata and cerebellum. The fourth ventricle and central canal are also found in this region. Concerning traumatic diaphragmatic injury, which of the following are true? Incorrect Is more commonly diagnosed on the left side TrueFalse Incorrect Blunt injuries produce larger tears than penetrating traumas TrueFalse Incorrect The commonest site for tears is the posterolateral aspect of the diaphragm TrueFalse Incorrect Insertion of a naso-gastric tube is absolutely contraindicated in left-sided diaphragmatic rupture TrueFalse Incorrect Abdominal computed tomography is the most sensitive investigation to identify diaphragmatic injury TrueFalse Diaphragmatic injuries result from either blunt or penetrating trauma. A traumatic diaphragmatic rupture is more commonly diagnosed on the left side, perhaps because the liver obliterates the defect or protects it on the right side. In addition, the appearance of bowel, stomach or a nasogastric (NG) tube is more easily detected in the left side of the chest. Right diaphragmatic ruptures are rarely diagnosed in the early post-injury period. The liver often prevents herniation of other abdominal organs into the chest. This, however, may not be representative of the true incidence of laterality and autopsy studies have revealed that leftand right-sided ruptures occur almost equally. Blunt trauma produces large radial tears measuring 515 cm, most often at the posterolateral aspect of the diaphragm. In contrast, penetrating trauma usually create only small linear incisions or perforations, which are less than 2 cm in size and may often take some time, even years, to develop into diaphragmatic hernias. If a laceration of the left diaphragm is suspected, a NG tube should be inserted. If the tube appears in the thoracic cavity on the chest film, the need for special contrast studies can be eliminated. Minimally invasive endoscopic procedures (thoracoscopy) may be helpful in evaluating the injury to the diaphragm in indeterminate cases. Abdominal computed tomography scan is usually not helpful because of its poor visualisation of the diaphragm. Magnetic resonance imaging is more accurate in visualising the anatomy of the diaphragm. It is very sensitive and specific and so is the investigation of choice. Surgical repair is necessary, even for small tears, because the defect will not heal spontaneously.

The superior vena cava (SVC) Incorrect has a valve at its entry into the left atrium TrueFalse Incorrect drains only the head, neck and upper body TrueFalse Incorrect receives the thoracic duct TrueFalse Incorrect ends behind the second costal cartilage TrueFalse Incorrect enters the heart at the level of the sternal angle TrueFalse The SVC drains all the structures above the diaphragm except the heart and lungs. It also receives the azygos vein, which drains the lumbar and subcostal regions. The SVC is formed behind the first costal cartilage by the union of the right and left brachiocephalic veins. It ends behind the third costal cartilage as it enters the right atrium. The SVC has no valves. The thoracic duct drains into the left brachiocephalic vein (or sometimes into the subclavian or internal jugular vein). In the surgical anatomy of the liver Incorrect segment I lies to the left of the portal vein TrueFalse Incorrect segment II lies medial to the porta hepatis TrueFalse Incorrect the caudate lobe lies anterior to the portal vein TrueFalse Incorrect the portal vein lies anterior to the Winslows foramen TrueFalse Incorrect three hepatic veins divide the liver into four sectors TrueFalse The caudate lobe (segment I according to Couinauds 1957 classification) lies posterior to the portal vein but anterior to the inferior vena cava. Three main hepatic veins divide the liver into four sectors, each of which receives a portal pedicle, with an alternation between hepatic veins and portal pedicles. According to this functional anatomy, the liver is divided into hemilivers (right and left) by the main portal scissura called Cantlies line. Nerves in direct contact with the humerus include: Incorrect median TrueFalse Incorrect radial TrueFalse Incorrect musculocutaneous TrueFalse Incorrect axillary TrueFalse Incorrect ulnar TrueFalse There are three nerves that come into close contact with the humerus: The radial nerve contacts the humerus in the spiral groove; the axillary at the surgical neck; and the ulnar at the medial epicondyle. Dupuytren's disease Incorrect

is caused by contraction of the palmar fascia TrueFalse Incorrect is more common in Caucasians than in people of African origin TrueFalse Incorrect most commonly affects the little finger TrueFalse Incorrect causes contracture of the intrinsic muscles of the hand TrueFalse Incorrect may be associated with retroperitoneal fibrosis TrueFalse Dupuytrens disease is a condition of unknown aetiology characterised by contraction of the palmar or digital fascia. It affects 13% of the population of North Europe and the USA. It is rare in the Far East and Africa. It is three times more common in males. Its incidence increases with age. It has a strong hereditary disposition. The ring finger is the most commonly affected finger; the little finger is the next most commonly affected digit. Dupuytrens disease causes contracture of the intrinsic muscles of the hand leading to flexion of the metacarpophalangeal joints and extension of the proximal interphalangeal joints the socalled intrinsic plus appearance. The following conditions are associated with Dupuytrens disease: knuckle pads (Garrods pads) penile fibrous plaques (Peyronies disease) plantar fibromatosis (Ledderhose disease). However, retroperitoneal fibrosis, however, is not associated with Dupuytrens disease. A CT scan section through the manubriosternal joint will demonstrate Incorrect the bifurcation of the brachiocephalic artery TrueFalse Incorrect the commencement of the aortic arch TrueFalse Incorrect T4 vertebral body TrueFalse Incorrect the bifurcation of the trachea TrueFalse Incorrect the thoracic duct crossing the midline TrueFalse A CT section at this level is at the level of T4. At this level, the arch of the aorta is commencing, the azygos vein enters the superior vena cava (SVC), the left recurrent laryngeal nerve loops round the ligamentum arteriosum and the bifurcation of the pulmonary trunk can be seen. The thoracic duct crosses the midline at T5. At the manubriosternal joint you have: - bifurcation of the trachea - start of the aortic arch - azygous vein entering the superior vena cava In the main bronchial airways Incorrect the left bronchus is longer than the right TrueFalse Incorrect the right main bronchus has a wider diameter than the left TrueFalse Incorrect aspiration pneumonitis is more common in the right lower lobe than the left TrueFalse

Incorrect the left main bronchus divides before entering the lung TrueFalse Incorrect foreign bodies lodge more commonly in the right than in the left main bronchus TrueFalse The right main bronchus is shorter (approximately 2.5 cm long), wider and runs more vertically than the left main bronchus. The right main bronchus gives off the upper lobe branch (before entering the lung) and passes inferior to the pulmonary artery before entering the hilum of the lung (approximately T5). It is important to remember the azygos vein, which arches over the right main bronchus from the posterior aspect as it passes to the SVC, and the pulmonary artery, which lies inferior and then anterior to it. The left main bronchus is about 5 cm long and, unlike the right, does not give off any branches before entering the hilum of the left lung at the level of T6. The abdominal inferior vena cava (IVC) Incorrect runs in the free edge of the lesser omentum TrueFalse Incorrect ascends to the right of the aorta TrueFalse Incorrect may be directly in contact with the right suprarenal gland TrueFalse Incorrect forms the posterior wall of the epiploic foramen TrueFalse Incorrect receives direct drainage from both the right and left suprarenal veins TrueFalse The IVC commences opposite the L5 vertebra. It runs on the right side of the aorta, upwards beyond the aortic opening of the diaphragm and extends to the central tendon of the diaphragm, which it pierces at the level of T8. The IVC lies behind the portal vein near the pancreas and bile duct, and forms the posterior wall of the epiploic foramen of Winslow. The right vein of the suprarenal gland is usually only a few millimetres long and enters the IVC directly. The left suprarenal vein is longer and enters the left renal vein. The spleen: Incorrect May be injured by a stab wound over the left tenth rib TrueFalse Incorrect Is a retroperitoneal structure TrueFalse Incorrect Venous drainage from the spleen passes into the portal system TrueFalse Incorrect The splenic artery passes to the splenic hilum in the gastrosplenic ligament TrueFalse Incorrect May be absent and replaced with multiple splenunculi TrueFalse The spleen lies in the concavity of the left hemidiaphragm with its long axis lying along the tenth rib. It is fully invested in peritoneum, making it a peritoneal structure. The arterial supply is the splenic artery, which reaches the spleen via the lienorenal ligament. Venous drainage is via the splenic vein into the portal vein. Occasionally the spleen may be replaced with multiple splenunculi. Trench foot Incorrect is caused by acute exposure to temperatures between 8C and 2C TrueFalse Incorrect is due to microvascular endothelial damage and vascular occlusion TrueFalse

Incorrect appears black even in the absence of deeper tissue destruction TrueFalse Incorrect is characterised by pruritic, redpurple lesions TrueFalse Incorrect can lead to gangrene TrueFalse Reply to HalaReport

Post #19 Hala Adel wrote5 hours ago Trench foot or cold immersion foot (or hand) is caused due to a non-freezing injury of the hands or feet. This is typically seen in soldiers, sailors, or fishermen, who are chronically exposed to wet conditions and temperatures just above freezing, i.e. 1.6C to 10C (35F to 50F). It occurs due to microvascular endothelial damage, stasis and vascular occlusion. Although the entire foot may appear black, deep tissue destruction may not be present. An alternating arterial vasospasm and vasodilatation occurs, with the affected tissue first cold and anaesthetic, progressing to hyperaemia in 24 to 48 h. This then leads to an intense painful burning and dysaesthesia, as well as tissue damage characterised by oedema, blistering, redness, ecchymosis, and ulcerations. Pruritic, redpurple skin lesions are a feature of chilblain or pernio. Complications of trench foot include local infection, cellulitis, lymphagitis, and gangrene. The following statements concern neuronal tracts in the spinal cord: Incorrect the fasciculus gracilis forms part of the dorsal column TrueFalse Incorrect the lateral corticospinal tracts control skilled voluntary movements TrueFalse Incorrect fine touch and proprioception are carried by crossed ascending neurones TrueFalse Incorrect the fasiculus cuneatus mainly carries sensory neurones from the lower limb TrueFalse Incorrect the lateral spinothalamic tracts conduct pain and temperature sensation TrueFalse The dorsal columns are white matter tracts formed by the fasciculus gracilis and fasciculus cuneatus. The fasciculus gracilis lies medial to the fasciculus cuneatus. They carry fine touch and proprioception. Uncrossed fibres are added from medial to lateral as the cord is ascended. Therefore the fasciculus gracilis mainly carries sensory fibres from the lower limbs, and the fasciculus cuneatus carries those from the upper limbs. The lateral spinothalamic tracts carry crossed axons conducting pain and temperature sensations. Consider the hindgut Incorrect The hindgut structures include the distal one-third of the transverse colon, the descending colon, sigmoid colon, rectum, and anal canal to the pectinate line TrueFalse Incorrect Branches of the inferior mesenteric artery supply the hindgut TrueFalse Incorrect The descending colon is secondarily retroperitoneal TrueFalse Incorrect The rectum contains taeniae coli, haustrations, and omental appendages TrueFalse Incorrect Failure of neural crest cells to migrate into the hindgut results in Hirschsprungs disease TrueFalse

The descending colon, the rectum, and the anal canal are secondarily retroperitoneal. The distal one-third of the transverse colon and the sigmoid colon are peritoneal. The distal one-third of transverse colon, descending and sigmoid colons contain taeniae coli, haustrations and omental appendages. The sigmoidrectal junction marks the end of the taeniae coli, the haustrations, and the omental appendices. Hirschsprungs disease is caused by a failure of neural crest cells either to migrate into the hindgut or to differentiate into terminal parasympathetic ganglia in the walls of the hindgut. The parotid duct Incorrect is approximately 1 cm long TrueFalse Incorrect crosses the masseter TrueFalse Incorrect is compressed by the buccinator TrueFalse Incorrect conveys mainly mucous secretions TrueFalse Incorrect lies on the middle third of a line between the intertragic notch of the auricle and the midpoint of the philtrum TrueFalse The parotid duct is approximately 5 cm long. It crosses the masseter, turning around its anterior border to pass through the buccal fat pad and pierce the buccinator. When intraoral pressure is raised, the submucous part of the parotid duct is compressed by the buccinator. The parotid gland is mainly a serous gland When the right main bronchus is dissected, the following structures may be encountered Incorrect the right phrenic nerve TrueFalse Incorrect the right vagus nerve TrueFalse Incorrect the right recurrent laryngeal nerve TrueFalse Incorrect the hemiazygos nerve TrueFalse Incorrect the azygos vein TrueFalse The vagus nerve lies just posterior to the right main bronchus and the azygos vein is at first posterior and then arches over the bronchus. The phrenic nerve is anterior to the bronchus. The right recurrent laryngeal nerve hooks around the right subclavian artery superior to the right main bronchus. The following structures are retroperitoneal: Incorrect Pancreas TrueFalse Incorrect Spleen TrueFalse Incorrect Liver TrueFalse Incorrect Abdominal aorta TrueFalse Incorrect Descending colon TrueFalse

Both liver and spleen are invested with peritoneum, whereas the other three structures are to be found attached to the posterior abdominal wall covered by the peritoneum. Note that bleeding from the liver and spleen will result in peritoneal blood, but pancreatic and aortic bleeding will cause a retroperitoneal haematoma often manifesting as flank discoloration. Transection of the anterior division of the mandibular nerve (CN Vc) in the infratemporal fossa results in Incorrect ipsilateral paralysis of the buccinator muscle TrueFalse Incorrect dysphagia TrueFalse Incorrect ipsilateral anaesthesia of the mandibular teeth TrueFalse Incorrect deviation of the jaw to the side of the lesion on protrusion TrueFalse Incorrect ipsilateral anaesthesia of the mucosa of the oral vestibule TrueFalse The anterior division of CN Vc has one sensory branch (the buccal nerve to the skin of the cheek and mucosa of the vestibule). All other branches are motor to the muscles of mastication (the masseter, temporalis and lateral pterygoid). The lower jaw teeth are supplied by the inferior alveolar branch of the posterior division of CN Vc. The buccinator muscle is supplied by the VIIth cranial nerve. Unopposed contraction of the contralateral lateral pterygoid muscle deviates the jaw to the side of the lesion during protrusion. None of the muscles of the pharynx are supplied by the anterior division of CN Vc, so dysphagia is not a feature of damage to this nerve. Umbilical hernia Incorrect is more common in Caucasians than in other races TrueFalse Incorrect is commonly associated with hypothyroidism TrueFalse Incorrect must be operated on before two years of age TrueFalse Incorrect becomes obstructed in 2% of cases TrueFalse Incorrect is more likely to resolve spontaneously if small TrueFalse Umbilical hernias are most frequently seen in low birthweight and black infants. Although it is a feature of hypothyroidism, most infants with umbilical hernias do not have hypothyroidism. The majority of hernias will disappear by one year of age, strangulation is rare, and operation is only indicated if the hernia persists until age 34 years, causes symptoms, or becomes progressively larger. Which of the following muscles are found in the medial fascial compartment of the thigh? Incorrect Adductor brevis TrueFalse Incorrect Sartorius TrueFalse Incorrect Obturator externus TrueFalse Incorrect Gracilis TrueFalse Incorrect Hamstring portion of adductor magnus TrueFalse

The medial fascial compartment of the thigh contains adductor longus, adductor brevis, adductor portion of adductor magnus, gracilis and obturator externus. Sartorius lies in the anterior fascial compartment and the hamstring portion of adductor magnus lies in the posterior fascial compartment of the thigh. A Kocher's incision Incorrect divides the Colles fascia TrueFalse Incorrect divides only the anterior rectus sheath TrueFalse Incorrect divides the external oblique muscle TrueFalse Incorrect involves the area innervated by T10 nerve root TrueFalse Incorrect divides the fascia transversalis muscle TrueFalse Incorrect divides the rectus abdominis muscle TrueFalse Scarpas fascia is divided in a Kochers incision. T10 corresponds to the umbilical area. The rectus sheath is deficient posteriorly only below the arcuate line, but is present in two layers in the subcostal region. Concerning the female breast: Incorrect the breast extends from the 2nd to the 5th rib TrueFalse Incorrect the breast extends from the lateral margin of the sternum to the mid-axillary line TrueFalse Incorrect the greater part of the gland lies in the deep fascia TrueFalse Incorrect the mammary gland is attached to the skin by Coopers ligaments TrueFalse Incorrect the mammary gland consists of 1520 lobes TrueFalse The breast extends from the 2nd to the 6th rib, and from the lateral margin of the sternum to the mid-axillary line. The greater part of the gland lies in the superficial fascia, and the gland is attached to the skin by suspensory ligaments (Coopers ligaments). The mammary gland consists of 1520 lobes, which radiate out from the nipple. Direct branches of the coeliac plexus include the Incorrect splenic artery TrueFalse Incorrect common hepatic artery TrueFalse Incorrect superior pancreaticoduodenal artery TrueFalse Incorrect right gastric artery TrueFalse Incorrect gastroduodenal artery TrueFalse As soon as the aorta passes below the aortic hiatus, it gives off the celiac plexus (T12). The

plexus has three direct branches: left gastric, hepatic and splenic (mnemonic: left-hand side). The right gastric and gastroduodenal arteries are branches of the hepatic artery. The superior pancreaticoduodenal artery is a branch of the gastroduodenal artery. The right renal artery Incorrect branches several times before entering the kidney TrueFalse Incorrect gives a branch to the ureter TrueFalse Incorrect lies anterior to the renal vein TrueFalse Incorrect lies anterior to the inferior vena cava TrueFalse Each renal artery usually divides into five segmental branches before entering the renal pelvis. It supplies the ureter and lies posterior to the renal vein, but anterior to the renal pelvis. After branching off the aorta, the renal artery passes behind the inferior vena cava. At the level of the T4 vertebra, a computed tomography scan of the body transversely shows the Incorrect arch of aorta TrueFalse Incorrect bifurcation of trachea TrueFalse Incorrect left brachiocephalic vein TrueFalse Incorrect azygos vein TrueFalse In the plane of Louis, the azygos vein arches over the hilum of the right lung before draining into the superior vena cava. Regarding fractures of the axis (C2) vertebra, which of the following are true? Incorrect Approximately 60% of all axis fractures involve the odontoid process TrueFalse Incorrect Type I odontoid fractures occur through the base of the dens TrueFalse Incorrect Type III odontoid fractures are the commonest type TrueFalse Incorrect Surgical fixation is indicted for type II odontoid fractures TrueFalse Incorrect Posterior element fractures are due to an extension type of injury TrueFalse Acute fractures of the axis (C2) vertebra represent about 18% of all cervico-spinal injuries and approximately 60% of axis fractures involve the odontoid process. The odontoid process is a peg-shaped bony protuberance that projects upward and is normally positioned in contact with the anterior arch of C1. It is held in place primarily by the transverse ligament. Type I odontoid fractures involve the tip of the odontoid peg, type II fractures are through the base of the dens (involving the junction of the odontoid peg with the body) and type III fractures occur at the base of the dens and extend obliquely into the body of the axis. Odontoid fractures are initially identified by a lateral cervico-spinal film or open-mouth odontoid views. In many cases, however, a computed tomography (CT) scan is required to further delineate the type and extent of the fracture. In children younger than 6 years of age, on plain radiography, the epiphysis may be prominent and may look like a fracture at this level. Type II is the commonest type of odontoid fractures. They require surgical reduction and immobilisation with a Halo and body cast. If the fracture is not healed (and so unstable) at 12 weeks, posterior fusion of C1 to C2 may be

indicated. The posterior elements of C2, ie, the pars interarticularis may be fractured (a hangmans fracture) by an extension type of injury. Patients with this fracture should be maintained in external immobilisation until specialised care is available. These fractures represent approximately 20% of all axis fractures. For acute tongue swelling, which one of the following is true Incorrect Occurs as a side-effect of angiotensin-converting enzyme (ACE) inhibitors TrueFalse Incorrect Initial treatment is with beta-blockers TrueFalse Incorrect Airway obstruction is not a feature TrueFalse Incorrect Steroids are contraindicated TrueFalse Incorrect Antihistamines usually take 6 hours for full effect TrueFalse Angio-neurotic oedema is tongue swelling secondary to ACE inhibitors. The most important initial management is to secure the airway; a nasotracheal tube may be required. Steroids (which may take 6 hours to take full effect) and antihistamine should be commenced immediately. Crohn's disease Incorrect is typically a submucosal inflammation TrueFalse Incorrect is associated with rosethorn ulcers on barium studies TrueFalse Incorrect is associated with mouth ulcers TrueFalse Incorrect may lead to a patient requiring lifelong parenteral nutrition TrueFalse Incorrect most commonly affects the distal colon and then spreads proximally TrueFalse Inflammation is classically described as transmural. Rosethorn ulcers are deep ulcers that traverse beyond the lamina propria and have a characteristic appearance. Apthous ulcers occur anywhere from the mouth to the anus. Short-bowel syndrome is why we try to avoid surgery whenever possible with Crohns disease, but in very severe cases where less than 50 cm of small bowel remains, malabsorption of essential fat soluble vitamins (A, D, E and K) as well as other essential nutrients requires lifelong parenteral nutrition. Crohns colitis is not uncommon but the small bowel is more often involved and usually seen by the time a colitis occurs. Crohns spreads as skip lesions and, unlike ulcerative colitis, the spread is not usually in a continuous fashion. Barretts oesophagus Incorrect occurs in approximately 40% of patients with gastro-oesophageal reflux disease (GORD) TrueFalse Incorrect never produces ulceration TrueFalse Incorrect presents with malignancy in one-third of cases TrueFalse Incorrect produces strictures at the squamocolumnar junction TrueFalse Incorrect histamine H2 blockers are the treatment of choice TrueFalse

10% of patients with gastro-oesophageal reflux disease (GORD) will develop Barretts oesophagus. It may ulcerate if left untreated, but medical treatment should be with proton pump inhibitors. Strictures are common and may lead to the symptoms of dysphagia with which the patient presents. Which of the following muscles abduct the thigh? Incorrect Quadratus femoris TrueFalse Incorrect Gluteus maximus TrueFalse Incorrect Gluteus medius TrueFalse Incorrect Gluteus minimus TrueFalse Incorrect Piriformis TrueFalse Both gluteus medius and gluteus minimus abduct and medially rotate the thigh at the hip joint. Gluteus maximus extends and laterally rotates the thigh at the hip joint. Quadratus femoris and piriformis both contribute to lateral rotation of the thigh. Which of the following are true? Regarding the muscles of respiration:

Incorrect Quiet inspiration is predominantly due to the action of the diaphragm TrueFalse Incorrect Active inspiration is caused by the internal intercostal muscles TrueFalse Incorrect The scalene muscles play an important role in quiet expiration TrueFalse Incorrect Internal and external oblique muscles are important in active expiration TrueFalse Incorrect The external intercostal muscles pull the ribs medially and inferiorly during active expiration TrueFalse Reply to HalaReport

Post #20 Hala Adel wrote5 hours ago The predominant muscle of inspiration during quiet breathing is the diaphragm, a dome-shaped musculofibrous septum separating the thorax from the abdominal cavity. As the diaphragm contracts, pleural pressure drops, which lowers the alveolar pressure. This draws air into the lungs due to the pressure gradient from the mouth to the alveoli. Expiration during quiet breathing is a passive process. This is caused by the relaxation of the respiratory muscles and the return of the elastic lung and chest wall to their normal resting volume. During exertion or activity, the external intercostals help in inspiration by raising the lower ribs superiorly and

laterally, so increasing the lateral and antero-posterior diameters of the thoracic cavity. The scalene muscles and sternocleidomastoids also help by raising and pushing out the upper ribs and the sternum. Active expiration is helped by the contraction of the abdominal wall muscles (internal oblique, external oblique, transversus abdominus and rectus abdominus). It increases the intra-abdominal pressure, which pushes up the diaphragm, so raising the pleural pressure and drives the air out of the lungs. The internal intercostals also help in active expiration by decreasing the thoracic volume (by pulling down medially and inferiorly). The portal vein Incorrect is formed behind the body of the pancreas TrueFalse Incorrect lies anteriorly to the free edge of the lesser omentum TrueFalse Incorrect drains the spleen TrueFalse Incorrect forms the central vein of each liver lobule TrueFalse Incorrect lies to the right of the superior mesenteric artery TrueFalse Incorrect is about 10 cm in length TrueFalse The portal vein is formed by the confluence of the superior mesenteric vein (lying to the right of the artery) and the splenic vein, behind the neck of the pancreas. It is about 5 cm long. The portal vein lies between the two layers of the lesser omentum, running almost vertically in the free edge where the lesser omentum forms the anterior boundary of the epiploic foramen. The terminals of the portal vein and the hepatic artery form, with the hepatic ductules, the triads of the liver in the corners of the hexagonal lobules. The central veins drain into the hepatic veins. The following could be appropriate management for a gunshot injury to the upper part of the neck (above the angle of the mandible) Incorrect Arteriogram TrueFalse Incorrect Endoscopy TrueFalse Incorrect Barium swallow TrueFalse Incorrect Clinical observation TrueFalse Incorrect Surgical exploration TrueFalse It is not appropriate to observe gunshot injuries: they need to be explored. An arteriogram may be of benefit: vascular injury is the greatest concern here, and it may be possible to embolise a bleeding vessel. This area is above the level of the trachea and oesophagus. Colorectal carcinoma Incorrect is associated with a low-fibre, high-fat diet TrueFalse Incorrect presents with rectal mass in approx 45% of cases TrueFalse Incorrect is found in the rectum in 25% of cases TrueFalse Incorrect is commonly associated with Krukenbergs tumours TrueFalse

Incorrect is treated by abdominal perineal resection as the surgical procedure of choice for tumours about 15 cm from the anal canal TrueFalse Approximately 45% of colorectal tumours are found in the rectum. Krukenbergs tumours are more commonly secondaries from gastric and ovarian cancer though can arise from colorectal tumours. Abdominal perineal resections are used for low rectal tumours, where tumours are about 8 cm from the anal canal. In oesophageal disorders Incorrect patients with oesophageal achalasia are usually younger than those presenting with oesophageal carcinoma TrueFalse Incorrect in oesophageal carcinoma, dysphagia is progressive unlike achalasia TrueFalse Incorrect oesophageal achalasia is treatable by dilating the lower oesophagus TrueFalse Incorrect oesophagitis due to Herpes simplex occurs in immuno-compromised patients TrueFalse Incorrect medical treatment of gastro-oesophageal reflux is successful in relieving regurgitation or vomiting TrueFalse In achalasia, patients have equal difficulty in swallowing both liquids and solids. In carcinoma, difficulties begin with swallowing solids and progress to include liquids. In 95% of cases, only surgery, if properly done, has any effect on curing regurgitation. The recurrent laryngeal nerve Incorrect supplies the cricothyroid muscle TrueFalse Incorrect partially supplies the trachea TrueFalse Incorrect lies alongside the inferior thyroid artery TrueFalse Incorrect should be retracted during tracheostomy to avoid damage TrueFalse Incorrect runs between the oesophagus and trachea in the neck TrueFalse Incorrect supplies the mucous surface of the vocal cords TrueFalse The recurrent laryngeal nerve supplies all the intrinsic muscles of the larynx except the cricothyroid and is sensory inferior to the vocal folds. In the neck the recurrent laryngeal nerves on both sides follow the same course, ascending in the tracheo-oesophageal groove. As the nerve passes the lateral lobe of the thyroid it is closely related to the inferior thyroid artery. The superior laryngeal nerve supplies the vocal cord mucosa. Which of the following are correct? Underwater - sealed chest drains Incorrect Avoid build-up of positive intrathoracic pressure in case of alveolar air leak TrueFalse Incorrect A 28 Fr drain is usually used to drain blood or pus TrueFalse Incorrect Does not get blocked TrueFalse Incorrect

Reduces the risk of positive-pressure ventilation TrueFalse Incorrect Accurately measures blood loss TrueFalse A chest drain is a conduit to remove air or fluid from the pleural cavity allowing re-expansion of the underlying lung. Drainage occurs during expiration when pleural pressure is positive. Unless it is an emergency situation, preprocedure chest X -ray should be performed. The drain is usually inserted under local anaesthesia using aseptic technique into the 5th intercostal space in the mid-axillary line, and over the upper border of the rib, to avoid intercostal vessels and nerves. A large drain (28 Fr or above) should be used to drain blood or pus. Benign prostatic hyperplasia (BPH) Incorrect mainly affects the peripheral zone TrueFalse Incorrect is a recognised cause of elevated serum prostate-specific antigen (PSA) TrueFalse Incorrect incidence is increased in males castrated before puberty TrueFalse Incorrect symptoms improve with oxybutynin TrueFalse Incorrect can be treated with 5-alpha reductase inhibitors TrueFalse Benign prostatic hyperplasia (BPH) mainly affects the inner transitional zone. The outer peripheral zone is usuallycompressed and feels smooth to digital rectal examination. Any palpable nodule or irregularity should raise the possibility of malignancy. BPH seems to be an androgen-driven disease. Castration prior to puberty seems to prevent the disease. Alpha blockers cause relaxation of smooth muscles and improve symptoms, whereas anticholinergic drugs could worsen symptoms and precipitate acute urinary retention. What are the differences between the right and left lungs Incorrect The right lung has three lobes TrueFalse Incorrect The right lung is shorter than the left TrueFalse Incorrect The right lung is larger and heavier TrueFalse Incorrect The anterior margin of the right lung is straight, unlike that of the left lung TrueFalse The right lung has three lobes, the left lung has two. The right lung is larger and heavier than the left but it is also shorter and wider because the right dome of the diaphragm is higher and the heart bulge more to the left. The anterior margin of the left lobe has the cardiac notch. The following are recognised complications of a rolling hiatus hernia: Incorrect oesophagitis TrueFalse Incorrect gastric volvulus TrueFalse Incorrect inhalational pneumonia TrueFalse Incorrect intermittent dysphagia TrueFalse Incorrect gangrene TrueFalse

The majority of hiatus herniae are sliding or axial in nature, these are often asymptomatic but are associated with oesophagitis, stricture formation, dysphagia, chronic anaemia and inhalational pneumonitis. Rolling herniae or para-oesophageal hiatal herniae usually affect elderly patients who present with intermittent dysphagia, pain after eating due to distension of the intrathoracic part of the stomach, cardiac symptoms due to pressure effects on the heart, and hiccough due to phrenic nerve irritation. Complications include incarceration, gangrene and gastric volvulus. Posterior hip dislocation Incorrect classically occurs when the hip is in the extended position TrueFalse Incorrect is a common injury TrueFalse Incorrect can occur with avascular necrosis TrueFalse Incorrect may include damage to the femoral nerve TrueFalse Incorrect is characterised by the leg being held flexed and medially rotated TrueFalse Posterior hip dislocation is an uncommon injury often occurring when the hip is flexed e.g. a road traffic accident. The two main complications are sciatic nerve damage and avascular necrosis. Sciatic nerve damage occurs because the sciatic nerve lies in close proximity to the posterior aspect of the joint capsule so is at risk in posterior dislocation. Avascular necrosis occurs due to tearing of the joint capsule, causing a disturbance of the blood supply to the femoral head. The trigeminal nerve Incorrect supplies the buccinator muscle TrueFalse Incorrect supplies the muscles of mastication TrueFalse Incorrect has ophthalmic and maxillary divisions, which are only sensory TrueFalse Incorrect is sensory to the temporomandibular joint TrueFalse Incorrect supplies sensation to the angle of the mandible TrueFalse The trigeminal (V) nerve has sensory fibres to the greater part of the skin of the face, mucous membranes of the mouth, nose and paranasal air sinuses. It provides motor innervation to the muscles of mastication (temporalis, masseter, pterygoid). The buccinator muscle is supplied by the facial nerve. The angle of the mandible is supplied by the great auricular nerve (C2C3). Reply to HalaReport

Post #21 Hala Adel wrote5 hours ago Regarding spinal cord syndromes, which of the following are true? Incorrect Central cord syndrome results from vascular compromise of the cord along the distribution of the anterior spinal artery TrueFalse Incorrect Position and vibration sense are preserved in anterior cord syndrome TrueFalse Incorrect

There is ipsilateral motor loss and contralateral loss of pain and temperature sensation in BrownSequard syndrome TrueFalse Incorrect The arms are more affected than the legs in central cord syndrome TrueFalse Incorrect Anterior cord syndrome has the best prognosis among all incomplete spinal injuries TrueFalse The central cord syndrome is thought to be due to vascular compromise of the cord in the distribution of the anterior spinal artery. Infarction of the cord in the territory of this artery could also result in the anterior cord syndrome. Anterior cord syndrome is characterised by paraplegia and a dissociated sensory loss with loss of pain and temperature sensation. Position, vibration and deep pressure sensations, all functions of the posterior column, are preserved. Brown Sequard syndrome, resulting from hemisection of the cord, usually causes ipsilateral motor loss and contralateral loss of pain and temperature sensation; there is also associated loss of position sense. The central cord syndrome is characterised by a disproportionately greater loss of motor power in the upper extremities than in the lower extremities, with varying degrees of sensory loss. The arms and hands are most severely affected since the motor fibres to the cervical segments are topographically arranged toward the centre of the cord. Anterior cord syndrome has the poorest prognosis of all incomplete spinal injuries. Which of the following are posterior relations of the kidneys: Incorrect Psoas major TrueFalse Incorrect Subcostal nerve TrueFalse Incorrect Peritoneum TrueFalse Incorrect Diaphragm TrueFalse Incorrect Pleura TrueFalse The kidneys are retroperitoneal therefore the peritoneum is an anterior relation. Posterior relations of the kidneys include the diaphragm, quadratus lumborum, psoas major, subcostal vein, subcostal artery, subcostal nerve and ilioinguinal nerve. The costodiaphragmatic recess of the pleura is an important posterior relation of the kidney as it can be inadvertently damaged during nephrectomy resulting in a pneumothorax. The femoral canal Incorrect allows for expansion of the femoral vessels TrueFalse Incorrect is a clinically important site of herniation of the small bowel TrueFalse Incorrect contains Cloquets node TrueFalse Incorrect contains the femoral artery TrueFalse Incorrect has the femoral vein lying medially TrueFalse The femoral sheath is a fascial tube derived from extraperitoneal intra-abdominal fascia. It extends under the inguinal ligament to surround the femoral vessels. The canal is a small space between the medial part of the sheath and the femoral vein. It contains fat and Cloquets node. Femoral hernias can be differentiated from inguinal hernias by locating the neck of a femoral hernia below and lateral to the inguinal canal. Compared with the lower end of the ileum, the upper end of the jejunum has Incorrect

a thicker wall TrueFalse Incorrect less fat at the mesenteric border TrueFalse Incorrect fewer circular folds TrueFalse Incorrect a wider lumen TrueFalse Incorrect more aggregated lymphatic follicles (Peyers patches) TrueFalse Incorrect more arterial arcades TrueFalse The jejunum has a thicker wall, less mesenteric fat, more plicae circulares, a wider lumen, fewer Peyers patches and fewer arterial arcades than the ileum. The 1st rib Incorrect has scalenus anterior muscle inserted onto the scalene tubercle TrueFalse Incorrect has the subclavian vein overlying the vertebral transverse processes TrueFalse Incorrect has the subclavian vein running lateral to the artery TrueFalse Incorrect is related to the pleura TrueFalse Incorrect is related to the cervicothoracic (stellate) sympathetic ganglion TrueFalse Incorrect is related to the upper two roots of the brachial plexus TrueFalse The 1st rib has the scalenus anterior muscle attached to the scalene tubercle, separating the subclavian vein (anteriorly) from the artery (posteriorly). The right suprarenal gland Incorrect lies against the bare area of the liver TrueFalse Incorrect extends behind the inferior vena cava (IVC) TrueFalse Incorrect receives blood from the right inferior phrenic artery TrueFalse Incorrect drains into the right renal vein TrueFalse Incorrect lies on the ninth rib TrueFalse The bare area of the liver is in direct contact with the right suprarenal gland and the diaphragm. The right suprarenal gland extends medially behind the IVC, separated from the 12th rib by the diaphragm. It typically has three arterial sources. It receives blood from the inferior phrenic artery, from a branch of the renal artery and from a branch directly from the aorta. The venous drainage is into the IVC by a very short vessel. The left suprarenal gland drains into its corresponding renal vein. Midline swellings of the neck include Incorrect

cystic hygromas TrueFalse Incorrect plunging ranulae TrueFalse Incorrect subhyoid bursae TrueFalse Incorrect branchial cysts TrueFalse Incorrect arteriovenous fistulae TrueFalse Swellings of the neck considered to be midline include thyroglossal cysts, pharyngeal pouches, plunging ranulae, subhyoid bursae, laryngoceles and lesions in the thyroid isthmus. Stimulation of the parasympathetic nervous system leads to Incorrect pupillary constriction TrueFalse Incorrect increased heart rate TrueFalse Incorrect smooth muscle relaxation TrueFalse Incorrect increased glandular secretion TrueFalse Incorrect decreased force of contraction of the heart TrueFals Reply to HalaReport

Post #22 Hala Adel wrote5 hours ago MRCS Part 1 Practice Questions ( Anatomy ) - 2 of 2 Here's the second part, Good Luck! Intervertebral disc collapse between L5 and S1 Incorrect would crush the L5 spinal nerve TrueFalse Incorrect would impinge into the sacral segments of the cord TrueFalse Incorrect usually causes pain to radiate over the medial malleolus TrueFalse Incorrect would exaggerate the tendon reflex at the ankle TrueFalse Incorrect may cause reduced sweating over the posterior aspect of the calf TrueFalse A collapsed L5S1 disc presses on the S1 spinal nerve (the L5 nerve passes above the prolapsed disc in the intervertebral foramen and so escapes damage). At the level of prolapse, the spinal canal contains the cauda equina and not cord per se. The S1 dermatome lies over the lateral malleolus. Exaggerated reflexes are diagnostic of an upper motor neurone lesion. The S2 dermatome occupies the posterior aspect of the calf. The anal canal Incorrect lies below the levator ani muscle TrueFalse

Incorrect has a longitudinal muscular coat TrueFalse Incorrect has a lymphatic drainage via the inguinal lymph nodes TrueFalse Incorrect has an external sphincter innervated by the pudendal nerve TrueFalse Incorrect possesses valves TrueFalse The levator ani forms part of the deep external anal sphincter. The anal canal has no longitudinal muscular coat. Lymph from the lower anal canal drains via the superficial inguinal nodes. The entire anal sphincter is innervated by the inferior rectal branch of the pudendal nerve (S2S4). The upper anal canal is thrown into vertical folds called anal columns. The anal valves are formed by horizontal semilunar folds of mucous membrane joining adjacent columns at their lower end. Anal valves are remnants of the proctodeal membrane. The anococcygeal body lies between the anal canal and the coccyx. The left brachiocephalic vein drains the Incorrect cervical vertebrae TrueFalse Incorrect bronchi TrueFalse Incorrect intercostal spaces TrueFalse Incorrect thoracic duct TrueFalse Incorrect thyroid gland TrueFalse The left brachiocephalic vein drains blood from: the cervical vertebrae via both vertebral veins; the thyroid gland by the inferior thyroid veins; the first left intercostal space via the left superior intercostal veins; and all the anterior intercostal spaces by the anterior intercostal veins draining into the internal thoracic veins. The thoracic duct enters the vein at its commencement behind the left sternoclavicular joint. The bronchial veins drain into the azygos/hemiazygos systems. Fracture of the following bones is rare and indicates high energy trauma which could be associated with hidden severe injuries Incorrect Scapula TrueFalse Incorrect Clavicle TrueFalse Incorrect Sternum TrueFalse Incorrect 10th rib TrueFalse Incorrect 1st rib TrueFalse Incorrect Shaft of humerus TrueFalse These bones are difficult to break, the finding of such an injury is worrisome, and a further pathology must be sought. The tongue

Incorrect receives sensory innervation from the vagus nerve TrueFalse Incorrect protrudes to the side of a unilateral lower motor neurone lesion TrueFalse Incorrect is active during the voluntary phase of swallowing TrueFalse Incorrect is retracted by the hyoglossus muscle TrueFalse Incorrect contains lymphoid tissue TrueFalse Incorrect has intrinsic muscles that are not attached to any bone TrueFalse The sensory innervation to the tongue is from the VIIth, VIIIth and IXth cranial nerves. The tongue deviates to the side of a XIIth cranial nerve lesion on protrusion, is active during the first stage of swallowing and contains the lingual tonsil in the dorsum of its posterior third. The tongue is retracted up and back by the styloglossus muscle, protruded by genioglossus and depressed by the hyoglossus. The phrenic nerves Incorrect are sensory to the peritoneum TrueFalse Incorrect transmit afferent fibres from the mediastinal pleura TrueFalse Incorrect receive sensory fibres from the lungs TrueFalse Incorrect supply the bronchi TrueFalse Incorrect pass anterior to scalenus anterior muscles TrueFalse The phrenic nerve arises from the spinal cord segments C35 and lie in front of the scalenus anterior muscle, passing between the subclavian vein anteriorly and the subclavian artery posteriorly. It crosses over the lateral surfaces of the mediastinal structures passing in front of the lung root to reach the diaphragm. The phrenic nerve supplies motor fibres to the diaphragm and carries sensory fibres from the diaphragmatic peritoneum, mediastinal pleura and the parietal pericardium. The bronchi and lungs themselves are supplied by branches of the autonomic nerves, principally via the pulmonary plexuses. Reply to HalaReport

Post #23 Hala Adel wrote5 hours ago Lymphatic ducts Incorrect contract due to filling TrueFalse Incorrect have no valves TrueFalse Incorrect if obstructed, lead to lymphoedema TrueFalse Incorrect have a parasympathetic innervation TrueFalse Incorrect

empty by pump action of the calf muscles TrueFalse Incorrect dilate in oedema TrueFalse The function of lymphatic vessels is to return the plasma capillary filtrate to the circulation. This task is achieved by increased tissue pressure, facilitated by intermittent skeletal muscle activity, contractions of lymphatic vessels and an extensive system of one-way valves. Lymphoedema is an accumulation of tissue fluid resulting from a fault in the lymphatic system very often, patients are diagnosed as having lymphoedema when the oedema is due to another cause. Lymphoedema can occur as a result of lymphatic obstruction secondary to infiltration of lymph nodes, frequently deep in the pelvis. Transection of the cervical part of the sympathetic chain at the root of the neck results in Incorrect vasomotor changes in the arm TrueFalse Incorrect ptosis TrueFalse Incorrect pupillary dilatation TrueFalse Incorrect ablation of sympathetic supply to the pulmonary plexus TrueFalse Incorrect loss of sweating over the C4 dermatome TrueFalse In Horners syndrome there is: ptosis, pupillary constriction and occasional enophthalmos, and dryness and flushing of the skin of the head and neck. The sympathetic supply to the lungs is preserved as this originates below the lesion directly from the T1T4 ganglia of the sympathetic chain. Sympathetic fibres pass to the arm via grey rami from the middle and inferior cervical sympathetic ganglia through all the roots of the brachial plexus. The musculocutaneous nerve Incorrect supplies skin over the shoulder TrueFalse Incorrect supplies the biceps brachii TrueFalse Incorrect becomes the lateral cutaneous nerve of the forearm (lateral antebrachial cutaneous nerve) TrueFalse Incorrect supplies the muscles of the anterior aspect of the arm TrueFalse Incorrect supplies skin on the medial aspect of the arm TrueFalse

The musculocutaneous nerve supplies the coracobrachialis, biceps brachii and brachialis muscles. It pierces the deep fascia just proximal to the elbow joint and becomes superficial. It is then called the lateral cutaneous nerve of the forearm, supplying skin on the lateral aspect of the arm. Abdominal aortic aneurysms (AAAs) Incorrect are due to diabetes mellitus in most cases TrueFalse Incorrect expand at 10 mm per year TrueFalse

Incorrect are inflammatory in 7% of cases TrueFalse Incorrect measuring 7 cm in diameter have a 5 year rupture rate of 40% TrueFalse Abdominal aortic aneurysms (AAAs) are caused by atherosclerosis in most cases. They expand at approximately 4 mm per year. The 5-year rupture rate for aneurysms measuring 7 cm is approximately 75%. Inspiration involves Incorrect descent of the hemidiaphragms TrueFalse Incorrect reduction of the vertical dimension of the chest TrueFalse Incorrect upward/forward movement of the first rib TrueFalse Incorrect contraction of the intercostal muscles TrueFalse Incorrect the long thoracic nerve of Bell (supplying the serratus anterior) TrueFalse The vertical dimension of the chest increases on inspiration. The ribs move upwards and outwards. However, the first rib does not move during respiration. The serratus anterior (supplied by the long thoracic nerve) is involved in respiration. The following statements concern the ankle joint Incorrect The deltoid ligament is attached to the lateral malleolus TrueFalse Incorrect The body and articular surface of the talus is wider anteriorly than posteriorly TrueFalse Incorrect The ankle is most stable in plantarflexion TrueFalse Incorrect The capsular ligaments in front of and behind the ankle joint are weak TrueFalse Incorrect The intermalleolar distance increases as the ankle plantarflexes TrueFalse The ankle (tibiotalar) joint is a hinge joint. As with most hinge joints there is strong support at the sides but not in front and behind. The deltoid ligament is attached above to the medial malleolus and fans out to attach below, mainly on the talus, but also on the calcaneus. On the lateral side there are three smaller ligaments (anterior and posterior talofibular ligaments and calcaneofibular ligament). The ankle joint is most stable in dorsiflexion. The intermalleolar distance increases in dorsiflexion due to the increased width of the anterior part of the talus bone. Lower limb amputation Incorrect through the knee affords the best rehabilitation TrueFalse Incorrect above the knee usually heals when equal anterior and posterior flaps are used TrueFalse Incorrect below the knee is classically performed with equal flaps TrueFalse Incorrect may be performed using a skew flap technique below the knee TrueFalse

Incorrect using Syme's technique is the operation of choice in patients with peripheral vascular disease TrueFalse The best amputation which affords good rehabilitation is a below the knee procedure preserving the joint. Classically, for a below knee amputation (BKA), a long posterior flap is fashioned containing muscle and vessels, which is then folded over the base to form an even stump. More recently, skew flaps have been introduced to make use of areas of tissue where the blood supply is optimal. Above knee procedures usually heal when equal anterior and posterior flaps are used. Generally, Syme's amputation should not be used in patients with peripheral vascular disease, and one-third are subsequently revised to a higher level because of poor healing, ulceration or poor function. The processus vaginalis Incorrect is formed by visceral peritoneum TrueFalse Incorrect forms a sac in which the testis descends through the inguinal canal TrueFalse Incorrect when present in adults, predisposes to direct inguinal hernia TrueFalse Incorrect forms the tunica vaginalis in the adult TrueFalse Incorrect invests the adult vas deferens TrueFalse The processus vaginalis is a parietal peritoneal sac which passes through the internal ring of the inguinal canal in the fetus, but which is normally obliterated after birth except for a small part that becomes the tunica vaginalis of the testis. The testis descends through the canal as a retroperitoneal structure and is therefore outside and behind the processus vaginalis. In cases of a persistent processus vaginalis, indirect inguinal hernias can ensue. Carpal tunnel syndrome is associated with Incorrect Pregnancy TrueFalse Incorrect Diabetes mellitus TrueFalse Incorrect Bennetts fracture TrueFalse Incorrect Hypothyroidism TrueFalse Incorrect Golfers elbow TrueFalse

Reply to HalaReport

Post #24 Hala Adel wrote5 hours ago Carpal tunnel syndrome is associated with rheumatoid arthritis, myxoedema, nephrotic syndrome, acromegaly, multiple myeloma, amyloidosis, diabetes mellitus, alcoholism, haemophilia, pregnancy, gout, wrist fractures and the menopause. A Bennetts fracture is a fracture of the first metacarpal and therefore does not affect the wrist. The nerve roots Incorrect of the ulnar nerve are C8, T1, and sometimes C7 TrueFalse

Incorrect of the musculocutaneous nerve are C5C6 TrueFalse Incorrect of the axillary nerve are C5C8 TrueFalse Incorrect of the radial nerve are C5C8, and T1 TrueFalse Incorrect of the long thoracic nerve are C5C7 TrueFalse The nerve roots of the musculocutaneous nerve are C5C7, and those of the axillary nerve are C5 and C6. The nerve roots of the median nerve are C5C8, and T1. In the base of the skull the: Incorrect foramen magnum transmits the basilar artery TrueFalse Incorrect foramen spinosum transmits the VIIth cranial nerve (CN VII) TrueFalse Incorrect foramen rotundum transmits the maxillary nerve TrueFalse Incorrect foramen ovale transmits the greater petrosal nerve TrueFalse Incorrect foramen lacerum transmits the mandibular nerve TrueFalse The foramen magnum transmits the vertebral arteries which unite at the lower border of the pons to form the basilar artery. The foramen spinosum transmits the middle meningeal vessels and the meningeal branch of the mandibular nerve. The foramen rotundum contains the maxillary nerve. The foramen ovale transmits the mandibular nerve, lesser petrosal nerve and accessory meningeal artery. The foramen lacerum transmits the internal carotid and greater petrosal nerve, which leaves as a nerve of the pterygoid canal. In surgical anatomy of the thyroid gland Incorrect the thyroid gland has a definite, fine capsule TrueFalse Incorrect Berrys ligament connects the thyroid to the cricoid cartilage and upper trachea TrueFalse Incorrect the inferior parathyroid glands are more constant in position than the superior parathyroid glands TrueFalse Incorrect the middle thyroid veins are more constant in position than the superior and inferior thyroid veins TrueFalse Incorrect unilateral recurrent laryngeal nerve division results in the contralateral vocal cord lying in the mid- or cadaveric position TrueFalse The thyroid gland has a definite, fine capsule, which allows a capsular dissection to preserve the recurrent laryngeal nerves. The superior parathyroid glands are more constant in position than the inferior. Because of their embryological migration, the inferior glands may be situated among the pretracheal lymph nodes or in the thymus as far as 10 cm from the thyroid. The middle thyroid veins are the least constant of the thyroid veins. The superior veins drain into the internal jugular vein; the inferior veins are very constant and drain into the brachiocephalic veins; and the middle veins are very variable and often multiple. Unilateral recurrent laryngeal nerve section results in the ipsilateral vocal cord lying motionless in the mid- or cadaveric

position. The voice is hoarse and weak. If both recurrent laryngeal nerves are divided, then the glottic space is narrowed and stridor develops. Recognised complications of endoscopic sphincterotomy during ERCP include Incorrect Acute pancreatitis TrueFalse Incorrect Gastrointestinal haemorrhage TrueFalse Incorrect Small bowel obstruction TrueFalse Incorrect Small bowel perforation TrueFalse Incorrect Enterocutaneous fistula TrueFalse Division of the sphincter of Oddi with the sphincterotome may cause pancreatitis, duodenal perforation or bleeding. Many patients have a transiently increased serum amylase but a few develop true acute pancreatitis with pain and ultrasound evidence of pancreatitis. The femoral triangle Incorrect contains the deep inguinal lymph nodes TrueFalse Incorrect is bounded by the inguinal ligament inferiorly TrueFalse Incorrect is bounded by sartorius laterally TrueFalse Incorrect has a floor formed by the fascia lata TrueFalse Incorrect contains the femoral nerve, artery and vein TrueFalse The femoral triangle is bounded by the inguinal ligament superiorly, sartorius laterally and adductor longus medially. Its floor is formed by the iliopsoas and pectineus. Its roof is formed by the fascia lata. It contains the femoral vein, artery and nerve from medial to lateral and also contains the deep inguinal nodes. A surgeon dissecting behind the right main bronchus is likely to encounter the Incorrect vagus nerve TrueFalse Incorrect phrenic nerve TrueFalse Incorrect recurrent laryngeal nerve TrueFalse Incorrect hemiazygos vein TrueFalse Incorrect azygos vein TrueFalse The azygos vein arches superiorly over the right bronchus. The vagus nerve lies just posterior to the right main bronchus, whereas the phrenic nerve is anterior. Complete rectal prolapse Incorrect Involves all layers of the rectal wall TrueFalse

Incorrect Is common in infants TrueFalse Incorrect Is commoner in men TrueFalse Incorrect May be complicated by rectal gangrene TrueFalse Incorrect May be treated by DeLormes procedure TrueFalse Partial rectal prolapse occurs in children. Complete rectal prolapse is a disease of elderly women. DeLormes procedure (excision of the mucosal component of the prolapse and plication of the muscle from below) has a low morbidity but a high incidence of incontinence and recurrence. Open or laparoscopic rectopexy is the procedure of choice. Regarding massive haemothorax, which of the following are true? Incorrect Results from a rapid accumulation of more than 1500 ml of blood in the chest cavity TrueFalse Incorrect Is commonly associated with distended neck veins TrueFalse Incorrect Results in a dull percussion note over the affected side hemi-thorax TrueFalse Incorrect Immediate evacuation of 1500 ml of blood is an indication for early thoracotomy TrueFalse Incorrect Emergency thoracotomy is necessary in about 80% of patients TrueFalse A haemothorax may result from a blunt (deceleration injury) or penetrating injury (disruption of the systemic or hilar vessels) to the thoracic cavity. Massive haemothorax results from the rapid accumulation of more than 1500 ml of blood or one-third or more of the patients blood volume in the chest cavity. Distension of neck veins is rare; they are usually flat secondary to severe hypovolemia. Rarely will the mechanical effects of massive intrathoracic blood shift the mediastinum enough to cause distended neck veins or a tracheal shift. The neck veins, however, may be distended if there is an associated tension pneumothorax. The important signs and symptoms of a massive haemothorax include hypoxia, chest pain, decreased chest expansion, absence of breath sounds on the affected side and percussion dullness over the affected hemithorax. Chest tube placement to decompress the chest cavity, along with simultaneous restoration of blood volume, is the first step in the management of massive traumatic haemothorax. Blood from the chest tube should be collected in a device suitable for autotransfusion. If 1500 ml is immediately evacuated, it is highly likely that the patient will require an early thoracotomy. In addition, patients who have an initial volume output of less than 1500 ml but continue to bleed (200 ml/h for 24 h) also require a thoracotomy. The decision should be made in such patients based on their physiological status rather than the rate of continuing blood loss. The majority of the patients can be managed conservatively with appropriate fluid resuscitation and chest decompression. Emergency thoracotomy is required in only about 10% of patients with massive haemothorax. Reply to HalaReport

Post #25 Hala Adel wrote5 hours ago Consider the male external genitalia Incorrect The corpora carvenosa forms the glans penis TrueFalse Incorrect The corpus spongiosum is a continuation of the crura of the penis TrueFalse Incorrect The paraurethral glands of Littr are adjacent to the penile urethra and function to lubricate the

penile urethra TrueFalse Incorrect Bucks fascia encloses the three erectile bodies of the penis TrueFalse Incorrect Fibromatosis of Bucks fascia may cause Peyronie disease TrueFalse The corpus spongiosum is situated at the ventral part of the penis, is a continuation of the bulb of the penis and forms the glans penis. The two corpora cavernosa are extensions of the crura and are situated on the dorsal aspect of the penis. Bucks fascia encloses the erectile bodies, the dorsal arteries and veins, and the dorsal nerves of the penis. Varicose veins Incorrect are most common in patients of Mediterranean origin TrueFalse Incorrect are more common in patients engaged in occupations involving long periods of standing TrueFalse Incorrect seen in pregnancy tend to regress after parturition TrueFalse Incorrect may be associated with a previous history of deep vein thrombosis TrueFalse Varicose veins affect females of Northern European origin five times more commonly than men and are particularly associated with previous deep vein thrombosis (DVT). Veins are most pronounced in patients who stand for long periods. Which of the following are correct? The right atrium Incorrect Is separated externally by the crista terminalis TrueFalse Incorrect Is separated internally by the sulcus terminalis TrueFalse Incorrect The crista terminalis extends between the two vena caval orifices. TrueFalse Incorrect It contains the fossa ovalis in its anterolateral wall TrueFalse Incorrect The opening of the coronary sinus contains a valve TrueFalse The two parts of the right atrium are separated externally by a groove on the posterior aspect of the atrium known as the sulcus terminalis and internally by the crista terminalis, which extends between the two vena caval orifices. The fossa ovalis is found on the interatrial septum, which forms the posteromedial wall of the right atrium. The opening of the coronary sinus is guarded by a semicircular valve that closes the orifice during contraction of the right atrium. Which of the following are correct? Parotid neoplasms Incorrect Are commonly bilateral TrueFalse Incorrect Are malignant in 50% TrueFalse Incorrect Presentation with facial nerve palsy implies malignancy TrueFalse Incorrect Usually present with maxillary numbness TrueFalse Incorrect

Magnetic resonance imaging (MRI) rather than computed tomography (CT) is the radiological investigation of choice for staging of parotid carcinomas TrueFalse Parotid neoplasms are usually unilateral, only 1520% of parotid tumours are malignant. The main mode of presentation is a symptom-less swelling often dating back several years. Presentation with facial nerve palsy is very suggestive of a malignancy. Rare presentations include trismus and referred pain via the trigeminal nerve. MRI is used for radiological staging as there is better soft tissue discrimination, imaging can be carried out in multiple planes and it is easier to detect cervical lymphadenopathy. Which of the following are correct? Cardiac surgery Incorrect Is performed with controlled hypertension TrueFalse Incorrect May be complicated by cardiac tamponade TrueFalse Incorrect Rarely results in postoperative arrhythmias TrueFalse Incorrect Always requires cardio-pulmonary bypass TrueFalse Incorrect Routine coronary artery bypass grafting (CABG) is associated with a mortality rate of 6% TrueFalse Cardiac surgery is performed with controlled hypotension, together with hypothermia. The hypothermia is used to decrease cellular metabolism and reduce energy requirements of the tissues. Cardiac tamponade is a well known complication of cardiac surgery, it usually presents in the early post-operative period with deteriorating cardiac function and cardiac arrest. CABG can sometimes be performed without cardiopulmonary bypass (off pump). Arrhythmias (usually atrial fibrillation) occur in 2040% of patients following surgery. Routine CABG is associated with a mortality rate of around 2%. The external jugular vein Incorrect receives a branch from the retromandibular vein TrueFalse Incorrect lies anterior to scalenus anterior TrueFalse Incorrect joins the subclavian vein TrueFalse Incorrect has no valves TrueFalse Incorrect pierces the deep cervical fascia TrueFalse The external jugular vein drains most of the scalp and side of the face. It begins near the angle of the mandible and is formed from the union of retromandibular and postauricular veins, receiving branches from the posterior external and transverse cervical veins. The external jugular vein has two pairs of valves which do not prevent regurgitation of the blood, or the passage of injection from below upward. The lower pair are placed at its entrance to the subclavian vein, the upper (in most cases) about 4cm above the clavicle. The external jugular vein lies anterior to scalenus anterior and pierces the deep fascia of the neck, usually posterior to the clavicular head of the sternocleidomastoid muscle before draining into the subclavian vein. Which one of the following is correct? The axillary artery Incorrect Gives off no branches in its first part TrueFalse Incorrect Is the continuation of the external carotid TrueFalse

Incorrect Is encompassed by the first digitation of serratus anterior TrueFalse Incorrect Is invested in a fascial sheath TrueFalse The first part of the axillary artery gives off one branch the superior thoracic. The axillary artery is the continuation of the subclavian. The upper part of serratus anterior lies posterior to the first part of the axillary artery. It is invested in a fascial sheath arising from the prevertebral fascia. Regarding traumatic aortic disruption, which of the following are true? Incorrect Immediate death may be prevented by pseudoaneurysm contained by the adventitial layer TrueFalse Incorrect Could lead to deviation of the trachea to the right TrueFalse Incorrect The aortic knob may be obliterated in plain radiography TrueFalse Incorrect Transoesophageal echocardiography is the gold standard in the diagnosis of this condition TrueFalse Incorrect In an unstable patient, helical contrast-enhanced computed tomographic (CT) scan is the most appropriate first-line investigation TrueFalse Traumatic aortic disruption, a time-sensitive injury, is a common cause of sudden death after an automobile collision or a fall from great height. A complete tear through the tunica intima, media and adventitia usually leads to rapid exsanguination and death. In aortic rupture survivors, immediate death is prevented due to the vascular continuity maintained by a pseudoaneurysm within an intact adventitial layer or a mediastinal haematoma. A large mediastinal haematoma may shift the trachea to the right. This condition has a variable course ranging from a relatively clinically silent period due to the contained rupture (pseudoaneurysm), to rupture of the pseudoaneurysm, exsanguination and death. Radiographic findings may include a widened mediastinum, obliteration of the aortic knob, deviation of the trachea to the right, obliteration of the space between the pulmonary artery and the aorta (obscuration of AP (aorto-pulmonary) window), depression of the left main stem bronchus, deviation of the oesophagus (nasogastric tube) and fractures of the first or second rib or scapula. False-positive and false-negative findings occur with each radiographic sign and, rarely (12%), no mediastinal or initial chest Xray abnormality is present in patients with great vessel injury. Although transoesophageal echocardiography is a useful, less invasive diagnostic tool, aortography is the gold standard in the diagnosis of blunt aortic rupture. Helical contrast-enhanced computed tomography (CT) of the chest is also an accurate screening method for patients with suspected blunt aortic injury. However, a patient who is haemodynamically abnormal should not be placed in a CT scanner. In stable patients, if enhanced helical CT of the chest is negative for mediastinal haematoma and aortic rupture, no further diagnostic imaging is necessary. If it is positive for blunt aortic rupture, the extent of the injury can best be ascertained by aortography The lesser omentum Incorrect is supplied by gastroepiploic arteries TrueFalse Incorrect is attached to the liver in the fissure of the ligamentum venosum TrueFalse Incorrect encloses the right gastric vessels TrueFalse Incorrect has the common hepatic bile duct in its free edge TrueFalse Incorrect is attached to the first part of the duodenum TrueFalse

Incorrect has considerable mobility TrueFalse The right and left gastric arteries supply the lesser omentum as they lie between its two peritoneal layers. The free edge of the lesser omentum is attached to the first 2 cm of the first part of the duodenum below and the fissure of the ligamentum venosum above. The common hepatic duct is joined by the cystic duct to form the common bile duct in the free edge of the lesser omentum. The greater omentum is quite mobile. The basilic vein Incorrect begins on the medial side of the dorsal venous arch TrueFalse Incorrect drains into the subclavian vein TrueFalse Incorrect is accompanied by the medial cutaneous nerve of the forearm TrueFalse Incorrect pierces the deep fascia in the arm TrueFalse Incorrect lies medial to the biceps tendon in the cubital fossa TrueFalse The basilic vein is a continuation of the ulnar stem of the dorsal venous arch in the hand. It lies medial to the biceps tendon in the cubital fossa and is medial to the medial cutaneous nerve of the forearm in the arm. The basilic vein ascends in the superficial fascia on the medial side of the biceps. It then pierces the deep fascia in the middle of the upper arm, is joined by the venae comitantes of the brachial artery, and becomes the axillary vein at the lower border of the teres major muscle. Reply to HalaReport

Post #26 Hala Adel wrote5 hours ago The diaphragm Incorrect contracts during forced expiration TrueFalse Incorrect is partially supplied with motor fibres by the intercostal nerves TrueFalse Incorrect has its central tendon pierced by the inferior vena cava TrueFalse Incorrect has its costal components derived mainly from the septum transversum TrueFalse Incorrect has its left dome at a higher level than its right dome TrueFalse The central part of the diaphragm is mainly derived from the septum transversum, whereas its periphery has body wall components. The intercostal nerves provide sensory fibres to the peripheral parts of the diaphragm, but the phrenic nerves supply all the motor fibres. The diaphragm contracts during inspiration but relaxes during expiration and is displaced upwards by raised intra-abdominal pressure. The caval hiatus lies in the central tendon, whereas the oesophageal hiatus is surrounded by muscle fibres of the diaphragmatic crura. Squamous-cell carcinoma of the oral cavity Incorrect Are more common on the Asian subcontinent TrueFalse Incorrect Cervical lymph node involvement is usually treated with radiotherapy TrueFalse

Incorrect Leucoplakia is premalignant TrueFalse Incorrect Radiotherapy is usually not required after complete surgical excision TrueFalse Incorrect Does not occur in non-smokers TrueFalse Squamous cell carcinoma (SCC) of the mouth is related to betel nut chewing, common in Asia. Lymph node metastasis is treated with radical neck dissection, usually followed by radiotherapy. Similarly primary excision is usually followed by radiotherapy. Leucoplakia is a risk factor, especially if associated with severe epithelial dysplasia. Due to the genetic basis of cancer, anyone is at risk; however it is rare in non-smokers. The ophthalmic artery Incorrect is a branch of the internal carotid artery TrueFalse Incorrect enters the orbit through the superior orbital fissure TrueFalse Incorrect supplies the ethmoidal air sinuses TrueFalse Incorrect supplies the cornea TrueFalse Incorrect supplies the skin of the forehead TrueFalse The ophthalmic artery is a branch of the internal carotid artery. It passes through the optic canal and supplies the ethmoidal air cells, part of the lateral wall of the nose, external nose, eyelids and forehead. It also supplies all the muscles of the orbit. Which of the following are correct? Medial ligament of the ankle Incorrect Has three separate bands TrueFalse Incorrect The deep part is longest TrueFalse Incorrect Inserts into the calcaneum TrueFalse Incorrect Has a superficial part TrueFalse Incorrect Can be seen on X-ray analysis TrueFalse The medial, or deltoid ligament of the ankle is attached at its apex to the medial malleolus. Below, the deep fibres are attached to the non-articular area on the body of the talus. The superficial fibres are fan shaped and extends to the tuberosity of the navicular, the spring ligament, the sustentaculum tali and the posterior tubercle of the talus. Which of the following are correct? Indication for resection of a lung tumour Incorrect Is precluded by a malignant pleural effusion TrueFalse Incorrect Is contraindicated if ipsilateral hilar lymph nodes are involved TrueFalse Incorrect Is curative in 60% of squamous carcinoma TrueFalse

Incorrect Includes palliation TrueFalse Incorrect Includes small cell carcinoma TrueFalse A lung tumour must be sufficiently localised to be suitable for resection. Involvement of ipsilateral hilar lymph nodes is not usually a contraindication to resection but the presence of malignant cells in a pleural effusion is. The 5-year survival rate following complete resection of non-small-cell lung cancer is in the region of 3040%. Small-cell lung cancers (also known as oatcell carcinoma) are highly malignant tumours that are usually disseminated at presentation. For the majority of patients chemotherapy is the treatment of choice, small-cell lung cancers are rarely suitable for surgical management. The transpyloric plane: Incorrect Lies mid-way between the jugular notch and the pubic symphysis TrueFalse Incorrect Lies at the level of the first lumbar vertebra TrueFalse Incorrect Defines the level at which the coeliac axis leaves the aorta TrueFalse Incorrect Is the plane where the portal vein is formed TrueFalse Incorrect Crosses the right costal margin at the tip of the ninth costal cartilage, the surface marking of the gall-bladder fundus TrueFalse The transpyloric plane is a convenient way to relate anatomical structures. It is an imaginary transverse plane with a surface marking mid-way between the jugular notch and the pubic symphysis. This corresponds to the level of the first lumbar vertebra. The surface marking of the fundus of the gall-bladder is at its junction with the right costal cartilage. It represents the point at which the superior mesenteric artery leaves the aorta and where the splenic and superior mesenteric veins join to form the portal vein. The medial ligament of the ankle Incorrect comprises three separate bands TrueFalse Incorrect is damaged in a sprained ankle TrueFalse Incorrect inserts into the calcaneum TrueFalse Incorrect has a superficial part TrueFalse Incorrect may be associated with an avulsion fracture on X-ray TrueFalse The medial ligament of the ankle, otherwise known as the deltoid ligament, has two layers. The deep part is narrow and much shorter than the superficial part, which is triangular in shape. The superficial part of the medial ligament is attached to the borders of the tibial malleolus, and has a continuous attachment from the medial tubercule of the talus along the edge of the sustentaculum tali and spring ligament to the tuberosity of the navicular bone. The lateral ligament consists of three separate bands, and it is this ligament which is usually damaged in inversion injuries (a sprain) of the ankle. The ligaments themselves cannot be seen on X-ray, although avulsion fractures may be detectable on X-ray. Portal hypertension may be caused by Incorrect Pylephlebitis after acute appendicitis TrueFalse

Incorrect Splenectomy TrueFalse Incorrect Tricuspid valve incompetence TrueFalse Incorrect Alcoholic cirrhosis TrueFalse Incorrect BuddChiari syndrome TrueFalse Portal hypertension with a pressure of over 20 mmHg is commonly caused by prehepatic problems such as portal vein thrombosis, hepatic disease such as cirrhosis and post hepatic problems such as tricuspid valve incompetence and BuddChiari syndrome of hepatic vein thrombosis. Which of the following are true? The right suprarenal gland Incorrect Has a longer vein than the left suprarenal gland TrueFalse Incorrect Receives an arterial supply directly from the aorta TrueFalse Incorrect Is crescentic in shape TrueFalse Incorrect Touches the bare area of the liver TrueFalse Incorrect Lies anterior to the inferior vena cava TrueFalse The left suprarenal vein is longer than the right, entering the left renal vein. Both glands receive an arterial supply directly from the aorta, as well as from the renal and inferior phrenic arteries. The right suprarenal gland is pyramidal in shape; the left is crescentic in shape. The anterior surface of the right suprarenal gland is overlapped medially by the inferior vena cava. Structures superficial to the sternocleidomastoid muscle include the Incorrect transverse cervical nerve TrueFalse Incorrect transverse cervical artery TrueFalse Incorrect great auricular nerve TrueFalse Incorrect external jugular vein TrueFalse Incorrect inferior thyroid artery TrueFalse The transverse cervical nerve emerges as a single trunk behind the posterior border of the sternocleidomastoid and is superficial to the muscle. The transverse cervical artery is found in the posterior triangle of the neck just above the clavicle. The great auricular nerve (C2C3) is a large trunk that passes vertically upwards over the sternocleidomastoid. The external jugular vein commences behind the angle of the mandible, formed by the union of the posterior auricular vein and the posterior division of the retromandibular vein. It descends obliquely across to the sternocleidomastoid and drains into the subclavian vein. For basal cell carcinoma (BCC) of the face, which one of the following is true Incorrect Is the commonest malignant facial skin tumour TrueFalse Incorrect

Basal cell carcinomas commonly metastasise TrueFalse Incorrect Radiotherapy is the current treatment of choice for basal cell carcinomas TrueFalse Incorrect Small BCCs (less than 1 cm) should be excised with a maximum margin of 5 mm to improve cosmetic appearance TrueFalse Incorrect Aggressive recurrence may occur with BCCs on the forehead if the disease is not eradicated sufficiently. TrueFalse BCC is the commonest skin tumour on the face and 90% of all BCCs occur in the head and neck region. Sunlight exposure and genetic factors are the main risk factors. BCCs rarely, if ever, metastasise and treatment is with complete surgical excision. Radiotherapy is reserved for recurrences, which are typically aggressive if they recur on the cheeks, nasolabial folds, medial canso and preauricular region. The phrase rather a large scar than a small tomb (Sir Harold Gillies) should always be taken into consideration when planning surgical excision tumours under 1 cm should have at least a 5-mm excision margin and those over 1 cm should have at least a 1-cm margin. In the investigation of varicose veins Incorrect a negative A sound using Doppler ultrasound (U/S) signifies valvular incompetence TrueFalse Incorrect Doppler U/S has a sensitivity of up to 80% TrueFalse Incorrect ascending phlebography is the most valuable technique for identifying perforators TrueFalse Incorrect varicography has little value in the assessment of vessels with unusual anatomy TrueFalse In Doppler ultrasound (U/S), the A sound is produced by squeezing the calf. When pressure is released there is no sound if the valves are competent, this is termed a positive A sound. Doppler U/S has a sensitivity of up to 97%; 87% of perforating veins are localised by ascending phlebography. The true vocal folds are Incorrect lined by respiratory epithelium TrueFalse Incorrect formed by the lower free edge of the quadrangular membranes TrueFalse Incorrect abducted by the lateral cricoarytenoid muscles TrueFalse Incorrect abducted by the posterior cricoarytenoid muscles TrueFalse Incorrect tensed by contractions of the cricothyroid muscles TrueFalse Incorrect innervated by sensory fibres of the internal laryngeal nerves TrueFalse The true vocal folds have a stratified squamous epithelium, innervated by the recurrent laryngeal branch (CN X), and are formed by the vocal ligament (the free edge of the quadrangular membrane forms the false vocal cord). Above the vocal cords, the larynx is sensorily innervated by the internal laryngeal nerve (CN X). The cords are adducted by the lateral cricoarytenoid muscle, abducted by the posterior cricoarytenoid and tensed by tilting the thyroid cartilage downwards and forwards by contracting the cricothyroid muscle. All the laryngeal muscles are supplied by the recurrent laryngeal nerve except for cricothyroid, which is supplied by the external laryngeal nerve.

Which of the following are true? The parotid duct Incorrect Is approximately 1 cm long TrueFalse Incorrect Crosses the masseter TrueFalse Incorrect Is compressed by buccinator TrueFalse Incorrect Conveys mainly mucous secretions TrueFalse Incorrect Lies on the middle third of a line between the intertragic notch of the auricle and the mid-point of the philtrum TrueFalse The parotid duct is approximately 5 cm long. It crosses the masseter, turning around its anterior border to pass through the buccal fat pad and pierce the buccinator. When intraoral pressure is raised the submucous part of the parotid duct is compressed by the buccinator. The parotid gland is mainly a serous gland. Reply to HalaReport

Post #27 Nezar Mohamed wrote5 hours ago Reply to NezarReport

Post #28 Hala Adel wrote5 hours ago An upper midline laparotomy involves incising: Incorrect The linea alba TrueFalse Incorrect The rectus abdominis TrueFalse Incorrect The transversus abdominis TrueFalse Incorrect The transversalis fascia TrueFalse Incorrect The visceral peritoneum TrueFalse A midline laparotomy is not a muscle cutting incision. The incision passes through the skin, subcutaneous flat, the linea alba, extraperitoneal fat, transversalis fascia and parietal peritoneum. The (common) bile duct Incorrect lies to the left of the hepatic artery in the lesser omentum TrueFalse Incorrect is supplied by the cystic artery TrueFalse Incorrect usually opens into the duodenum separately from the pancreatic duct TrueFalse

Incorrect crosses in front of the neck of the pancreas TrueFalse Incorrect passes anterior to the right renal vein TrueFalse The common bile duct lies to the right of the hepatic artery. It joins the pancreatic duct at the ampulla of Vater. The ampulla itself usually opens into the duodenum. It crosses a groove between the head of the pancreas and the second part of the duodenum, in front of the right renal vein. The facial nerve Incorrect pierces buccinator muscle TrueFalse Incorrect is the main supplier of the muscles of mastication TrueFalse Incorrect is involved in taste TrueFalse Incorrect includes the submandibular branch as one of the three main divisions originating from within the parotid gland TrueFalse Incorrect emerges from the skull through the stylomastoid foramen TrueFalse The buccal branch of the nerve does pierce buccinator muscle after supplying it and the muscles of the upper lip. The trigeminal (V) nerve is predominantly associated with the muscles of mastication. These include the masseter, temporalis and pterygoid muscles. There are five, not three, main divisions of the nerve originating from the parotid gland, and the lowest or fifth branch is the cervical. The facial nerve does pass through the stylomastoid foramen A severely displaced Colles fracture Incorrect may lead to delayed rupture of the extensor pollicis longus tendon TrueFalse Incorrect has the distal radius displaced in a volar direction TrueFalse Incorrect usually requires 6 weeks immobilisation TrueFalse Incorrect more commonly requires external fixation in the older rather than the younger patient TrueFalse Incorrect can be associated with Sudeks atrophy TrueFalse Rupture of the extensor pollicis longus tendon may occur as a late complication of a displaced Colles fracture. A Smiths fracture is the reverse Colles fracture where the distal segment is palmar flexed rather than dorsiflexed. Most Colles fractures are treated in plaster for 6 weeks, but in young patients it may be necessary to restore normal alignment by internal fixation, especially when cosmetic appearances or type of occupation may be adversely affected by residual deformity or loss of movement. Sudeks atrophy can follow trivial hand injuries and is thought to be related to autonomic dysfunction. About the knee Incorrect the popliteal muscle is intracapsular TrueFalse Incorrect the medial longitudinal ligament is attached to the medial meniscus TrueFalse

Incorrect the menisci are covered in synovial membrane TrueFalse Incorrect the anterior cruciate ligament is attached to the medial condyle TrueFalse Incorrect the posterior cruciate ligament is stretched when the knee is in full extension TrueFalse * The popliteal tendon is intracapsular. * The medial collateral ligament is attached to the medial left meniscus, which is thus less mobile than the lateral meniscus. * There is no synovial membrane over the articulating surfaces of synovial joints. * The anterior cruciate ligament is attached to the lateral femoral condyle. * Both cruciate ligaments are tense in full extension. Consider the hip joint Incorrect it is a hinge joint TrueFalse Incorrect a thick and tight fibrous capsule increases stability TrueFalse Incorrect the quadratus femoris is a lateral rotator of the hip TrueFalse Incorrect the iliofemoral ligament prevents overextension of the hip joint TrueFalse Incorrect the rim of the acetabular labrum increases hip stability TrueFalse The hip joint is a ball and socket synovial joint. The capsule is thicker and tighter than that of the shoulder joint. The acetabular labrum encloses the femoral head beyond its equator, increasing the stability of the joint. Which of these muscles are commonly innervated by the obturator nerve? Incorrect Gracilis TrueFalse Incorrect Semimembranosus TrueFalse Incorrect Pectineus TrueFalse Incorrect Adductor longus TrueFalse Incorrect Obturator internus TrueFalse The obturator nerve (L2, L3, L4) exits the obturator foramen and divides into anterior and posterior branches. The anterior branch innervates adductor brevis, adductor longus, gracilis and pectineus. The posterior branch innervates obturator externus and part of adductor magnus. Semimembranosus is one of the hamstring muscles and is innervated by the tibial portion of the sciatic nerve. Obturator internus is innervated by the nerve to obturator internus, which, also supplies the superior glamellus. Concerning the axillary artery: Incorrect it begins at the medial border of the first rib TrueFalse Incorrect it ends at the inferior border of the teres major TrueFalse Incorrect

the pectoralis minor divides the axillary artery into three parts TrueFalse Incorrect the first part of the axillary artery gives off the thoracoacromial artery TrueFalse Incorrect the lateral thoracic artery branches from the second part of the axillary artery and is larger in men than in women TrueFalse The axillary artery begins at the lateral border of the first rib as the continuation of the subclavian artery, and ends at the inferior border of the teres major. For descriptive purposes the axillary artery is divided into three parts by the pectoralis minor. The first part has one branch (superior thoracic), the second part gives off two branches (thoracoacromial and lateral thoracic), and the third part gives off three branches (subscapular, anterior circumflex humeral, and posterior circumflex humeral). The lateral thoracic artery is larger in women, and it supplies the lateral part of the mammary gland. The recurrent laryngeal nerve Incorrect supplies all intrinsic laryngeal muscles TrueFalse Incorrect supplies the cricothyroid muscle TrueFalse Incorrect supplies sensation to the subglottic region TrueFalse Incorrect is sensory to the supraglottic region TrueFalse Incorrect supplies the sternothyroid muscle TrueFalse The recurrent laryngeal nerves are sensory to the subglottic region and supply all the intrinsic muscles except the cricothyroid muscle. Phaeochromocytomas: Incorrect are usually malignant TrueFalse Incorrect are associated with multiple endocrine neoplasia type 2a (MEN2a) TrueFalse Incorrect are rarely bilateral TrueFalse Incorrect are associated with multiple endocrine neoplasia type 1 (MEN1) TrueFalse Incorrect secrete aldosterone TrueFalse Phaeochromocytomas are tumours of chromaffin tissue that secrete catecholamines. Patients present with hypertension and sympathetic hyperactivity. 10% of such tumours are malignant, 10% are bilateral, 10% are extra-adrenal and 10% are familial. They are associated with MEN2a and 2b. Structures related to the superficial part of the submandibular gland include Incorrect platysma TrueFalse Incorrect the mandibular branch of the facial nerve TrueFalse Incorrect

the facial artery TrueFalse Incorrect the facial vein TrueFalse Incorrect deep cervical fascia TrueFalse The submandibular gland is a lobulated gland made up of a superficial and a deep part, which are continuous with each other around the posterior border of the mylohyoid muscle. Part of the gland lies inferolaterally, enclosed in an investing layer of deep cervical fascia, platysma muscle and skin. Laterally it is crossed by the cervical branch of the facial nerve and vein. The facial artery is related to the posterior and superior aspects of the superficial part of the gland. Which of the following are correct? The following structures would be encountered during a carotid endartectomy Incorrect Hypoglossal nerve TrueFalse Incorrect Omohyoid TrueFalse Incorrect Facial vein TrueFalse Incorrect Superior thyroid artery TrueFalse Incorrect Internal jugular vein TrueFalse All of the above are near to the carotid arteries and would be expected to be encountered during the course of a carotid endartectomy. Which of the following are correct? Left coronary artery: Incorrect Divides into circumflex and left anterior descending arteries TrueFalse Incorrect Supplies the left atrium TrueFalse Incorrect Supplies the sinoatrial (SA) node in most cases TrueFalse Incorrect Supplies the atrioventricular (AV) node in most cases TrueFalse Incorrect Arises from the anterior aortic sinus TrueFalse The left coronary artery arises from the left posterior aortic sinus behind the pulmonary trunk. After a short course it divides into two main arteries, the circumflex and the left anterior descending, otherwise known as the anterior interventricular artery. Around 60% of hearts have the right coronary artery supplying the SA node and in 40% of hearts the SA nodal artery arises from the left coronary artery. The right coronary artery supplies the atrioventricular (AV) node. The left coronary artery supplies the vast majority of the left ventricle and left atrium. Part of the right ventricle is supplied by the left coronary artery. Bleeding from the middle meningeal artery following head injury Incorrect mainly affects the posterior branch TrueFalse Incorrect results in an extradural haematoma TrueFalse Incorrect may produce ipsilateral pupillary constriction TrueFalse

Incorrect is usually caused by a trivial incident TrueFalse Incorrect typically causes a biconvex-shaped lesion on CT TrueFalse Bleeding from the middle meningeal artery following head injury usually leads to an extradural haematoma. This is usually a tear of the anterior branch of the middle meningeal artery, with an underlying linear skull fracture. The characteristic picture is of a head injury with a brief episode of unconsciousness followed by a lucid interval. The patient then develops a progressive hemiparesis, stupor and rapid transtentorial coning with an ipsilateral dilated pupil. This is followed by bilateral fixed dilated pupils, tetraplegia and death. Which of the following are true? The inguinal canal Incorrect Is about 1.5 cm long TrueFalse Incorrect Has the fascia transversalis along the whole length of its posterior wall TrueFalse Incorrect Has a deep inguinal ring lying 5 cm above the middle of the inguinal ligament TrueFalse Incorrect Has the lacunar ligament in the medial part of its floor TrueFalse Incorrect Has the inferior epigastric artery medial to its deep ring TrueFalse The inguinal canal is about 4 cm long. The posterior wall of the canal has the conjoint tendon medially and the transversalis fascia throughout. The deep inguinal ring lies about 1.25 cm above the mid-point of the inguinal ligament. The floor of the inguinal canal is the unrolled lower edge of the inguinal ligament, re-inforced medially by the lacunar ligament. Which of the following are true? Regarding the spinothalamic tract: Incorrect Conveys vibration and position sense to the brain TrueFalse Incorrect The secondary axons of the tract synapse in the thalamus TrueFalse Incorrect Axons from the lumbar region synapse medially TrueFalse Incorrect Lesion to this tract could lead to loss of pain sensation on the opposite side of the lesion TrueFalse Incorrect Lesion to this tract could lead to diminished temperature and touch sensation from the same side of the body as the lesion TrueFalse The spinothalamic tract conveys pain, temperature, touch and pressure sensations from one side of the body to the opposite side of the brain. Vibration and position sense are conveyed via the posterior column. The first neurone of the spinothalamic tract synapses in the posterior horn; the next neurone crosses to the right side of the spinal cord and synapse in the thalamus, after ascending through the cord and brainstem; the third neurone arises in the thalamus to pass to the cortex. The secondary axons of the spinothalamic tract ascend through the brainstem to

synapse in the thalamus. Axons from the cervical region synapse medially while axons from the lumbar region synapse laterally. A lesion of the spinothalamic tract anywhere in the brainstem would lead to a loss of pain sensations from the opposite side of the body. Temperature and touch sensations would also be diminished from the opposite side of the body but not totally lost because other pathways may also convey these modalities. A lesion of the spinothalamic tract at the level of the spinal cord would lead to loss of pain sensations on the opposite side, beginning .one level below the level of the lesion Reply to HalaReport

92# Post Khalid Ayad (Egypt) wrote2 hours ago Reply to KhalidReport

03# Post Mostafa M.Safwat (Egypt) wroteabout an hour ago 1 MRCS Part .... onexamination.com 06 ... .... .... ---------------------------- 8002 sba 005 mcq 0031 ... ... ... ... mihd rapidshare mihd request ticket .. mihd ): EgyMedicine.Net for free medical books (70,000 members 102561http://www.egymedicine.net/forumsx/t29406.html#post ... IDM flashget dap RESUME 04

3 ... .. http://rapidshare.de/files/39918888/Que.rar.html http://rapidshare.de/files/40132484/Que_2.zip.html http://mihd.net/4nweykf )( Reply to MostafaReport

Reply
Post reply

Advertise

Shoof Kora

More Ads Facebook

Vous aimerez peut-être aussi